CEN

¡Supera tus tareas y exámenes ahora con Quizwiz!

Hemophilia B

aka Christmas disease Factor IX deficiency

What are the s/s of cirrhosis/liver failure? How is it managed?

jaundice; ascites; arterixis; spider angiomas; abdominal pain; dark urine; chronic fatigue; easy bruising; elevated LFTs, bleeding times, alkaline phosphatase, ammonia & direct bilirubin; low albumin & platelets Lactulose (rid ammonia); neomycin (decrease ammonia production); replace calcium, potassium & vitamin K

distributive shock

maldistribution of blood flow & vasodilation. "The Pipe" Either septic, anaphylactic, or neurogenic

What is all included w/the brainstem?

midbrain; pons; medulla Responsible for vital functions & reticular activating system (sleep, wake patterns)

How do you treat tension pneumothorax?

needle decompression & chest tube placement

What is obstructive shock?

obstruction of blood flow; relieve the obstruction

Contralateral

opposite side

vaso-occlusive crisis

pain in extremities; priapism. (All do blood flow becoming OBSTRUCTED) Most common of the crisis' Hydration & warm packs over areas of pain

What is odynophagia?

pain with swallowing

What are the s/s of a brown recluse bite & how is it managed?

painless bite; pruritus, redness & blister in 1-3 hours; bluish gray macule w/halo of pallor & erythema (RED, WHITE, & BLUE) Fever, chills, n/v, malaise within 24 hours of bite NECROTIZING ULCERATING WOUND (tissue sloughing) over time (7-14 days) Tx w/wound care, REMOVAL OF NECROTIZING TISSUE; HBO; ABX; steroids; tetanus immunization

Patient presents with slight epigastric pain initially that begins to increase in intensity & radiates to back; it is noted that they have elevated amylase & lipase. What abdominal organ has been impacted?

pancreas

What are the manifestations of acute radiation syndrome?

pancytopenia; severe nausea; vomiting; abdominal pain; confusion

Hypoactive bowel sounds are noted in?

paralytic ileus

How do you treat Pericardial Tamponade?

pericardiocentesis & surgical repair

What is eclampsia?

preeclampsia + seizures At risk for up to 3 weeks postpartum Tx: same as preeclampsia plus BENZODIAZEPINES Emergent c-section

What is stroke volume influenced by?

preload, afterload, contractility

What is Kussmaul breathing?

rapid, deep, labored breathing; seen w/DKA

Causes of hyperkalemia

renal failure; burns; crush injury; tumor lysis syndrome; rhabdomyolysis; adrenal insufficiency Medications including ACE-I & NSAIDS

What does the Parasympathetic nervous system do?

rest and digest; decreases HR (like vagal response) as seen in neurogenic shock (no SNS, UNOPPOSED PARASYMPATHETIC)

Ipsilateral

same side

What do you need to know about a MESENTERIC INFARCTION?

seen in those with h/o afib. Will have severe abd pain d/t blood supply being cut off & bowel becoming necrotic.

What is proprioception?

sense of body position and movement

What is intussusception? How is it managed?

severe colicky pain; inconsolable; drawing legs up to chest. May palpate sausage shaped mass CURRANT JELLY STOOL Air or contrast edema; surgery if in shock

What is the antidote for TCAs?

sodium bicarbonate

Afterload

the PRESSURE/RESISTANCE against which the ventricle must OVERCOME to OPEN valve & PUMP blood OUT. Measured by SVR (systemic vascular resistance). *Is only LOW in DISTRIBUTIVE SHOCK*

What is stroke volume?

the amount of blood ejected by the heart in any one contraction

What is emphysema?

the destruction of alveoli; "pink puffer"; thin, BARREL CHEST (increased anterior: posterior diameter); pursed-lip breathing; lung over inflation; low diaphragm of CXR

What is pheochromocytoma?

tumor of the adrenal medulla that stimulates release of adrenaline—>tachycardia & hypertensive crisis. Tx'd w/ alpha blocking agent like PHENTOLAMINE (REGITINE); Nitroprusside; labetalol. Betablocker w/o alpha blockage is contraindicated.

What is testicular torsion?

twisting of the spermatic cord

What are examples of sedatives?

barbituates ("-tal"), benzodiazepines ("-pam" "-lam")

What is Guillain-Barre syndrome?

damage to myelin sheath leading to a TINGLING SENSATION IN EXTREMITIES; loss of DTR & ASCENDING SYMMETRICAL WEAKNESS/PARALYSIS Care is supportive

LVAD (left ventricular assist device)

used for pts w/ESHF. Battery operated mechanical pumps which help LV PUMP blood to body. Pt's typically do NOT have palpable peripheral pulses. BP typically not able to be measured by automated cuffs; use Doppler to get MAP. Use manual BP to arm; inflate >120 then slowly deflate while having Doppler probe over brachial artery; pressure reading at which arterial flow become audible/visible is the MAP. Nml 60-90.

Pulse Pressure

difference between systolic and diastolic pressure Narrowed seen in early shock Widened seen in increased ICP

What is pulse pressure?

difference between systolic and diastolic pressure NARROWED is seen in EARLY shock WIDENED is seen in increased ICP

obstructive shock

due to obstruction of blood flow. Relieve obstruction. E.g. Tension pneumothorax; cardiac tamponade; abdominal compartment syndrome.

What are the s/s of LSD ingestion & how is it managed?

euphoria; panic; HALLUCINATIONS; paranoia; psychosis REDUCE STIMULATION; benzodiazepines for agitation; HALDOL for psychosis

What are S/S of mononucleosis & how is it managed?

fatigue; myalgia; LYMPHADENOPATHY; abdominal pain Managed: MONOSPOT (+ in 2nd week of illness); labs (CBC & LFTS); analgesics; corticosteroids; warm salt gargles.

What is dysfunctional uterine bleeding?

fewer than 21 days between bleeding; typically painless Dx: CBC; bleeding times; pelvic/transnational U/s Tx: LOW DOSE ORAL CONTRACEPTIVE THERAPY; iron supplements; treat hypovolemia

What is the occipital lobe responsible for?

vision

Preload

volume of blood or pressure in ventricles at end of diastole. Measured by CVP. (Right atrial pressure)

What is electrical alternans?

when the amplitude of the QRS complex varies from beat to beat.

What is a COLLES fracture?

fracture of the distal radius; aka SILVER FORK DEFORMITY

What is ringworm?

fungal infection of skin, hair or nails that is spread through people & animals via fomites (objects) CIRCULAR, RED, scaly, itchy rash with a CENTRAL CLEARING; can cause bald patches on scalp Tx w/OTC topical antifungals; systemic antifungals (griveofulvin; terbinafine; clotrimazole)

If you see an inferior (or RV) MI what should you do?

get a R sided EKG- ST elevation at V4R at 5th ICS, R. MCL

What is Cardiac Output?

heart rate x stroke volume

cardiac output

heart rate x stroke volume Normal 4-8L per minutue

What are the benefits of autotransfusion?

whole blood w/RBC, plasma, platelets, clotting factors No risk of transfusion reaction Warm & not frozen w/preservatives

What is POIKILOTHERMIA?

inability to regulate temperature

nephrogenic diabetes insipidus

inadequate response to ADH

shock

inadequate tissue perfusion Compensation through SNS & RASS system Tx effective if MAP > 60, increased pulse quality & warm extremities, u/o at least 0.5ml/kg/hr; normalization of BASE DEFICIT &/or SVO2 65-75%

infective endocarditis

inflammation of the endocardium affecting heart valves. Causes: cardiac sx, IV drug use, body piercing. S/s: fever; chills; night sweats; myalgias; NEW onset murmur (d/t vegetation of valves); pleuritic CP; splinter hemorrhages in nail beds; conjunctival petechiae; Osler's nodes (tender nodules on digits); Janeway lesions (macules on palms/soles); Roth spots (retinal hemorrhages) Dx: increased WBC; + blood cultures; increased ESR; echo Tx: IV ABX; possible valvular repair.

What is cholecystitis?

inflammation of the gallbladder

acute pericarditis

inflammation of the pericardium. Causes- viral, neoplasticism, uremic, connective tissue disorder. S/s: sudden onset of SHARP RETROSTERNAL CP that worsens w/INSPIRATION, reclining position, cough, or movement; fever/chills; tachycardia/pnea; pericardial friction rub (best heard over L. Sternal border). EKG: global (diffuse & widespread) CONCAVE ST segment ELEVATION w/PR depression & tall T waves (except aVR) Tx: NSAIDS & steroids (If REFRACTORY to NSAIDS). Leaning forward position of comfort

What are the s/s of GERD & how is it managed?

"Burning" CP; regurgitation; dysphagia d/t lump in throat Antacids; H2 blockers (ranitidine; cimetidine); PPIs (omeprazole; lansoprazole); dopamine antagonist (keto loperamide); acid-protective (sucralfate)

Treatment of hyperkalemia

"C BIG K DROP" Calcium gluconate 10%-10ml IV over 10 minutes (CARDIAC STABILIZER) Beta-agonists- salbutamol 10-20mg in 4ml NS nebulized over 10mins OR Bicarbonate 8.4% (50mEq) IV over 5 mins (intracellular shift) Insulin 10units IVP (intracellular shift) Glucose D50W- 1 amp over 5 mins (MAINTAIN GLUCOSE) Kayexylate 15-30g in 15-30ml (70% sorbitol) PO for GI removal (CHRONIC renal failure) Diuretics- Lasix 40-80mg IVP (renal removal) Rop Renal unit- CRRT for removal. ***Kayexalate- often used; usually useless.

A patient w/a carboxyhemoglobin of >40% could be exhibiting what?

"Cherry red" skin; seizures, coma, arrest

What is bacterial vaginosis

"FISHY" ODOR; GRAYISH DISCHARGE. Dx: clue cells; pH >4.5; + "whiff" test (KOH) Tx: Flagyl PO x 7 days or gel for 5 days; Clindamycin cream for 7 days.

How is ARDS diagnosed & managed?

"Ground glass" appearance or "white out" on CXR Intubate & ventilate w/low tidal volume & increased PEEP; PRONE POSITIONING

cardiogenic shock

"The pump is pooped" LV failure

hypovolemic shock

"The tank". Fill the tank & plug the hole. Loss of circulating volume/blood Direct pressure to site; replace loss w/fluids/blood products

What are Calicum Channel Blockers?

(-Dipine) Control atrial fibrillation & HTN Diltizazem (Cardizem); Norvasc; Nifedipine

What are angiotensin receptor blockers?

(ARBS) (Sartans) Inhibits angiotensin 2 receptors->vasodilation to reduce BP Avapro; Cozaar; Diovan

What complimentary alternative therapies interact w/ANTICOAGULANTS & DM medications?

(Gs): garlic, ginger, GINGKO, GINSENG

I See All Leads

(I)nferior: II, III, aVF (S)eptal: V1-V2 (A)nterior: V1-V4 (L)ateral: I, aVL, V5, V6

What are angiotensin converting enzyme inhibitors?

(Prils) Affect the renin-angiotensin aldosterone system (RAAS) Block conversion of angiotensin 1 to angiotensin 2 Reduce BP Ex: Lisinopril, Lotensin, Vasotec

What happens w/Brown-Sequard Syndrome?

**2 words—2 sides** Transverse hemisection (essentially damage to 1/2 of the spinal cord) Usually d/t stab or GSW Loss of motor function on side of injury (ipsilateral) Loss of pain & temperature on opposite side of injury

What is the normal value for base excess/deficit on an ABG

+/- 2

What numerical grade is given for "Normal" DTRs?

+2

What is Amyotrophic Lateral Sclerosis (ALS)?

- lou gehrig's disease - genetic disorder that leads to progressive loss of voluntary muscle control (grip strength) but retains intelligence/personality

ABCDE you are a such a QTi prolonger

-anti-Arrhythmics (sotalol, amiodarone, procainamide) -antiBiotics (flouroquinolones, macrolides, amingolycosides) -antipsyChotics (haldol, risperidone, Thorazine, geodon) -antiDepressants (SSRIs, TCAs) -antiEmetics (zofran, droperiodol, compazine) *Don't forget hypomagnesemia or hypocalcemia.

What is the normal ICP?

0-15

What is a normal ICP?

0-15mm Hg

What is the pediatric dose for epinephrine?

0.01mg/kg Q 3-5 mins

What is the pediatric dose of atropine & when is it used?

0.02mg/kg for increased vagal tone or primary AV block; may repeat once

What is the pediatric dose of adenosine?

0.1mg/kg; repeat at double dose 0.2mg/kg

What is the synchronized cardioversion dose for pediatrics?

0.5-1j/kg up to 2j/kg

What is the "goal" u/o for burns?

0.5-1ml/kg/hr-adult (1-2ml/kg/hr peds) At least 75-100ml/hr for ELECTRICAL BURNS

How do we give TPA?

0.9mg/kg (max of 90mg) Bolus 10% of dose over 1 minute Remainder 90% as a drip over the next hour (MAKE SURE TO WASTE THE EXTRA PRIOR TO ADMINISTRATION)

In regards to the rules of nines in an adult the genitalia is how much?

1%

What is the rule of PALMS?

1% of the TBSA

What is the IV resuscitation for adults for dehydration?

1-2 L

What is a normal urinary output for an infant?

1-2ml/kg/hr

What is a poor outcome for APGAR?

1-3

What ESI # requires LIFE SAVING INTERVENTIONS immediately?

1. E.g. OD w/RR of 8; unresponsive w/BGL of 20; cardiac/respiratory arrest; trauma requiring fluid/blood

What is a massive transfusion protocol ?

10 units of PRBCS + plasma & platelets

Neonates with hypoglycemia should be given how much dextrose?

10-12.5%

Hypoglycemic neonates should receive what?

10-12.5% dextrose IV

Sutures of the back should be removed when?

10-14 days

When are staples removed from a scalp??

10-14 days

When are the sutures removed for an arm/leg?

10-14 days

When should staples in the scalp be removed?

10-14 days

Measles begin to be contagious when?

10-14 days BEFORE RASH

What is normal intraocular pressure?

10-20

What is seen with chronic toxicity of salicylates?

100mg/kg per day x2+ days Confusion, hallucinations, GI irritation (N/V, hematemesis), hypoglycemia, renal failure

How much fluid do you give for a neonate during resuscitation?

10ml/kg

What is the rate of infusion for IVF for neonates/infants for dehydration?

10ml/kg

How much PRBCs do you give a peds patient? And when?

10ml/kg After 2 boluses if blood loss

In pediatric hemorrhagic shock when do you give blood & how much?

10ml/kg AFTER 2 boluses

How is hypoglycemia treated in a pediatric patient?

10ml/kg D10W 2-4ml/kg D25W

At what time is the uterus palpable during pregnancy?

12-14 weeks.

Normal FHT are ?

120-160

What are normal fetal heart tones?

120-160

When are the sutures removed for an over the joint??

14 days

What size needle is needed for a NEEDLE DECOMPRESSION?

14-16g

What is the treatment for hypoglycemia in a CONSCIOUS patient?

15grams of rapid acting PO glucose; repeat as needed; follow w/a complex carbohydrate

In regards to the rules of nines for an adult the anterior torso and posterior torso are each how much?

18% ** 9% for upper anterior torso; 9% lower anterior torso; 9% upper posterior torso; 9% lower posterior torso

What is the lidocaine pediatric dose & what is it used for?

1mg/kg; refractory VF/pVT

What is the maximum amount of fluid that can be injected into the vastus lateralis of an infant?

1ml

How is the IVF given w/burns regardless of what formula is used?

1st half given over 1st 8 hours from time of burn injury Remaining half over next 16 hours.

How is the fluid given for burns?

1st half of TOTAL VOLUME of fluid over 1st 8 hours from TIME OF BURN INJURY Remaining half over next 16hours

What rib fractures are associated w/great vessel or brachial plexus injury?

1st or 2nd

What ESI should be assigned for a penile fracture?

2

What is flail chest?

2 or more adjacent ribs fractured in 2 or more places resulting in a FREE FLOATING segment w/PARADOXICAL CHEST WALL motion (asymmetry) during respiration; muscle spasm obscures flailing initially Tx: intubation/ventilation w/PEEP; prepare for SURGERY FOR RIB FIXATION; monitor for associated pneumothorax/hemothorax (breath sounds, percussion)

SIRS

2 or more of the following: HR >90; Temp >100.4 or <96.8 RR >20 or PaCO2<32 WBC >12k or <4 or >10% bands

What is the pediatric "dose" for defibrillation?

2-4 joules/kg initially then 4 joules/kg, then 4-10 joules/kg

What is the initial defibrillation range for pediatrics?

2-4j/kg

Defib dose in Peds

2-4j/kg for 1st shock. Up to 10j/kg

What treatment is provided for nerve gas agents? (Like Sarin)

2-PAM & ATROPINE

What ESI # is a HIGH RISK SITUATION?

2. E.g. PE risk; suicidal patient; victim of assault

When should continuous fetal monitoring via cardiotocography be performed? And what are bad signs?

20 weeks; loss of variability & decreased fetal movement is bad

What is the half life of diazepam?

20-40hours ***prolonged monitoring**

What is the rate of infusion for IVF for peds for dehydration?

20ml/kg

What is the treatment of gastroenteritis in a pediatric patient?

20ml/kg IV BOLUSES OF CRYSTALLOIDS; repeat until perfusion improves

How much fluid do you give a peds patient?

20ml/kg over 5-10mins

What are the normal values on an ABG for HCO3?

22-26

A fat embolism is most common when?

24-72hrs s/p long-bone or pelvic fracture

Hypoglycemic children less than 2 years old w/a depressed LOC should receive what?

25% dextrose IV

How is a Right ventricular MI managed?

250cc IVF or DOBUTAMINE for hypotension (increases contractility). Treatment is effective if BP is increasing

What is the defibrillation dose for pediatrics?

2j/kg; repeat at 4j/kg; subsequent shocks 4-10j/kg

The American burn association (modified Brooke formula) calculation for fluid resuscitation is?

2ml/kg/TBSA

The American burn association recommends how much fluid for burns?

2ml/kg/TBSA (thermal burns) 3ml/kg/TBSA for pediatrics 4ml/kg/TBSA for electrical burns

Sutures placed in the face should be removed when?

3-5 days

When are sutures removed for the face?

3-5 days

All assistive devices should be fitted so the arm(s) are bent/flexed to what?

30 degrees

What size chest tube should be used for a hemothorax & where should it be placed?

32-36F. at the 5th ICS AAL or MAL.

What are the normal values on an ABG for PaCO2?

35-45

What is the average incubation period of genital herpes?

4 days

What are the most common rib fractures?

4-9

For a neonate- how many bpm for ventilation during resuscitation?

40-60

What is the Parkland formula?

4ml/kg (instead of 2ml/kg) for adult thermal burn

What is the Parkland Formula for initial resuscitation of a burn patient?

4ml/kg/TBSA

What is the parkland formula for burn fluid resuscitation?

4ml/kg/TBSA

What is the Parkland Formula for initial resuscitation of a burn patient?

4ml/kg/TBSA (adults)

How long does it take for dermabond (wound bonding agent) take till it sloughs off?

5-10 days

What is the antidote for ethylene glycol & methanol toxicity?

5-10% ethanol infusion or FOMEPIZOLE

When should delivery of the placenta occur?

5-10mins s/p infant born

How much fluid do you give a peds patient in cardiogenic shock?

5-10ml/kg over 10-20 mins

What occurs during the 1st stage of Lyme's disease?

5-7 days. Flu like symptoms & RED "BULL'S EYE" RASH (erythema migrans)

What happens in severe toxicity of salicylates?

500mg/kg Metabolic acidosis, hypotension, RENAL FAILURE, seizures

What is the pediatric amiodarone dose?

5mg/kg

What discharge teaching is needed for a pediatric patient w/gastroenteritis?

5ml or 1tsp of ORAL REHYDRATION at frequent intervals. Do not use apple juice since it is hyperosmolar & may worsen diarrhea.

What is the usual time period between exposure to UV light & initiation of keratitis?

6-10 hours

What is normal adult tidal volume? And what is it decreased to in ARDS?

6-10ml/kg/ideal body weight 4-6ml/kg/ideal body weight

What are normal O2 sats using the pre-Ductal (right wrist) at during neonatal resuscitation?

60-65% at 1 minute 90% at 10 minutes

Insulin gtts should be titrate to reduce blood sugar levels by what?

65-125mg/dL/hr

Sutures in the scalp should be removed when?

7 days

When are sutures removed for the scalp/trunk?

7-10 days

How long should adhesive strips (steri strips) be left on for?

7-10 days or until they fall off on their own

What is the pH on an ABG to be COMPENSATED on the ACID side?

7.35-7.37

What are the normal or compensated values for pH w/an ABG?

7.35-7.45

What is the pH on an ABG to be COMPENSATED on the ALKALOSIS side?

7.43-7.45

What is the acute toxic dose of APAP?

7.5g in adults 10g in child >6 yrs

Where should the MAP be kept at with an LVAD patient?

70-90

What makes up the SCAPHOID?

8 carpal bones

Sutures of chest should be removed when?

8-10 days

Sutures of the hands/fingers should be removed when?

8-10 days

What is the best timeframe for a defusing session following a traumatic event?

8-12 hours of event & preferably prior to participants going to bed

Sutures of the forearm/leg/foot should be removed when?

8-14 days

What is an ischemia stoke?

80% from embolus or thrombus formation Hx: uncontrolled HTN; afib; DM; prostatic heart valves Focus on LAST KNOWN WELL & symptom onset S/s: unilateral facial/extremity weakness/paralysis (CONTRALATERAL TO CLOT); DYSARTHRIA (slurred speech); dysphagia (difficulty swallowing, drooling); expressive (difficulty speaking) or receptive (difficulty understanding spoken word) APHASIA; and visual disturbances (homonymous hemianopsia (sees only 1 side of each eye) Dx: Non contrast CT within 45 mins to R/O BLEED (does not show ISCHEMIA) Tx: peripheral rTPA w/in 3-4.5hrs or intra-arterial repercussion or mechanical thrombectomy to revascularize the ischemic penumbra

What are the normal values on an ABG for PaO2?

80-100

In regards to the rules of nines for an adult one anterior leg & posterior leg are each how much?

9%

In regards to the rules of nines in an adult the head/neck are how much?

9% (7%head; 2%neck)

In regards to the rules of nines for an adult the arms are how much?

9% circumferential

What is a normal compartment pressure?

<10

How do you determine MAP?

= ((2x DBP) + SBP)/3

What is a Prolonged QTi?

> 1/2 of the total R-R length

What should the ETCO2 be at for quality BLS?

>10

1st degree HB

>20 sec PR interval

What is an abnormal ICP?

>20mm Hg sustained

At what pressure do you prepare for a fasciotomy?

>30

When is excessive vaginal bleeding considered?

>8 pads/day or >12 tampons/day

What happens in mild toxicity of salicylates?

@150mg/kg N/V, dizziness, tinnitus

What happens in moderate toxicity of salicylates?

@250mg/kg Tachypnea, hyperpyrexia, sweating, agitation

What hepatitis is contracted through the fecal-oral route?

A & E cause the VOWELS FROM THE BOWELS

What HEPATITIS "VOWELS FROM THE BOWEL"?

A & E- FECAL-ORAL; ; food & H2O sources; vaccine available E.g. Missionary Trip scenario

What diagnostic exam is completed FIRST to diagnosis an abdominal injury?

A FAST exam

What is a cardiac contusion?

A blunt cardiac injury from a blunt force to chest; usually from MVC; acts of violence or sports injury that results in damage to heart especially the RIGHT VENTRICLE (due to positioning of the heart) S/s: CP unrelieved by NTG; dysrhythmias (ST, PVCs, VT); ST segment elevation; hypotension; cardiac failure Tx: based on findings; OXYGEN; prepare for admission for CONTINUOUS CARDIAC MONITORING; possible vasopressors & POSITIVE INOTROPES to increase BP

An avulsion injury where the skin is pulled away from the scalp, hands, digits, foot & toes is called?

A degloving injury

If you suspect bloody CSF leakage what do you check for?

A halo

What is a TIA?

A mini stroke. A warning sign. 10-15% will experience a stroke w/in 3 months 50% within next 48hrs Symptoms typically resolve w/in 10-20mins Classified as resolving w/in 24HOURS. Reversible ischemic neurologic deficit (RIND) is a cerebral infarct that lasts >24hrs but <72hrs Manage BP, bgl, & COAGULATION

If you stroke the bottom of your adult patient's foot and their toe's fan upward what was elicited?

A positive Babinski which is an ABNORMAL finding

Erythromycin, HALDOL, TCAs, HYPOMAGNESEMIA, hypocalcemia can all cause what?

A prolonged QTi

What is hypocalcemia?

A serum calcium level less than 8.5 mg/dL Seen in PANCREATITIS; hypoparathyroidism; low albumin; burns; malignancy; & hyperventilation

What is hypercalcemia?

A serum calcium level that exceeds 10.5 mg/dL.; seen in renal disease; prolonged immobility; malignancies; hyperparathyroidism S/s: lethargy; decreased DTRs; constipations; "metallic taste"; RISK OF KIDNEY STONES

What is hypomagnesemia?

A serum magnesium level less than 1.5 mg/dL. Seen in ACUTE/CHRONIC ALCOHOLISM; malnutrition; malabsorption; thiazide & loop diuretics S/s: ventricular dysrhythmias (POLYMORPHIC VENTRICULAR TACHYCARDIA-TORSADES DE POINTES); agitation; hyperreflexia Tx: magnesium sulfate administration 1-2gm IV (rapid if emergency; over 2 hours if non-emergent) or IM depending on severity; monitor for respiratory depression, hypotension, decreased DTR

What is hypokalemia?

A serum potassium level less than 3.5 mEq/L.

What is hyponatremia?

A serum sodium level less than 135 mEq/L; serum osmolality < 275) From vomiting; diarrhea; burns Hypervolemic hyponatremia seen in fluid overload; SIADH; excessive water ingestion S/s: fatigue; diarrhea; RISK OF SEIZURES Tx: if caused by vomiting, diarrhea or burns- 0.9% NS replacement HYPERTONIC SALINE; WATER RESTRICTION

What is hypernatremia?

A serum sodium level that exceeds 145 mEq/dL; serum osmolality 2x Na+ >295 Seen in renal failure = sodium & H20 excess (tx'd w/diuretics & dialysis) & hypovolemic hypernatremia (DKA; HHS; DI) S/s: thirst; dry membranes; orthostatic hypotension Tx the cause; correct slowly w/D5W or 0.45NS to prevent cerebral edema; sodium restriction; vasopressin (ADH) for DI

What is hyperkalemia?

A sreum potassium level that exceeds 5mEq/L.

What is a hypothesis?

A statement of prediction of the relationship between 2 or more variables used to guide research

A vaccine is available for what hepatitis?

A; B

How is a traumatic amputation managed?

ABCs; bleeding control (PRESSURE OR TOURNIQUET TO STUMP); immobilization; ABX; tetanus Prepare early for transfer to appropriate facility for replantation Lightly brush off gross material; GENTLY RINSE WITH STERILE SALINE (NO IODINE) Wrap amputated part in saline moistened gauze (DO NOT SOAK IN SALINE OR IT WILL SWELL) Place amputated part in sealable plastic bag & label w/pt info Place bag on separate bag of ice; AVOID DIRECT CONTACT BETWEEN AMPUTATED PART & ICE

How is rhabdomyolysis treated?

ABCs; large VOLUME OF IVF (6-12 LITERS/24HRS) TO PRODUCE U/O > 100-200ML/HR; 1-2 amps of sodium bicarbonate in NS (URINE ALKALIZATION PH>6.5); loop diuretics; mannitol; hemodialysis

At what fundal height is a fetus considered viable?

ABOVE THE UMBILIUS; 20-24wks

What is the most common cause for using NIPPV?

ACUTE PULMONARY EDEMA

What is carpopedal spasm characterized by?

ADDuction of thumb; flexion of metacarpophalangeal joints; extension of the interphalangeal joints; flexion of the wrist

You tell your patient to give an "A OK" motion to test what nerve?

AIN (branch of the median nerve)

What is the priority for MAXILLARY fractures (LeFort)?

AIRWAY

With a GCS of 8 or < what are you doing?

AIRWAY

What are complications of Diphtheria?

AIRWAY OBSTRUCTION; myocarditis; infertility

What reperfusion dysrhythmias are common with fibrinolysis?

AIVR; Vtach

What is a temporal arteritis?

AKA giant cell arteritis Inflamed temporal artery PALPABLE CORD LIKE Age >50 Throbbing HA in temporal area & JAW PAIN (W/CHEWING); fever; temporary unilateral vision loss Tx w/ STEROIDS

What are common causes of Intrarenal ARF?

AKI; glomerulonephritis; acute tubular necrosis (ATN) from RHABDOMYOLYSIS; meds like NSAIDS & ACE-I. Kidney damage that can progress to CRF

What causes deep penetration until neutralized (requiring large amounts of irrigation), liquification, or saponification?

ALKALIS. (E.g. lye, ammonia, drain cleaners)

How is Pheochromocytoma initially treated?

ALPHA blocker (PHENTOLAMINE)

Where do expect to be fractured if your patient has RACCOON EYES (PERIORBITAL ECCHYMOSIS) & RHINORRHEA

ANTERIOR fracture

How is a pelvic fracture managed?

APPLY PELVIC BINDER OVER GREATER TROCHANTER ASAP FOR PELVIC RING FRACTURES MASSIVE TRANSFUSION PROTOCOL Prepare for embolization (REBOA) or surgery (post op risk of DVT/PE/fat emboli)

How is DI treated?

AQUEOUS VASOPRESSIN IV/SQ; lysine vasopressin spray; DDAVP (desmopressin); fluid replacement. Monitor for cerebral edema

Refractory hypoxemia d/t surfactant loss that causes severe dyspnea & tachypnea is known as?

ARDS

You decrease tidal volume & increase PEEP in what disorder?

ARDS

What education needs to be provided regarding gouty arthritis?

ASA & thiazide diuretics should be avoided (increase uric acid production) Decrease intake of alcohol & purine rich foods (red meat; organ meats; sardines; mussels)

How is a humerus fracture managed?

ASSESS FOR BRACHIAL NERVE INJURY Sling & swath for PROXIMAL; sugar tong splint if MID-SHAFT HUMERAL fracture; surgery for distal humerus fracture

How is cholinergic toxicity managed?

ATROPINE until secretions dry; PRALIDOXIME (2-PAM); benzodiazepines

How is neurogenic shock managed?

AUGMENT VASCULAR TONE WITH FLUIDS; VASOPRESSORS (LEVOPHED) & INOTROPES (DOPAMINE)

What is a potential complication of a hip fracture?

AVASCULAR NECROSIS of the femoral head

What diagnostic test is used for "lost" foreign bodies in the vagina?

Abdominal U/S

What is a ruptured diaphragm?

Abdominal contents herniate into the chest & compress the lungs, heart & vessels S/s: dyspnea; "gurgling"; PERISTALTIC SOUNDS in lower left chest w/PROGRESSIVE SCAPHOID ABDOMEN (anterior abdomen appears sunken & concave); abdominal pain radiates to LEFT SHOULDER (KEHR'S SIGN); elevated left diaphragm on CXR Tx: ABCS. Emergent LAPAROTOMY SURGICAL REPAIR

What is pelvic inflammatory disease?

Abdominal pain & dyspareunia w/vaginal discharge; CERVICAL MOTION TENDERNESS ("chandelier sign") Tx: IV/IM ABX, analgesics; ADMIT IF PREGNANT

How does an anterior hip dislocation present?

Abducted; external rotation

What EKG changes would you see w/an old infarction?

Abnml (DEEP & WIDE) Q wave

What findings are associated w/a diffuse atonal injury?

Abnml posturing; loss of brain stem reflexes; HTN; hyperthermia; excessive sweating

What is DIC?

Abnormal activation of the body's coagulation cascade from trauma, SEPSIS, pancreatitis, HELLP (ob) S/s: bruising, petechiae, purpura, hematuria, end-organ failure

ITP

Abnormal bruising & bleeding. RAPID decrease in platelets after recent viral illness Seen more in peds.

A "friction burn" is known as what & how is it managed?

Abrasion Consider pain control prior to cleaning w/soap & water; apply NON-ADHERENT DRESSING

What are the s/s of diverticulitis?

Abrupt onset of CRAMPY PAIN; localized to the LLQ; a/n/v; alternating episodes of EXPLOSIVE DIARRHEA & SEVERE CONSTIPATION.

What is a panic attack?

Abrupt onset of INTENSE FEAR S/s: sensation of CHOKING or breathlessness, palpations, chest pressure, trembling, paresthesia, "going insane"

The separation from the uterine wall rupturing arterial vessels & leading to hemorrhagic shock is seen with that is seen w/trauma, advanced maternal age, cocaine use is known as?

Abruptio Placentae

Severe tearing back pain & rising fundal height are seen in what?

Abruptio placentae

Patient presents via EMS with abdominal pain that radiates to back with MINIMAL DARK RED VAGINAL BLEEDING & abnormal FHTs. What is possibly the cause? How is it managed?

Abruptio placentae Fluid/blood replacement; turn patient on LEFT SIDE; OB consult; prepare for immediate c-section

What is an ominous sign in asthma?

Absence of wheezing

What is a hemothorax?

Accumulation of blood in pleural space w/respiratory distress & s/o HYPOVOLEMIC SHOCK S/s: SOB; dullness w/percussion; s/s of SHOCK (tachycardia; delayed cap refill; cool & clammy skin) Tx: BLOOD PRODUCTS; chest tube placement at 5-6th ICS; emergent sx if INITIAL DRAINAGE >1500ML; w/200ml/hr over next 2-4hrs

A tear in the Achilles' tendon from a sudden UNEXPECTED DORSIFLEXION (SPRINTER or bball player who hears "pop" when pushing off) is called?

Achilles' tendon rupture

What injury can be caused by forced or unexpected DORSIFLEXION?

Achilles' tendon rupture

What is an adverse effect of Cipro?

Achilles' tendon rupture

What causes limited penetration d/t immediate scarring?

Acids

Disseminated Intravascular Coagulation (DIC)

Acquired dysfx of clotting system leading to consumption of factors Triggers: OB complications (e.g. abruptio placenta); sepsis; pancreatitis; trauma S/s: petechiae; purpura; AMS Dx: INCREASED PT, PTT, d-dimer & fibrin split products (what breaks down clotting factors); DECREASED platelets, fibrinogen, H/H. **everything you want in blood is GONE** Tx: correct the trigger; O2; fluids; pRBCs; FFP; Platelets. **better if platelets increasing**

How is APAP toxicity managed?

Activated charcoal if w/in 2 hrs ingestion; pt's 4 hour level is at risk level on nomogram. N-ACETYLCYSTEINE (ACETADOTE) is administered PO/IV to replenish essential enzymes

How is salicylate toxicity managed?

Activated charcoal initially & repeated Gastric lavage (only if pt presents w/in 1 hr & level >500mg/kg) URINARY ALKALIZATION (add 100mEq of sodium bicarbonate to each liter of IVF & infuse at 200-300ml/hr) Hemodialysis for severe toxicity Benzos for seizures activity

What is involved during the mitigation phase of disaster management?

Activities taken to reduce recognized vulnerability

What is aqueous humor the can't move into anterior chamber—>INCREASE IN IOP, compression of CN 2 (OPTIC NERVE) & can lead to blindness w/in hours if left untreated?

Acute Angle Closure Glaucoma

What is bronchiolitis? How is it managed?

Acute LOWER respiratory illness from a virus (RSV; rhinovirus; influenza) Common in <2 yrs (esp preemies) 1-3 day h/o URI w/cough & rhinitis; progresses to respiratory distress w/wheezing & crackles Nasal suctioning w/bulb syringe. Saline drops.

A viral infection (d/t influenza A or B, RSV, Coxsackie, etc) that causes a dry cough, sore throat, stuffy nose, fatigue, & low grade fever is known as? How is it managed?

Acute bronchitis Adequate hydration, avoid smoke, use a vaporizer, cough medication. ABX ARE NOT NEEDED unless secondary infection present

What is the unopposed alpha stimulation?

Acute increase in BP and/or worsening coronary artery vasoCONSTRICTION after administration of a betablocker (if given to patients who recently did cocaine e.g.)

What is meningitis?

Acute inflammation of the meninges from virus/bacteria (group B streptococcus or Neisseria meningitides- meningococcal) S/s: altered LOC; fever; HA; nuchal rigidity; photophobia; KERNIG'S & BRUDZINSKI'S signs Tx: assume bacterial & institute ISOLATION immediately. ABX stat. Assist w/LP (side lying position) BACTERIAL CSF-high pressure, cloudy, LOW GLUCOSE.

What is delirium?

Acute onset & inattention

What are s/s of an anaphylaxis reaction?

Acute rapidly progressive systemic reaction S/s: urticaria; pruritus; ANGIOEDEMA

A transplant rejection that occurs 1wk-1yr s/p due to T-cell response is called and how is it managed?

Acute. Increase immunosuppressive therapy.

A patient with what disorder can experience s/s of diminished catecholamine release (bradycardia, hypotension, hypogylemia & bradypnea) & excessive urinary output & sodium loss (d/t lack of mineralocorticoids) & elevated potassium (d/t kidneys preserving K) during severe illness or trauma?

Addison's Disease (insufficient adrenal function—>decreased production of both mineral corticoids & glucocorticoids)

How does a posterior hip dislocation present?

Adducted; internal rotation

Narcan should be given to what?

Adequate respiration only (not to wake up or put in withdrawal)

What is the part of the treatment of lithium toxicity?

Administration of large volumes of IVF to enhance renal excretions

A patient suddenly discontinued taking their steroids. what are they at risk of developing?

Adrenal Crisis

Diuresis w/profound DEHYDRATION, HYPERKALEMIA w/potentially life threatening arrhythmias & HYPOGLYCEMIA are indications of what?

Adrenal crisis

What is the ABA recommendation for fluid resuscitation?

Adult: 2ml/kg/TBSA Peds: 3ml/kg/TBSA

Superior Vena Cava Syndrome

Advanced lung cancer tumor OBSTRUCTS blood draining from head, neck, & upper torso S/s: dyspnea; dysphagia; HA; facial swelling Tx: O2; place in SEMI-FOWLERS (facilitates breathing & decrease facial swelling); corticosteroids; diuretics; onc consult to remove tumor

What are Dromotropes?

Affect automaticity (electrical impulse) of the heart at the AV node + increases conduction (epinephrine) - decreases conduction speed (digoxin)

What are inotropes?

Affect contractility (force of contraction) + strengthen contraction (Dopamine) - weaken contraction (beta blockers/CCBs)

Angiotensin-converting enzyme (ACE) inhibitors

Affect the renin-angiotensin-aldosterone system Block conversion of angiotensin I to angiotensin II to reduce BP Adverse effects include: dry NP cough (leads to noncompliance); life threatening angioedema; renal impairment

When does irreversible damage occur in compartment syndrome?

After 4-6 hours OF ISCHEMIA

What is measured by SVR?

Afterload

What is evaluated by SVR?

Afterload (resistance to ventricular emptying)

What are s/o serotonin syndrome?

Agitation, restlessness, confusion, tachycardia, diaphoresis, diarrhea, HA, shivering, piloerection, hyperthermia, seizures, cardiac dysrhythmias, & LOC

What isolation involves a NEGATIVE PRESSURE ROOM; N-95 masks for providers; surgical masks for patients (when outside of private room); & is used for illnesses such as TB & MEASLES?

Airborne

Patient presents with a mandibular fracture. What are they at risk for and whY?

Airway compromise d/t tongue

How is an anaphylaxis reaction managed?

Airway management (POSSIBLE SURGICAL AIRWAY); O2 1st; IVFs; EPINEPHRINE (1MG/ML) 0.2-0.5MG IM (sq uneven absorption); progress to IV epinephrine INFUSION if needed. DIPHENHYDRAMINE + H2 blockers (-tidines); albuterol; methylprednisolone (no immediate effect)

2nd degree HB-type I

Aka Wenckebach/Mobitz 1 Progressive prolongation of PR interval until DROPPED QRS complex

What is preeclampsia?

Aka gestational or pregnancy induced HTN. Disorder leading to decreased oxygenation & perfusion; associated w/coagulopathies, gestational HTN, HA; diplopia; edema, & PROTEINURIA; can be present POSTPARTUM Tx: OB consult; continuous fetal monitoring; support maternal ABCs; minimize stimulation; admit to OB; antihypertensives; MAGNESIUM loading dose & infusion-decreases seizures threshold- (MONITOR RESPIRATORY EFFORT, LOC, BP & patellar reflexes; prepare to reverse w/CALCIUM GLUCONATE 10% IN 10ML OVER 10 MINUTUES)

What should be avoided in a patient who is taking Keppra & why?

Alcohol (lowers the seizure threshold)

What are the worse burns to the eye?

Alkali chemicals (ammonia; lye; lime)

What is an example of a dangerous nasal foreign body?

Alkaline button batteries d/t causing saponification of tissue quickly

What type of chemical causes LIQUEFACTION OR SAPONIFICATION (destroys tissue)?

Alkalis. (Esp lye & ammonia) Which is why large volumes of irrigation is required.

What is something we can do in the ED to decrease denial & anger?

Allowing of family presence

What type of ray can be blocked by clothing or outer layer of skin?

Alpha

What are the S/s of hypertensive emergency/crisis?

Altered LOC; CP; dizziness; epistaxis; HA; seizures; visual disturbances

What is the management for a human bite ("fight bite")?

Amoxicillin/Clavulanate abx; prophylaxis for communicable disease

What lab values will increase with administration of Hetastarch?

Amylase

A degenerative nerve disorder that will result in difficulty w/breathing & swallowing as it worsens is called?

Amyotrophic lateral sclerosis (ALS)

What is Wolfe Parkinson White Syndrome?

An accessory pathway is carrying impulses rapidly through the heart-> tachycardia There will be a presence of a DELTA wave (short PR interval, slurring upstroke of the QRS)

What is hypermagnesemia?

An elevated serum magnesium level. >2.5 mEq/L Seen in renal failure & laxative abuse S/s: RESPIRATORY DEPRESSION; bradycardia; hypotension; decreased DTRs Tx: stop magnesium (if infusing); furosemide; CALCIUM GLUCONATE (10ml or 10% over 10mins); dialysis

EMS calls reporting that patient had a sudden LOC; but now the patient is alert; upon arrival to the ED rapidly deteriorates. Patient is noted to have an ELEVATED SBP w/ wide pulse pressure; Bradycardia; and an altered respiratory pattern. What are they exhibiting?

An epidural hematoma. Check out that Cushing's Triad!!

What are the subtypes of distributive shock?

Anaphylactic Septic Neurogenic

What are the s/s of pediatric anaphylaxis & how is it managed?

Angioedema; facial swelling; difficulty swallowing; respiratory distress; s/o shock IM EPINEPHRINE IN THE VASTUS LATERALIS H1 (Benadryl) & H2 antihistamines (help w/hives) Beta-2 agonists (albuterol) (bronchospasm) Educate on SAFE: seek immediate medical help; allergens; follow up; Epi kits (2) w/them always

How is an arterial clot in the popliteal diagnosed?

Ankle brachial index

What are the types of an incomplete SCI?

Anterior Brown-Sequard Central Posterior

The disruption of the lateral tracts of the spinal cord (which transmit sensory impulses for pain, Temperature & crude touch) are seen in what incomplete cord syndrome? How can deficits be tested?

Anterior Cord Syndrome. Induce pain (poking with sharp needle)

What presents with CRUSHING CP, DYSPNEA, ventricular dysrhythmias, cardiogenic shock (crackles & S3)?

Anterior MI

ST elevation in leads V1-V4 are indicative of what?

Anterior MI (LAD)

Loss of motor function, pain, crude touch & temperature but retains proprioception is seen in what?

Anterior cord SCI

An abducted arm with the inability to bring affected arm to chest or touch the opposite ear are seen with what dislocation?

Anterior shoulder

What are the types of Incomplete Cord Injuries?

Anterior; Brown-Sequard; Central; Posterior

What type of Epistaxis most common?

Anterior; bright red blood (e.g. picking nose)

What biologic agent toxicity causes black eschar w/cutaneous anthrax?

Anthrax. **infected animal hides**

What toxicity causes "Blind as a bat; mat as a hatter; hot as hades; dry as a bone"; HTN; TACHYCARDIA; flushing; DELIRIUM; hallucinations?

Anticholinergic toxicity

"Hot as a hare, Red as a beet, Dry as a bone, Blind as a bat, & Mad as a hatter" are all effects of what type of toxicity?

Anticholinergics

What toxicity causes the patient to not be able to see,spit or shit?

Anticholinergics

What shouldn't be given to a patient w/chronic glaucoma & why?

Anticholinergics (scopolamine) d/t increases IOP

What are examples of ADJUVANT MEDICATIONS for pain?

Antiemetics; antidepressants; corticosteroids.

What is an example of something you would find ethylene glycol in?

Antifreeze.

What are examples of anticholinergics?

Antihistamines; TCAs; Jimson Weed

What is the treatment of HIV/AIDS?

Antiretroviral meds (-virs) to keep VIRUS FROM REPLICATING TREAT OPPORTUNISTIC INFECTIONS Vaccine on the horizon

What is a situational crisis?

Anxiety d/t a life event (sudden illness, injury or DEATH OF FAMILY MEMBER) Grieving varies by cultures, personality & coping skills

What should NOT be given to patients with chicken pox?

Any ASA containing products (d/t association with REYE'S SYNDROME—>liver dysfunction—>INCREASED AMMONIA)

A tear at the ligamentum arteriosum seen w/fractures of the sternum, 1st or 2nd ribs or scapula is known as?

Aortic Dissection

Periumbilical pain that migrates to RLQ; abdominal pain w/rebound tenderness; nausea; anorexia; guarding; low grade fever is seen with?

Appendicitis

An obstruction of the appendix that results in ischemia & bacterial invasion is called? How is it managed?

Appendicitis WBC; U/s (shows enlargement & fluid collection); CT Keep NPO; prophylactic IV broad spectrum ABX; prepare for laparotomy

What should not be done for a venomous PIT VIPER bite?

Application of ice on bite

Patient presents with an eyelid laceration- what is the course of management?

Apply an ice pack (real ice; not chemical) to reduce lid edema & assist in approximating edges Specialty consult if laceration through the lacrimal gland

How is an amputation managed?

Apply tourniquet (or two) to stump to stop the bleed Lightly brush of gross debris; gently irrigate w/NS (avoid distilled water or iodine) Wrap amputated part in saline moistened sterile gauze; place in sealed, labeled plastic bag & place on top of separate bag of ice avoiding direct contact between tissue & ice

What education should be provided for otitis externa?

Apply warm compresses; KEEP EAR DRY; no objects in ear; earplugs in while swimming/bathing.

What education needs to be provided regarding crutches?

Arm piece should be 2 INCHES (2-3 finger widths) below axilla Keep 6 INCHES TO SIDE for steady gait Hand piece should be placed so that elbow is FLEXED AT 30 DEGREES

What should be suspected in a diver who loses consciousness on ascent or loses consciousness w/in 10mins of surfacing?

Arterial Gas embolism

What are the s/s of an epidural hematoma?

Arterial bleed; sudden loss of consciousness or short period of unconsciousness f/b lucid period & subsequent deterioration Dilated nonreactive pupil on IPSILATERAL SIDE

What medication should not be given to a patient in thyroid storm and why?

Aspirin; elevates T4

What is the management of a forearm fracture?

Assess for MEDIAN nerve damage Splint w/elbow flexed 90 degrees; sling to support arm

What additional assessment is needed for Calcaneus fracture & how is it managed?

Assess for associated LUMBAR SPINE injuries Apply compression dressing & crutches

How is a clavicle fracture managed?

Assess for axillary nerve, subclavian and or axial artery damage Ice, sling & swath; figure of 8

What is Neonatal Resuscitation?

Assist w/breathing AT 40-60 BREATHS PER MINUTE if HR <100; 2 thumb encircling chest compressions at 1/2 AP diameter if HR < 60 despite PPV AT 3:1; GOAL IS TO GET HR >100 PREDUCTAL (RIGHT WRIST) O2 SAT AT 1 MINUTE IS 60-65%, 90% IN 10MINS Meds: epinephrine & glucose per AHA (last resort) Narcan not indicated IVF bolus for NEWBORN IS 10ML/KG

Right Ventricular MI

Assoc w/inferior MI from PROXIMAL occlusion of RCA PRELOAD dependent; use caution w/preload reducing agents (NTG & MSO4) Require volume administration

Dyspnea; increased WOB; tachypnea; accessory muscle use; EXPIRATORY WHEEZING; prolonged expiratory phase; Low o2 saturation are also s/s of?

Asthma

Peak Expiratory Flow Rate (PEFR) is used to diagnose what?

Asthma

What is a chronic disorder of the lower airways that manifests as hyperactivity; bronchospasm; mucus production? How is it managed?

Asthma. Dx w/peak flow; CXR to r/o PNA Tx w/SABAs such as albuterol; anticholinergics (ipratropium); PO steroids; fluids; IV magnesium; terbutaline; Heliox; ketamine

What are the s/s of genital herpes & how is it treated?

Asymptomatic PAINFUL GROUPED VESICLES on an erythematous base on genitalia/btx; fever; malaise; HA; myalgias; lymphadenopathy; dysuria. Antiviral therapy (the VIRs) to REDUCE SEVERITY OF SYMPTOMS & RECURRENCE (in other words not a cure) Avoid sexual activity until lesions are dry.

When do you obtain an APGAR score?

At 1 minute & 5 minutes

What are the s/s of CO poisoning & how is it managed?

At 10-20%: HA, n/v At 20-40%: dyspnea; confusion & lethargy At 40-60%: ST SEGMENT DEPRESSION (d/t HYPOXIA); arrhythmias; seizures; coma; arrest; "cherry red" skin at >60%: death Tx w/HIGH FLOW O2 via tight fitting mask until level <10% or AROUND 60-90 MINUTES. Consider HYPERBARIC OXYGENATION (HBO) for pregnant patients (fetus most vulnerable)

What is a viable fundal height?

At umbilicus (20wks)

What shouldn't be used w/a transplanted heart? What should be used instead?

Atropine TX WITH ISOPROTERONOL instead

How is symptomatic bradycardia managed?

Atropine 0.5mg Transcutaneous pacing w/rate set at 60-80; turn milliamps until capture (do not assess carotid pulse) Dopamine or Epinephrine infusion

What is the antidote for organophosphates?

Atropine/2 PAM

What is myasthenia gravis?

Autoimmune disease Affects mainly women in 20-30s Affects acetylcholine binding sites leading to muscle fatigue; PTOSIS (drooping eyelids); dysphagia; & respiratory paralysis Dx w/ TENSILON Tx: Neostigmine

Coolness below the level of injury w/flushing above the level of injury is associated w?

Autonomic Dysreflexia

The impaired driver makes requests regarding the care given is an example of what?

Autonomy

The right to make's one own choices & have those choices respected is what?

Autonomy. E.g. DNR

What discharge instructions should be provided for spontaneous AB?

Avoid douching, tampons, & intercourse for at least 2 weeks; monitor body temperature BID x 5days; seek medical attention if fever >37.7 C (100F); bleeding > than heavy period or clots > a quarter.

What discharge teaching should be given to patient w/GERD?

Avoid fatty foods, chocolate, caffeine, alcohol, CCBs, & Valium. Eat smaller meals. Sit up after eating.

What discharge instruction must be provided for a patient w/Herpes?

Avoid intercourse during outbreaks; have protected sex always; C-section may be scheduled for pregnancy/delivery to avoid transmission MUST NOTIFY FUTURE PARTNERS

What education will you provide your patient re:diverticulitis?

Avoid straining; low fat/low fiber diet; LOW RESIDUE DIET DURING ACUTE PHASE; then increase fiber in diet. Stool softeners and INCREASE WATER intake Instruct the elderly to increase the natural fiber their diets.

An avulsion where skin is pulled away from the scalp, hands, digits, feet, & toes is known as?

Avulsion

Peeling of skin from underlying tissue is called...?

Avulsion

The peeling of skin from the underlying tissue is known as?

Avulsion

What are the post-procedure criteria?

Awake; able to ambulated; able to swallow; VS WNL/baseline.

What is the treatment for Chlamydia?

Azithromycin 1gm PO once or Doxycycline 100mg BID x7 days

Elevation of BUN & creatinine (not associated w/renal failure) is known as what?

Azotemia

What hepatitis is transmitted by contact w/blood or body fluids & is often seen in IV DRUG USERS & those who engage in unprotected sex?

B

What Hepatits is BODY FLUIDS BASED?

B- sexually transmitted; body fluids; human bites (vaccine available)

Pericardial tamponade

BECK'S TRIAD (JVD, hypotension, & MUFFLED heart sounds); Kussmaul sign (increase in JVD on inspiration); PULSUS PARADOXUS (decrease in BP on INSPIRATION); ELECTRICAL ALTERNANS (alternating amplitude of QRS) Risk of PEA Perform pericardiocentesis Left Xyphoid process. Better if BP increasing

When should the peak flow be performed on a patient w/asthma?

BEFORE & AFTER EACH NEBULIZER

An oath to help others (in the example of an unresponsive patient) is what?

BENEFICENCE

What are lab changes seen w/DKA?

BGL 250-600; elevated BUN; KETONES in urine; PH < 7.35 (METABOLIC ACIDOSIS); monitor POTASSIUM carefully

What is hypoglycemia in an infant?

BGL < 40

What is hypoglycemia in an adult?

BGL <60

Hypoglycemia

BGL <60-70. **Decreased LOC- check POC (bgl)** Early s/s: shakiness; hunger; cool, diaphoresis skin; tachycardia; pallor. ** BETA BLOCKERS MASK these SYMPATHETIC response** Tx: PO intake 10-15g simple carbs (conscious pts). IV dextrose (unresponsive) IM glucagon INDUCES vomiting (higher r/o aspiration)

What dx findings would be seen in DKA?

BGL > 200; pH <7.3 (metabolic acidosis); HCO3 <15 (wide anion gap); ketonemia; ketonuria

What is Hyperosmolar Hyperglycermic Syndrome?

BGL > 600; significant volume depletion (HYPOVOLEMIC SHOCK); AMS changes (lethargy; COMA); seizures; shallow breathing (but no fruity smell) Develops SLOWLY.

What labs changes are seen HHS/HHNK?

BGL >600; EXTREMELY ELEVATED BUN; glucose in urine but NO KETONES; NORMAL pH

Descending symmetrical paralysis is seen in what?

BOTULISM

A depression of the knuckles & fracture of the 4th or 5th METACARPAL is known as?

BOXERS (amateur boxer)

What d/c education should be provided for Epistaxis?

BP management (posterior) Avoid blowing/picking nose; cool mist humidifier (anterior bleed)

What is a side effect of taking PHENAZOPYRIDINE for pain r/t UTI pain?

BRIGHT ORANGE urine

How is a toe fracture managed?

BUDDY TAPE toes; wear hard-soled, open toed shoes

What sign's do Infants present with w/meningitis?

BULGING FONTANELLES; OPISTHOTONOS (BACKWARD ARCH) & HIGH PITCHED CRY

How is prerenal ARF dx'd & tx'd?

BUN: Creat ratio- 20-40:1 (e.g. BUN 90: creat 2.8) Low urine sodium (<20); HIGH SPECIFIC GRAVITY (>1.020); high urine osmolality Tx: IV CRYSTALLOIDS (increase circulation—>increasing u/o)

What are the lab changes & management for intrarenal ARF?

BUN:Creat ratio 10-15:1 (e.g. BUN 50: creat 11) High urine sodium (>20); low specific gravity (<1.010); & low urine osmolality Tx: cessation of nephrotoxic drugs; DIALYSIS

What are the s/s of magnesium toxicity? How is it managed?

BURP {B}P decreases {U}rine output decreases {R}espiratory rate decreases (<12 bpm) {P}atellar reflex absent (absent DTR) S/o fetal distress Stop magnesium. Administer CALCIUM GLUCONATE IV

Excessive fluids in or infection of the bursa d/t OVERUSE, REPETITIVE MOVEMENTS, inflammatory disease, infection, & trauma is called...?

BURSITIS

What causes a fishy smelly vaginal discharge?

BV & Trichomonas (tx w/Flagyl)

What is Ludwig's Angina?

Bacterial infection submandibular s/p tooth abscess S/s: difficulty swallowing; DROOLING; swelling & redness of neck; TONGUE SWELLING Tx: maintain airway; ABX

What is epiglottitis?

Bacterial infection; incidence has decreased since HIB vaccine S/s: triad of Ds- Drooling, Dysphagia, & Distress; "hot potato" voice;abrupt onset of high fever Dx: thumbprint sign on lateral neck Xray; "turtle" sign (extend neck) to open airway Tx: keep patient CALM (no IV or labs); consult expert to secure airway STAT

The CSF from the LP completed come back with a low glucose...what could cause this?

Bacterial meningitis

You just assisted with a LP. The CSF results come back as high pressure; cloudy & w/low glucose. What does it sound like?

Bacterial.

Patient presents w/painful cystic mass in near vagina, what it is and how is it treated?

Bartholin's cyst; WARM COMPRESS; I/D

How is TBSA determined for fluid resuscitation in burns?

Based on partial & full thickness; NOT SUPERFICIAL

Patient presents w/otorrhea (CSF from ear); mastoid ecchymosis (Battle's sign) & hemotympanum (blood behind eardrum) following trauma-what is going on?

Basilar skull fracture involving middle fossa

Patient presents w/rhinorrhea (CSF leakage from nose); periorbital ecchymosis; & anosmia (can't smell) following trauma- what is going on?

Basilar skull fracture involving the anterior fossa

Battle's sign is associated with what ? How long does it take to manifest?

Basilar skull fracture; 12-24hrs to manifest

Hallucinations, paranoia, excited delirium, tachycardia, HTN are s/e of what type of ingestion?

Bath salts; synthetic cannabinoids

What does it mean to act w/justice?

Be fair & impartial; TREAT ALL EQUALLY

What discharge education should be provided a patient with a threatened abortion?

Bed rest for 1-2 days (except for bathroom); nothing in vagina

What education needs to be provided for a patient w/meniere's?

Bed rest; SLOW POSITION CHANGES TO AVOID FALLS; limit activity & sodium/sugar intake; avoid caffeine, smoking & alcohol

How do you treat renal trauma?

Bed rest; increase fluid intake if stable; repair lacerations; MONITOR U/O CLOSELY

Patient presents with unilateral facial never (VII) paralysis & unable to CLOSE AFFECTED EYE. What is the potential cause & how is it managed?

Bell's Palsy. Tx w/steroids, protect eye from injury; use artificial tears; do facial massage

What is a complication of Lyme disease??

Bell's Palsy....Facial Palsy's!!

What is a complication of Lyme Disease?

Bells Palsy

What presents w/unilateral facial paralysis d/t CN VII (FACIAL) inflammation?

Bells Palsy

How are DYSTONIC REACTIONS managed?

Benadryl initially Then Artane or Cogentin

Anticholinergic toxicity can be caused from what?

Benadryl, meclizine, scopolamine, Imodium, & atrovent

What is the duty to help others?

Beneficence. E.g. CARE FOR UNRESPONSIVE PATIENT

How are unpleasant illusions that can be experienced with Ketamine be relieved?

Benzodiazepines

What is the mainstay treatment of seizures?

Benzodiazepines

What is the Glasgow Coma Scale?

Best eye opening, verbal response, & motor response; 3-15. 13-15 minor head injury 9-12 moderate <8 severe : AIRWAY

What ray can be blocked by glass, wood or plastic?

Beta

what masks signs of hypoglycemia?

Beta blockers

Bradycardia, HYPOGLYCEMIA, & hypotension are commonly seen with what toxicity? How is it managed?

Beta blockers GLUCAGON 3-10 MG IV (competes w/receptor sites & neutralizes effects; enhances contractility) transcutaneous pacing Dextrose

What should be avoided if CP is d/t stimulant use?

Beta blockers.

How do you evaluate responsiveness to treatment w/compartment syndrome?

Better when pain is decreased & pressure < 20

Higher pressure on inhalation & lower pressure on exhalation is seen with what type of NIPPV?

BiPap

What is bulimia?

Binging and purging Laxative use (hypokalemia) Diuretic use (hyponatremia) Diet pills (cardiac arrhythmias & HTN) Induced vomiting (erosion of tooth enamel; callous on index finger; MALLORY-WEISS TEARS)

What is schizophrenia?

Bizarre behavior including inability to care for self or manage activities of daily living TYPICAL ONSET IN EARLY 20'S S/s: DELUSIONS, HALLUCINATIONS, DISORGANIZED SPEECH Tx: orient to reality FOR SAFETY; use short, concrete sentences (avoid figures of speech); administer antipsychotics; WATCH FOR EXTRAPYRAMIDAL SYMPTOMS (DYSTONIC REACTIONS)

What complimentary alternative therapy is used for "hot flashes" & can cause liver failure especially when mixed w/alcohol, APAP, or Lipitor?

Black Cohosh.

What spider is characterized by a red hourglass on the abdomen of the female?

Black Widow

Patient presents with c/o feeling an immediate sting that progressed to a dull ache w/in 20 mins then c/o ABDOMINAL CRAMPING, MUSCLE SPASMS; HTN, tachycardia, N/V & weakness. What is the cause & how is it managed?

Black widow spider Tx w/ice; elevation; ANALGESICS & BENZODIAZEPINES (to control ms spasms) Antivenin cautiously (only d/t serum sickness)

What injury is most associated w/PELVIC FRACTURES?

Bladder; the fuller the bladder is the greater the chance of injury

Disruption of tissue, evisceration, & traumatic amputation occurs d/t massive air movement & high velocity projectiles (e.g. bombs; GSW) is called?

Blast injury

What are esophageal varices?

Bleeding from dilated blood vessels SECONDARY TO LIVER DISEASE (from portal HTN) Treatment same as upper GI bleed. Questionable use of balloon tamponade (Sengstaken-Blakemore or Minnesota tube)

What are the s/s of hemophilia?

Bleeding of soft tissues, muscles or joints (HEMATHROSIS)

Ingesting methanol (e.g. windshield wiper fluid) can lead to what?

Blindness

What are the s/s of a urethral injury & how is it managed?

Blood at the urinary meatus; high-riding prostate; perineal ecchymosis; gross hematuria; contrast extravasation w/a retrograde urethrogram URINARY CATHETER IS CONTRAINDICATED Suprapubic catheter may be required.

What is a PE?

Blood clot from a DVT; air emboli from diving; amniotic fluid emboli; FAT EMBOLI (12-48hrs s/p LONG BONE FX- watch for chest/axilla PETECHIAE) S/S: SUDDEN ONSET OF SOB; TACHYPNEA; TACHYCARDIA; cough w/HEMOPTYSIS; SYNCOPE; fever; accentuated S2 heart sounds Dx: elevated D-dimer; peaked P waves & depressed T waves on EKG; V/Q scan; PULMONARY ANGIOGRAPHY Tx: O2; anticoagulants/fibrinolytics; embolectomy

What is a DVT?

Blood clot in the peripheral vein in pelvis, thigh, or LE d/t sluggish blood flow Risk factors: VIRCHOW'S TRIAD (venous stasis); vein damage; hypercoagulability S/s: achy; throbbing pain (more common w/walking) Risk of PE Tx: elevate elevated extremity; COMPRESSION; anticoagulants

What is a hyphema?

Blood in the anterior chamber; often d/t a direct blow

Causes of normolytic anemia

Blood loss; renal disease; malignancy; sickle cell disease

What happens in sequestration crisis? How is it managed?

Blood pools in spleen & other visceral organs—>severe hypovolemic shock Restore blood volume

What is involved in damage control resuscitation?

Blood products in a 1:1:1 ratio of PRBCs, plasma, & platelets Allow permissive hypotension to limit amt of blood exanguinated Administer TXA 1 g IV bolus to prevent clot breakdown

What causes preload to decrease?

Blood/fluid loss or in VASODILATION. Tx: volume FIRST. Inotropes (Dopamine). Vasopressors (epinephrine)

What are common causes of Red or Orange urine?

Blood; warfarin blood thinner; phenazopyridine (azo; pyridium)-used to tx pain in UTI; Isoniazid & Rifampin (TB tx); ibuprofen or sulfasalazine; metronidazole (dark orange to brown); laxatives w/Senna; metals (e.g. lead & mercury); foods like beets, blackberries, rhubarb, carrot juice

What are the s/s of emergent delivery?

Bloody show; RUPTURE OF MEMBRANES; frequent contractions; desire to bear down; "baby is coming"; crowning of head

What happens with a blow in orbital wall fracture?

Bone fracture into orbit toward eye; EXOPHTHALMOS (EYE BULGING) E.g. baseball hitting lateral side of face

What happens with a blow out orbital wall fracture?

Bone fractures away from eye; ENOPHTALAMOS (eye sunk into socket) Occurs w/direct blow to socket

How does Leukemia cause neutropenia?

Bone marrow manufactures leukemic (abnormal) immature WBCs that aren't functioning properly (they broke) & NOT PROVIDING ADEQUATE PROTECTION FROM INFECTION

Osteomyelitis is commonly seen where?

Bottom of foot; "fight bites"

What biologic agent toxicity causes ptosis, flaccid paralysis, & blurred vision?

Botulism

What causes PTOSIS & SYMMETRICAL DESCENDING PARALYSIS? And commonly seen in scenarios such as local honey ingestion by an infant or ingestion of food left out at picnic?

Botulism.

Fracture of the 5th metacarpal "pinky" finger from striking an object w/a closed fist is known as?

Boxer's fracture

Patient presents with pain, tenderness & swelling around knuckle after a fight. What is the possible diagnosis & how is it managed?

Boxer's fracture Posterior ulna splint/gutter splint; ulner nerve block for pain control. Assess for "fight bite"

What are possible complications of an anterior shoulder dislocation?

Brachial plexus & axillary artery compromise

What is a OMINOUS sign in a pediatric patient?

Bradycardia

What are the common characteristics of neurogenic shock?

Bradycardia & hypotension

What are s/s of neurogenic shock & how is it managed?

Bradycardia; hypotension; venous pooling in periphery; warm skin; temperature instability Vasoactive medications to increase HR & promote blood flow

What is the Monro-Kellie doctrine?

Brain tissue, blood volume, and CSF are balanced in a state of dynamic equilibrium--if an increase in one occurs, the volume of one of the other components must decrease or ICP will increase. Brain 80%, CSF 10%, Blood 10%

What happens during carbon monoxide poisoning?

Breathe in CO—>the CO attaches to & replaces oxygen on the hemoglobin molecule—>carboxyhemoglobin & reduces O2 content of blood

What is a Ventricular Assist Device?

Bridge while awaiting heart transplant Patient will have an AUDIBLE HUM; barely palpable pulses; MAP 60-90 w/DOPPLER over BRACHIAL/RADIAL ARTERY Contact rep & defibrillate (AP pads) as needed but CPR may damage device

"Shark fin" appearance on the end tidal capnography waveform is seen in what?

Bronchospasm

What are the common characteristics of Anaphylactic shock?

Bronchospasm & vasodilated vessels

What spider is characterized by a "violin-shape"?

Brown Recluse (FIDDLE BACK)

What are the s/s of a spontaneous abortion?

Brown/bloody discharge to profuse vaginal bleeding w/passage of tissue or POC

What are examples of pediatric abuse regarding bruises?

Bruises in various stages of healing; shapes of identifiable objects (pattern injuries); human bite marks

What supplies are needed for an imminent delivery?

Bulb syringe; scissors; cord clamps; towels

The inflammation of the JOINTS that perform repetitive motion (shoulders; knees; elbows; hip) that causes pain, swelling & stiffness of the joint is known as?

Bursitis

What is a gram + anaerobic bacillus ABX ASSOCIATED profuse frequent diarrhea accompanied w/abdominal cramping/pain; fever; loss appetite; dehydration?

C-Diff

What hepatitis is CIRCULATION BASED (I.E. BLOOD ONLY)?

C. BLOOD EXPOSURE. No vaccine

While a ROTOR-WING (helicopter) allows for rapid point to point transfer it's downsides are? What are some considerations for using this type of transport?

CABIN NOT PRESSURIZED & small (air expands; vibration) DECOMPRESS STOMACH w/gastric tube prior to transport to decrease risk of aspiration. Air splints will expand (so not appropriate if transporting this way) MAY PLACE CHEST TUBES FOR SMALLER PNEUMOTHORAX.

What is Trousseau's sign?

CARPOPEDAL SPASM

How do you dx appendicitis?

CBC to leukocytosis (>10K w/ >10% BANDS); initial CT or u/s; freq reassessment with ULTRASOUND

How is sepsis/septic shock dx'd & managed?

CBC; LACTATE; abg; procalcitonin; bmp; cultures (prior to ABX); CXR Early fluid resuscitation at 30ml/kg ISOTONIC FLUIDS; ABX; VASOPRESSORS; mechanical ventilation; glucose control <150mg/dL

What do you see w/Central Herniation?

CHANGE IN LOC; restlessness that progresses to coma (indicating reticular activating system impairment) Abnormal breathing rate/pattern (Cheyne-Stokes) Dilated/Nonreactive pupils Bradycardia

What do you not give for an iron OD?

CHARCOAL (iron doesn't bind to)

How is heavy metal poisoning managed?

CHELATING THERAPIES: BAL in oil, DMSA or EDTA which bind to metals & facilitate excretion

What is the hallmark sign of Prinzmetal's angina?

CHEST PAIN AT REST

Patient presents with severe dyspnea, tachypnea, tachycardia, weak thready pulses & moist cool skin. What is going on? How is it managed?

CHF High flow O2; NTG; IV morphine; diuretic therapy; DOBUTAMINE (to increase contractility); digoxin (increase contractility & decrease HR)

What are the s/s of TB?

CHRONIC COUGH; NIGHT SWEATS; FEVER; CHILLS; weight loss; anorexia; fatigue

What are the s/s of hypocalcemia?

CHVOSTEK'S SIGN; TROUSSEAU'S SIGN; tetany; confusion; PROLONGED QTI

Patient presents with an eyelid laceration. What also needs to be assessed?

CN II & III

Agitation, aggressiveness, MYDRIASIS, HTN, tachycardia, hyperthermia, dysrhythmias, & paranoia are seen in what toxicological emergency?

CNS stimulants

What are the 3 C's of TCA OD?

COMA (altered LOC); CARDIAC DYSRHYTHMIAS (VT); CONVULSIONS (seizures)

What are s/s of PCP ingestion & how is it managed?

COMBATIVE BEHAVIOR; INCREASED PHYSICAL STRENGTH; LACK OF PAIN SENSATION Reduce stimulation; protect patient from harming themselves or others. Benzodiazepines for agitation; antihypertensives for HTN

What do you need to monitor for with a tibia/fibula fracture?

COMPARTMENT SYNDROME

What are potential complications of a tibial or fibular fracture?

COMPARTMENT SYNDROME Volkmann's contracture

Giardia is found in what?

CONTAMINATED WATER

A progressive & irreversible disease characterized by diminished lung capacity that presents w/hacking cough & tachypnea is known as?

COPD

Compensated Respiratory acidosis is commonly seen w/what?

COPD

How is a human bite managed?

COPIOUS IRRIGATION & debridement; LEFT OPEN & BULKY DRESSING APPLIED TO DECREASE MOVEMENT PROPHYLACTIC ABX

What is the treatment for Ménière's disease?

CORTICOSTEROIDS; MECLIZINE (MOTION SICKNESS); antihistamines; diuretics; anticholinergics

unstable angina

CP UNRELIEVED by rest or NTG

What is STABLE angina pectoris?

CP that occurs w/physical exertion, short in duration; relieved by rest or NTG - TROPONIN

Stable angina

CP w/EXERTION; relieved by rest/NTG.

What is UNSTABLE angina?

CP w/little physical exertion; longer; unrelieved by rest/NTG - TROPONIN

acute chest syndrome

CP; dyspnea; cough; hypoxemia Most common cause of DEATH from SCD

what are the s/s of a cardiac tamponade? How is it managed?

CP; tachycardia; PEA; dyspnea; electrical alternans; kussmaul's sign; PULSUS PARADOXUS; BECK'S TRIAD (hypotension, distended neck veins, muffled heart sounds) Pericardiocentesis at left xyphoid; needle aimed toward left shoulder

Continuous airflow at a single set pressure to keep airway open is what type of NIPPV?

CPAP

Cardiac Arrest-Shockable

CPR 100-120compressions/min. Defibrillator- shock at 120-200j BIPHASIC. Restart CPR at 2min cycles. IV/IO for epinephrine 1mg every 3-5mins. Advanced airway w/waveform capnography. Amiodarone 300mg or Lidocaine 1-1.5mg/kg. Treat reversible causes- Hs & Ts- Point Of Care UltraSound (pocus) fluids, sodium bicarbonate 50mEq Get ETCO2 >10 w/compressions.

Patient presents with c/o cramping in RLQ or periumbilical; diarrhea; SMALL INTESTINE scarring & anal fissures. What is going on & how will you treat?

CROHN'S DISEASE. Fluid resuscitation, antidiarrheal, anti-inflammatories, corticosteroids, immunosuppressants, surgery. Anticholinergics for diarrhea.

What are s/s of conjunctivitis & how is it managed?

CRUSTY EYELIDS; sensation of foreign body; CONJUNCTIVAL ERYTHEMA; discharge (bacterial = purulent; allergic/viral = serous); pruritis w/allergic reaction. ABX (systemic if d/t gonococcal); antivirals; COMPRESSESS & decongestants for allergic rx. Educate on AVOIDING CONTACT LENSES & EYE MAKEUP; using compresses; avoiding spread (no pools/hot tubs; don't share linens; wash yo damn hands

WIDENED PULSE PRESSURE, profound bradycardia; & altered respiratory pattern (Cheyne-Stokes) is know as what? When is it seen?

CUSHING'S TRIAD. Late sign of increased ICP

How is TB dx'd & managed?

CXR; SPUTUM CULTURE FOR ACID-FAST BACILLI Standard & airborne precautions (NEGATIVE ROOM STAT) 6 MONTHS of combination ABX (RIFAMPIN & ISONIAZID)

Your patient reports burning PLASTICS or CARPETS, what are they at risk for?

CYANIDE POISONING

What type of fracture has the potential for concurrent LUMBOSACRAL COMPRESSION FRACTURE?

Calcaneus

A patient w/status Epilepticus is most likely to have an elevation in what lab value?

Calcium

How is hypocalcemia treated?

Calcium gluconate

What is the antidote for a CCB toxicity?

Calcium.

What is the antidote for calcium channel blockers?

Calcium; glucagon; insulin

How is hypertensive crisis managed?

Calculate MAP; DECREASE MAP BY 20-25% OVER 1-2HRS w/nipride (preload & afterload reducer; protect from light)

How is postpartum hemorrhage managed?

Call OB for help. Apply O2 & IV access. Activate massive transfusion protocol w/PRBCs, platelets, & plasma. Consider TXA. Palpate uterus to assess for atony; perform UTERINE FUNDAL MASSAGE. Small doses of ketamine or fentanyl for pain. Start meds to address uterine tone (OXYTOCIN) at 10U IM or 40U in 1L of NS-run open until uterus firm then at 200ml/hr. Consider misoprostol, methylergonovine, &/or Carboprost Examine vaginal tract for trauma/retained products.

Why do you want to avoid rubbing areas of frostbite??

Can take off layers of skin

What causes cottage cheese like vaginal discharge?

Candida (tx w/Nystatin or Diflucan)

What should be monitored for a patient who presents with an inhalation injury?

Carbon monoxide poisoning; get COHb level; administer 100% high flow O2

A patient with anorexia is at risk for?

Cardiac dysrhythmias d/t electrolyte abnormalities

How is TCA OD managed?

Cardiac monitoring; intubation; SODIUM BICARBONATE for urine alkalization); lidocaine & MAGNESIUM SULFATE (if polymorphic VT develops)

What shock is d/t the pump being pooped? How is it treated?

Cardiogenic (pump failure); tx by optimizing the hemodynamics

LV dysfunction from an anterior MI can cause what?

Cardiogenic pulmonary edema/CHF

Trousseau's Sign

Carpopedal spasm when inflating BP cuff associated with hypocalcemia

What is a study that generally highlights an UNUSUAL or INTERESTING CASE a practitioner has encountered?

Case study

How is a colles fracture managed & what is a potential complication?

Casting or surgical reduction Median nerve injury

What type of bite has a high risk of infection (usually PASTEURELLA)?

Cat bite

What animal bite has the highest rate of infection?

Cat d/t long fangs that penetrate into tissue; saliva that contains PASTEURELLA (can cause cellulitis or osteomyelitis) Leave wound OPEN UNLESS ON FACE. PROPHYLACTIC ABX

What are long term complications of a lightening injury?

Cataracts Hearing loss

What stage of pertussis presents w/coryza; sneezing; low grade fever?

Catarrhal

What are the risks of suicide?

Caucasian; family history of attempts; BH hx; substance abuse; h/o abuse; prior attempts; CHRONIC PHYSICAL ILLNESS

What is a fractured larynx?

Causes: direct blow to neck (Hi ya! Karate kick!); ATV driver into cable etc S/s: dyspnea; DSYPHONIA (hoarseness); SQ EMPHYSEMA; Hamman's crunch synchronized w/HB Tx: fiberoptic bronchoscopy for intubation & prepare for sx

What is the treatment for Gonorrhea?

Ceftriaxone IM once; Azithromycin PO once or Doxycycline x 7 days

Disproportionately greater weakness in upper extremities as compared to the lower extremities is likely to occur in what spinal cord injuries?

Central

Loss of perfusion to the retina & if circulation not RESTORED W/IN 60-90MINS—>blindness d/t emboli (AFIB INCREASES RISK), thrombosis, HTN, temporal arteritis is called?

Central Retinal artery occlusion

An embolus that originates in the carotid artery/cardiac valves blocks the ophthalmic artery causing what?

Central retinal artery occlusion

With an inevitable abortion what is going on?

Cervical OS is open

What is Chandelier sign?

Cervical motion tenderness, indicative of PID, ruptured tubal pregnancy and endometrious

What side effect does Rifampin have?

Changes blood fluid to ORANGE.

What are CONTINGUOUS lead changes?

Changes in 2 or more leads Inferior-II,III, aVF Anterior-V1-V4 Lateral- I, aVL, V5, V6

Chvostek's sign

Cheek & lip (facial) spasm when Cheek is tapped associates with hypocalcemia

If a neonates HR <60 what do you do?

Chest compressions at 3:1 breath

What are the s/s of aortic dissection & how is it managed?

Chest or back pain; unequal pulse strength or BP; tachycardia; hypotension; paraplegia; WIDENED MEDIASTINUM on CXR Massive transfusion protocol & emergent sx

What is an adverse effect in Fentanyl?

Chest wall rigidity

What virus becomes LATENT after primary infection?

Chicken Pox (—>shingles)

What STI is ASYMPTOMATIC 75% of the time??

Chlamydia

What is the common cause of epididymitis?

Chlamydia

What causes mucopurulent vaginal discharge?

Chlamydia (tx w/Doxycycline)

A transplant rejection that occurs gradually over months to years is called and how is it managed?

Chronic Need re-transplantation

What occurs during the 3rd stage of Lyme's Disease?

Chronic arthritis

Recurrent episodes of Inflammation of the upper airways—>"wet cough" & thick mucus production; normal RR; polycythemia; COR PULMONALE is what type of COPD?

Chronic bronchitis

What is Parkinson's disease?

Chronic degenerative disease affecting the dopamine pathway S/s: tremor at rest; facial "mask"; "COGWHEEL" RIGIDITY; bradykinesia Tx: carbidopa (levodopa)

What is asthma?

Chronic reactive airway disease characterized by airway hyperactivity, inflammation, & bronchial constriction Triggered by environmental irritants S/s: chest tightness; dyspnea; tachypnea; cough; PROLONGED EXPIRATORY TIME; RESPIRATORY ALKALOSIS/ACIDOSIS; WHEEZING EARLY; SILENT CHEST; pulsus paradoxus (OMNIOUS sign) Dx: Peak expiratory flow rate (PEFR) by exhaling fully 3x & noting best reading (70-90% of personal best-use inhalers; <70% seek medical attention; <40% severe) Tx: short acting beta-agonists (SABA-albuterol) to relax smooth muscles (s/e is tachycardia); anticholinergics (Atrovert) to limit secretions of mucus & inhibit bronchial muscle contraction; corticosteroids to reduce inflammation; magnesium to inhibit bronchial muscle contraction. Education: use SPACER w/inhalers or NEBULIZERS to increase medication delivery.

What is bipolar disorder?

Chronic recurring cycles of depression & elation (mania) S/s: major DEPRESSIVE EPISODE F/B INAPPROPRIATE ELATION; increased energy (HIGHEST R/O HARMING THEMSELVES & OTHERS); insomnia; pressured speech; grandiose notions; poor judgement; racing thoughts; impulsivity; risk taking behavior; promiscuity Tx: provide SAFETY; MINIMIZE EXTERNAL STIMULI Medications: LITHIUM (CONTROLS SEVERE MOOD SWINGS); Carbamazepine

What treatment is provided for Anathrax?

Cipro or doxycycline

Adolescent patient presents to ED following football practice c/o being unable to raise arm; deformity and swelling noted. What type of fracture occurred & how is managed?

Clavicle Sling & swath or FIGURE EIGHT SPLINT

With a threatened AB, what should the os be?

Closed

What type of HA is characterized by unilateral pain in/behind one eye?

Cluster

Myasthenia Gravis, Gillian Barre, & ALS do not affect what?

Cognitive function

What is it called when subjects w/certain conditions or who receive a certain treatment are FOLLOWED OVER TIME & COMPARED WITH ANOTHER GROUP who are not affected by that condition or treatment? E.g. PTSD IN TBI PATIENTS

Cohort studies

What fractures commonly occur while falling onto an outstretched hand?

Colle's or Smith

What is a subdural hematoma?

Collection of blood between the dura & subarachnoid layer d/t tear of the BRIDGING VEINS (VENOUS-SLOWER PROGRESSION) seen mostly in elderly (anticoagulants) & in chronic alcoholics

What is a epidural hematoma?

Collection of blood between the skull & dura because of a TEMPORAL bone impact & laceration of the MIDDLE MENINGEAL ARTERY (RAPID progression) S/s: brief LOC f/b LUCID period (AWAKE) f/b 2nd LOC; IPSILATERAL PUPIL DILATION w/CONTRALATERAL weakness/paralysis

What is a peritonsillar abscess?

Collection of pus located between the tonsil and extends beyond into the neck & chest tonsils S/s: severe throat pain; DEVIATED UVULA; fever; halitosis; pain that radiates to ear; erythematic tonsils Tx: throat culture; IVF; analgesics; ABX; STEROIDS; I/D

A distal radius fracture (bent backwards) w/possible ulna injury usually d/t a fall on an outstretched hand is known as?

Colles fracture

What is Carbon Monoxide poisoning?

Combustible fumes from malfunctioning furnace, exhaust, etc Shifts the oxyhemoglobin dissociation curve to the LEFT impairing the ability of hemoglobin to release O2 so SPO2 is 100% but DON'T TRUST IT!!- arterial PaO2 is more reliable. S/s: HOARSE VOICE (dysphonia); facial burns; carbonaceous sputum; stridor; wheezing Dx: CARBOXYHEMOGLOBIN (COHb) level & airway evaluation Tx: Airway management as needed; 100% O2 until CARBOXYHEMOGLOBIN <10%; HBO for pregnant females since fetus the most vulnerable

What are the s/s of a Le Fort III?

Commonly UNRESPONSIVE; malocclusion; IMMENSE SWELLING ("beach ball"); severe hemorrhage

What is a posterior type of Epistaxis?

Commonly d/t HTN & COAGULOPATHIES; More serious form. Heavier bleeding; darker red; drips out of nares & down throat; leads to clots MONITOR AIRWAY.

Excessive pressure develops within body cavity enclosed by fascia; as pressure increases, circulation decreases & edema increases—> ischemia & necrosis is called...?

Compartment syndrome

What are the risks of a closed fracture?

Compartment syndrome

What is thyroid storm?

Complication of hyperthyroidism. Increased temp, HR, RR, & BP; pulmonary edema; flushing, agitation,& tremors Tx: decrease HR w/beta blockers; PTU to inhibit hormone synthesis; iodine to slow hormone synthesis (given 1 hour later); APAP

What is myxedema coma?

Complication of hypothyroidism Decreased METABOLISM, T, HR, RR, & BP Tx: rewarm; ventilate, rehydrate, & thyroid hormone replacement (levothyroxine; thyroxine)

What EKG changes will you see w/Pericarditis?

Concave global (widespread, diffuse) ST segment elevation in MOST leads w/o RECIPROCAL CHANGES Tall, peaked T waves in ALL leads EXCEPT aVR

What is sickle cell disease?

Congenital (gene from both parents) hemolytic anemia (SS) causing "sickling" of RBCs.

Inflammation of membrane that lines the eyelid & sclera d/t bacterial/viral/fungal infection, allergic reaction, or chemical irritation is called??

Conjunctivitis

Measles (Rubeola)

Conjunctivitis, coryza (inflammation of mm in nose), cough & Koplik spots Standard & airborne precautions

What do you see w/2nd degree Type II HB?

Consistent PR interval w/blocked P waves->inconsistent R-R interval (PR consistent, RR inconsistent)

What are the PARASYMPATHETIC responses?

Constricts pupils; stimulates salivation & tears; CONSTRICTS airways; SLOWS HR; stimulates digestion; stimulates pancreas to release insulin & digestive enzymes; DILATES blood vessels IN THE GUT

What isolation involves a wearing gloves when entering room & changing after contact w/infectious material; involves SINGLE USE EQUIPMENT; used in illnesses such as MRSA & C-DIFF?

Contact

What is Chvostek's sign?

Contraction of the ipsilateral facial muscles elicited by tapping facial nerve (just anterior to ear)

What are presentations that are associated with urethral/bladder trauma?

Contusions or rupture caused by straddle injuries, genital trauma, or foreign bodies Associated w/PELVIC FRACTURES S/s urge to but inability or difficulty w/voiding; blood at urinary meatus; high-riding prostate Dx: Cystogram Tx: Suprapubic catheter placed by urologist for 7-10 days

What stage of pertussis presents w/gradual recovery?

Convalescent

How is heat exhaustion managed?

Cool the patient

What are s/s of compensating hypovolemic shock?

Cool, clammy skin & normal sensorium Vasoconstriction of compensatory shock could result in normal SBP & elevated DBP

How is asphalt exposure managed in the ED?

Cool; apply fat emollients (e.g. abx ointment) to loosen; peel off when cool

How are phenol (carbolic acid) exposures managed?

Copious irrigation w/50% PEG (miralax) & H2O

Crofab/Anavip is not indicated for what?

Coral snakes

What type of snake has neurotoxic venom that can cause RESPIRATORY PARALYSIS?

Coral snakes (Red on yellow kill a fellow; red on black, venom lack)

What is CAUDA EQUINA syndrome?

Cord compression of L5-S1 "horse-tail"; from fall onto coccyx resulting in "SADDLE ANESTHESIA"; sciatica-type back pain; bowel, bladder, & sexual dysfx

Inflammation of epithelium of cornea d/t trauma, bacterial/fungal/parasitic/viral infection is called?

Corneal Ulcerations

Patient presents with c/o sensation of a foreign body, pain, tearing, eyelid spasms & photophobia. What is the possible diagnosis & what is the management?

Corneal abrasion Topical anesthetic; fluorescein staining & examination. Patching is NOT recommended

Bradycardia

Correct underlying cause (e.g. meds or respiratory distress) Atropine 0.5mg Q3-5mins for low degree blocks; max of 3mg Transcutaneous pacing- evaluate for electrical spike & mechanical capture (check your femoral pulse NOT CAROTID) Epinephrine or Dopamine infusion

What are s/s of pyelonephritis? How is it managed?

Costovertebral angle flank pain; gross hematuria; malodorous urine; fever; chills; pyuria; leukocytosis; hematuria ABX; admit patient if pregnant

How can atelectasis be prevented?

Cough & deep breath Incentive spirometry

What is Chronic Bronchitis?

Cough & sputum production for at least 3 months during 2 consecutive years; "blue bloater"; stocky build; polycythemia (thicker blood & increased Hgb->clots); COR PULMONALE; enlarged heart on CXR

What is the management for a tibial/fibular fracture?

Cover w/sterile dressing (many are OPEN FRACTURES) Splint as found unless neurovascular compromise present Surgical repair or casting

What are the s/s of diverticulitis & how is it managed?

Crampy pain that localized in LLQ; N/V; fever; alternating episodes of explosive diarrhea & severe constipation NPO; NG tubes; IVFs; ABX; anticholinergics (reduce spasms); surgery if ruptured diverticulum

What cranial nerve is involved w/trigeminal neuralgia?

Cranial Nerve 5

What cranial nerve is involved w/Bell's Palsy?

Cranial Nerve 7

RACEMIC EPINEPHRINE & humidification are the treatment of choice for what?

Croup

What are examples that lead to poor outcomes for a traumatic amputation?

Crush injuries; contamination; comorbidies; age; poor nutrition

Patients with what disorder have an increased risk of HTN, obesity glucose intolerance, FRACTURES, impaired immune function & wound healing?

Cushings

What blocks aerobic metabolism—>cellular hypoxia refractory to oxygenation?

Cyanide

What interferes w/cellular respiration (causes a oxyhemoglobin curve to shift to the left) d/t hemoglobin holding onto O2?

Cyanide poisoning

Patient presents with c/o HA, dizziness, & seizure like activity. You notice a smell of BITTER ALMONDS on their breath. What should you suspect & how is it managed?

Cyanide poisoning. Tx w/cyanide kit (INHALED AMYL NITRITE), IV sodium nitrite, IV sodium thiosulfate Or Cyanokit -HYDROXOCOBALAMIN (vitamin B12) (will cause red urine) Sodium bicarbonate to tx metabolic acidosis.

What is the antidote for cyanide poisoning?

Cyanokit (hydroxocobalamin)

What hepatitis requires you to have B to get it?

D

What is acute adrenal insufficiency?

D/t SUDDEN DISCONTINUATION OF GLUCOCRTICOIDS; infection; trauma; surgery; burns S/s: s/o shock (tachycardia & HYPOTENSION); HYPONATREMIA; HYPOGLYCEMIA; HYPERKALEMIA Dx: CORTISOL LEVEL; cmp Tx HYPOVOLEMIC SHOCK W/0.9% NS (tx's hyponatremia & dilutes potassium too); replace glucocorticoids (HYDROCORTISONE); & mineralocorticoids (dexamethasone) immediately; DEXTROSE for hypoglycemia, insulin/dextrose for hyperkalemia

Hypokalemia

D/t V/D, overuse of diuretics, acute alcoholism, cirrhosis, intestinal tract diseases, GI suctioning S/S: muscle weakness, abdominal distention, paralytic ileus, shallow respirations, leg cramps, hyporeflexia Tx: replace K (don't exceed 40mEq). Correct hypomagnesemia

Intrarenal AKI/RF

D/t acute tubular necrosis; nephrotic agents (aminoglycosides, NSAIDS, contrast); crush injury; RHABDOMYOLYSIS; HTN; DM Tx: based on cause

Hypomagnesemia

D/t alcoholism, cirrhosis, malnutrition, malabsorption syndromes, diuretic therapy S/S: agitation, increased DTRs, muscle weakness, & twitching Tx: Magnesium sulfate 1-2gm IV (rapid if emergency; over 2 hrs if non-emergent) or IM; monitor for magnesium toxicity (respiratory depression, hypotension, decreased DTR)

What is peripheral vascular disease?

D/t damaged venous valves S/s: leg pain; delayed wound healing; cellulitis; ulcers

anaphylactic shock

D/t histamine release

Hypocalcemia

D/t pancreatitis; low albumin levels; CCB overdose; multiple transfusions; hyperphosphatemia S/s: MUSCLE TWITCHING; + chvostek's sign; + Trousseau's sign; hyperactive DTR; prolonged QT-i Tx: replace calcium by adding 2 amps of calcium gluconate 10% in D5W over 10-20mins

Why is fluid resuscitation a priority in DKA?

D/t patient being in HYPOVOLEMIC SHOCK

What is an open pneumothorax?

D/t penetrating trauma; visible open chest wound w/SUCKING SOUND & BUBBLING OF BLOOD around wound & SQ EMPHYSEMA Tx: Cover w/3 SIDED OCCLUSIVE (non porous) dressing at END EXHALATION & prepare for chest tube; REMOVE DRESSING if TENSION PNEUMO DEVELOPS (decreasing O2 sats)

What is Noncardiogenic pulmonary edema?

D/t submersion injury; rapid ascent while scuba diving; HAPE (high altitude pulmonary edema); inhalation of toxic gases; heroin OD S/s: SOB; tachypnea; cough; crackles; wheezes; pink frothy sputum Tx: JUDICIOUS fluid administration; NIV; intubation/ventilation w/low tidal volume (4-6ml/kg d/t sick lungs) & HIGH PEEP to decrease chance of ARDS

Why is atropine ineffective in a transplanted heart?

D/t the heart being denervated

Prinzmetal variant angina

D/t vasospasm of coronary arteries; NOT PLAQUE. Hallmark sign is CP at REST; usually between MIDNIGHT & EARLY MORNING. Cyclic in nature. More common in women Seen in stimulant use. Avoid beta blockers if stimulant related. ST elevation w/WIDENED R wave. Tx: Nitrates & CCB. *Betablockers are contraindicated; may exacerbate vasospasm d/t unopposed ALPHA stimulation

How is organophosphate poisoning managed?

DECONTAMINATE FIRST Atropine & 2PAM until bronchial secretions dry

How is radiation exposure managed?

DECONTAMINATE; abcs; lab specimens; consult specialists

What lab results are seen with DIC?

DECREASED PLATELETS, FIBRINOGEN, hemoglobin & hematocrit INCREASED FIBRIN DEGRADATION PRODUCTS, d-dimer & bleeding times (PT,PTT)

What is the antidote for iron toxicity?

DEFEROXAMINE (turns urine PINK as iron is excreted)

What disorder is characterized by inadequate release of ADH leading to high urinary outputs which can lead to severe dehydration?

DI

Cardinal sign of SIADH

DILUTIONAL HYPONATREMIA

What lab changes are seen with SIADH?

DILUTIONAL HYPONATREMIA (risk of SEIZURES); increased specific gravity

In what type of situation is the afterload low?

DISTRIBUTIVE SHOCK

What causes afterload to be decreased?

DISTRIBUTIVE shock (anaphylactic, neurogenic, or septic) Tx: vasopressors (epinephrine, norepinephrine, phenylephrine)

When is afterload is DECREASED?

DISTRIBUTIVE shocks (neurogenic, septic, anaphylactic) & vasoDILATORS Tx: VasoPRESSORS (Norephinephrine)

How are missile injuries managed?

DO NOT REMOVE IMPALED OBJECTS. ABCS. Bleeding control. STABILIZE ANYH OBJECT; PRESERVE ANY FORENSIC EVIDENCE. PROJECTILE PATH AN BE UNPREDICTABLE. Most chest & abdominal wounds require SURGICAL INTERVENTION

What injuries are seen more in children from a LAP RESTRAINT?

DUODENAL (HOLLOW) ORGAN INJRY & CHANCE FRACTURE (T12-L2)

A pt with peripheral venous insufficiency is at risk for what?

DVT

What are extrapyramidal reactions?

DYSTONIA (muscle contractions); oculogyric crisis (deviation of eyes); torticollis (spasms of neck muscles w/twisting to side) Seen w/Haldol, Thorazine, Droperidol

Abnormal extension posturing is what?

Decerbrate

What management is required for nerve agent toxicity?

Decontaminate Atropine until bronchial secretions dry 2-PAM Benzodiazepines

What is the priority treatment for chemical burns?

Decontamination w/COPIOUS IRRIGATION

Abnormal flexion posturing is what?

Decorticate

NTG & nipride do what to afterload?

Decrease

With testicular torsion what is seen on the Doppler U/S?

Decrease or absence of vascular flow

What are the s/s of PARASYMPATHETIC (CHRONIC) pain?

Decreased BP & HR; slowed speech; pupillary constriction.

How can you tell a patient with a spontaneous pneumothorax has improved?

Decreased RR

Prerenal AKI/RF

Decreased blood flow to kidneys E.g. hypovolemia, decreased C.O Tx: based on cause

What are S/S of a ruptured diaphragm? How is it managed?

Decreased chest expansion on affected side; tachypnea; decreased SpO2; peristaltic "gurgling" sounds audible in lower to mid chest; progressive scaphoid abdomen Emergency laparotomy required.

What are the s/s of a fat embolism & what is complication of it?

Decreased mental status (restlessness; agitation); respiratory distress (dyspnea; hypoxia); PETECHIAL RASH to head, neck, anterior thorax, & axilla; patchy infiltrates on CXR Complication of ARDS

What are the s/s of SIADH?

Decreased u/o; WATER INTOXICATION; cerebral edema

What are the s/s of orbital cellulitis & how is it managed?

Decreased visual acuity; proptosis (bulging eyeball); painful ocular movement; fever Warm compresses, ABX, ophthalmology consult

Why is a pelvic binder applied to a pelvic ring fracture? And where should it lay?

Decreases available space for blood to accumulate Lays over bilateral greater trochanters w/enough pressure to close the pelvic ring

What does magnesium sulfate do for gestational HTN?

Decreases the seizure threshold

Why must you be hemodynamically stable for NIPPV?

Decreases venous return to heart; risk for aspiration; barotrauma

What is a s/o effective treatment in a patient w/serotonin syndrome?

Decreasing temperature

What is the antidote for Iron?

Deferoxamine

Anemia

Deficiency of healthy RBCs to carry oxygen to body S/s: fatigue, malaise, pallor, SOB, pica (craving clay)

What are the s/s of hemothorax & how is it managed?

Defined as a rapid accumulation of blood of more than 1500ml of blood in the pleural space (massive hemothorax) Restlessness; anxiety; dyspnea; tachypnea; chest pain; s/o shock; flat neck veins; decreased breath sounds on injured side; dullness to percussion. Insert 2 large bore IVs; TXA therapy; large bore chest tube inserted at the 5th ICS, AAL or MAL connected to an atrium. Autotransfusion. Surgery is likely for a massive hemothorax.

Patients on lithium should avoid what to prevent lithium toxicity?

Dehydrated; meds like NSAIDS, diuretics, tetracycline, phenytoin, & cyclosporins

What is multiple sclerosis?

Demyelination of axons-> weakness, unsteady gait & altered sensation in extremities & face Tx'd w/steroids & immunosuppressants (INTERFERON)

Adrenal Crisis

Depletion of adrenal glucocorticoids & mineralocorticoids from ADDISON's or abrupt withdrawal of LT steroid use S/S: resting tachycardia; orthostatic hypotension; s/o HYPOVOLEMIC shock (dry mm, poor skin turgor) Dx: LOW sodium & glucose; HIGH potassium, calcium, BUN, & Hct (dehydration) Tx: ABCs. 0.9% NS to tx hyponatremia & hypovolemic shock. Replace glucocorticoids (hydrocortisone) & mineralocorticoids (dexamethasone). Treat hypoglycemia w/DEXTROSE. Administer insulin/dextrose is potassium elevated.

How is DI treated?

Desmopressin (DDAVP) or aqueous VASOPRESSIN then FLUID RESUSCITATION

Financial abuse is mostly seen with who?

Developmenally disabled & elderly who require caregivers.

DKA

Develops RAPIDLY. Type 1 DM S/s: ACETONE odor of breath, abdominal pain, KUSSMAUL respirations, tachycardia, hypotension, N/V Labs: glucose >250mg/dL; pH <7.35 (METABOLIC ACIDOSIS), HCO3 <20mmol/L (wide ANION GAP), ketonemia, elevated BUN (dehydration). Ketones + in urine Tx: IV 0.9NS at 15-20ml/kg (adult) or 10-20ml/kg (peds) over 1-2hrs; regular insulin infusion (NO boluses) at 0.1 u/kg/hr (when POTASSIUM STABLE) until ANION GAP CLOSES. Add Dextrose containing fluid when blood sugar 200-250. Acidosis corrects self w/proper fluid administration. Monitor blood sugar Q1-2hrs. Don't D/c insulin until pH>7.3

Insufficient secretion of ADH is called?

Diabetes Insipidus

Low levels of ADH can lead to?

Diabetes Insipidus

What are the features of a venomous snake?

Diamond (triangular) shaped heads; vertical, elliptical pupils; fangs; single row of caudal plates

What are s/s of gastroenteritis?

Diarrhea w/N/V; crampy lower abdominal pain; s/o dehydration; spleen enlargement (bacterial)

What are the s/s of Giardia & how is it managed?

Diarrhea; steatorrhea (fatty feces); abdominal cramping; BLOATING; weight loss Rehydrate. METRONIDAZOLE (flagyl); tinidazole

What is orthopnea?

Difficulty breathing w/laying down; seen w/pulmonary edema

What are the s/s of peritonitis & how is it managed?

Diffuse pain w/rebound tenderness & guarding; fever; s/o shock Fluid resuscitation; ABX (3rd generation cephalosporin- Cefotaxime)

What are the s/s of peritonitis

Diffuse pain; REBOUND TENDERNESS; GUARDING; fever

YELLOW/GREEN HALOS IN VISION, n/v, ventricular DYSRHYTHMIAS, heart blocks, & hypotension is commonly seen in what toxicity? How is it managed?

Digitalis Glycosides Tx w/digoxin immune fab (digibind; digifab)

Patient presents with GI distress & YELLOW-GREEN HALOS in vision. What is the potential cause & how is it managed?

Digoxin Toxicity Tx w/digi Bind (Digi Fab)

A yellowish discoloration to the vision is associated with what?

Digoxin toxicity

Cycloplegic agents do what?

Dilate & paralyze pupil

What are the SYMPATHETIC responses?

Dilates pupils; inhibits salivation & tearing; CONSTRICTS blood vessels; RELAXES airways; ACCELERATES HR; stimulates glucose production/release; inhibits digestion; stimulates SECRETION of epinephrine/norepinephrine from adrenal medulla

Thick, gray pseudomembranous coating on tonsils & pharynx is seen with?

Diphtheria

A thick gray pseudomembranous coating in back of throat is seen in what? How is it managed?

Diphtheria. Tx w/PCN

What are common causes of an anterior shoulder dislocation & how does it present?

Direct blow to shoulder or fall on extended arm Inability to raise area or ADDuct

What are possible causes of a patellar fracture & how is it managed?

Direct trauma d/t fall or impact w/dashboard Commonly OPEN FRACTURES Surgical repair or long leg splint

What is a Monteggia's fracture?

Dislocation of radius; fracture of the ulna

Maternal Cardiac Arrest

Displace uterus to LEFT. Prepare for emergency C-section.

BP DIFFERENCE OF 20MM hg between arms can be seen in what?

Dissecting AORTIC ANEURYSM

Lower extremity weakness, pulsating mass, & RIPPING/TEARING CHEST PAIN that radiates to back is seen with?

Dissecting aortic abdominal aneurysm

Anaphylactic shock is an example of what type of shock?

Distributive

Neurogenic shock is an example of what kind of shock?

Distributive

Septic shock is a type of what?

Distributive

What form of shock is Anaphylaxis?

Distributive Shock.

What shock is due to the maldistribution of blood? How is it treated?

Distributive. Tx w/vasopressors

What decreases preload?

Diuretics

What is the inflammation of the sigmoid (LARGE) colon from entrapped fecal matter?

Diverticulitis

What is No charcoal in PHAILS mean?

Do not administer charcoal for gastric decontamination related to: Pesticides/Petroleum Hydrocarbons/Heavy metals Acids/Alkalis/Alcohols Iron Lithium Solvents

What are discharge instructions for a patient w/Hepatitis B, C or D?

Do not donate blood. Practice safe sex.

What are discharge instructions for a patient w/ Hepatitis A or E?

Do not prepare food for others

What are examples of POSITIVE INOTROPICS?

Dobutamine & Dopamine. But correct HYPOVOLEMIA first.

What type of bite is often associated with underlying CRUSH INJURY?

Dog Bites; leave open & consider rabies prophylaxis or watch dog closely

While turning your supine patient's head to the left you notice that their eyes deviate to the RIGHT. Which OCULOCEPHALIC reflex is this an example of?

Doll's eyes. And the fact that their eyes deviated to the OPPOSITE side that their head was turned showed that their brainstem is intact so YAY for them!

What discharge teaching should be provided for a patient w/Chlamydia?

Doxycycline puts you at risk for SUNBURN Monitor for s/o WORSENING INFECTION— IT MAY BE PID Clindamycin cream weakens condoms (use other form of birth control)

What discharge teaching is needed for a patient with kidney stones?

Drink large amounts of water; STRAIN URINE; no caffeine

Sickle Cell Disease education

Drink lots of fluids Avoid getting over-heated or very cold Treat infections quickly Decrease physical/emotional stress Avoid high altitudes Avoid smoking

Where does the weakness associated with myasthenia gravis typically start?

Drooping of the eyelid or difficulty keeping eyelid closed.

What isolation involves wearing a mask when w/in 3 FEET of pt; a surgical mask for patient when outside of private room; & is used for illnesses such as MENINGITIS, PERTUSSIS, MUMPS, & RUBELLA?

Droplet

What are CHRONOTROPES

Drugs that alter the HR at the SA node + Increase (Atropine, Milrinone) - Decrease (Cardizem)

What are the adverse effects of ACE inhibitors?

Dry NP cough (leads to noncompliance) Angioedema; renal impairment (esp w/NSAIDS)

What are the s/s of appendicitis?

Dull, steady PERIUMBILICAL PAIN w/anorexia & nausea early RLQ PAIN (MCBURNEY'S POINT) later (around 24-48rs) w/REBOUND TENDERNESS (ROVSING'S SIGN-hard to assess in peds); markle sign (heel jar); obturator sign (pain on R.hip flexion); psoas sign (pain on extension of hip)

When you are percussing the abdomen, what sound should be heard over the SOLID organs?

Dullness

What are the 3 types of PUD?

Duodenal (ages 30-55; pain prior to eating; relieved by eating & antacid use) Gastric (ages 55-70; PAIN AFTER EATING; TENDS TO BE CHRONIC) Stress (d/t stress of severe illness or trauma)

Lap Restraint Injury

Duodenal (hollow organ) injury & Chance fracture (T12-L2); seen more in children. Prepare for OR d/t contamination.

When is testicular torsion most common?

During ADOLESCENCE

What 4 elements must be present to establish malpractice?

Duty-nurse has the duty to perform care. Breach of duty- person failed to complete/adequately complete duty Proximate Causation- breach caused damage (physical,emotional, psychological or social) Damages- need to exist.

How do you dx & tx hypertensive emergency/crisis?

Dx w/12 lead ekg, CXR, U/a & BUN/Creat Tx w/ O2; IV; art line (need to do an ALLEN's test for RADIAL placement). Infuse Nitroglycerin or Nitroprusside to slowly decrease MAP by a Maximum of 25% in 1st 2hrs of treatment. Labetalol has a slower onset but is safer & effective during pregnancy

A patient w/a carboxyhemoglobin of >20% could be exhibiting what?

Dyspnea & confusion

What are the s/s of a tracheobronchial injury? How is it managed?

Dyspnea or tachypnea; dysphagia; dysphonia (hoarseness); airway obstruction; subcutaneous emphysema in the neck & face; decreased breath sounds; hemoptysis; pneumothorax w/continuous air leak in atrium; pneumomediastinum Avoid intubation d/t possibly causing more damage or contribute further to airway occlusion; use fiber optic bronchoscopy instead.

What are the s/s of flail chest & how is it treated?

Dyspnea, chest wall pain & contusions; diminished breath sounds Prepare for intubation & mechanical ventilation

What are the S/S of pulmonary contusion? How is it managed?

Dyspnea; increased WOB; hypoxia; chest wall contusion Maintain SpO2 94-98%; administer IVF judiciously

What are signs of respiratory distress?

Dyspnea; nasal flaring; retractions; TRIPOD POSITIONING; pallor; cyanosis; difficulty speaking in full sentences; mental status change; grunting

What are the s/s of asthma?

Dyspnea; tachypnea; Expiratory wheezing; prolonged Expiratory phase

EKG changes w/hypomagnesemia

Dysrhythmias such as prolonged QTi; TORSADES; VT/VF

What is the exclusion criteria of TPA?

E/o intracranial bleed, tumor, head trauma, AV malformation; current internal bleeding; PLATELETS <100K

What education would you provide your patient re: esophagitis/GERD?

EAT SMALL, FREQUENT MEALS; do not lie flat w/in 2hrs of eating; avoid chocolate, spicy foods, tobacco, caffeine, etoh, & carbonates drinks; loss some weight

What should be performed on adult patient with asthma due to similar symptoms?

EKG

How is epistaxis managed?

ELEVATE HOB; SUCTION AVAILABLE; IVF; pinch nostrils firmly for 10-15mins (anterior) Progress to cauterizing w/silver nitrate or electrocautery; nasal packing soaked in TXA, phenylephrine, or lidocaine w/epinephrine.

What are the labs levels seen with hyperthyroidism?

ELEVATED T3 & T4; LOW TSH

What would you triage an ectopic pregnancy?

ESI 2

How should a sickle cell patient be triaged?

ESI 2 (d/t sickle cell crisis; at risk for chest syndrome & sequestration)

How is neutropenia triaged?

ESI level 2

How is vaginal bleeding managed?

ESTROGEN THERAPY; fluid resuscitation; iron supplement (anemia); gynecology referral

What occurs during the 2nd stage of Lyme's Disease

Early disseminated disease; days to weeks s/p CRANIAL NEUROPATHIES (BELL'S PALSY); cardiomyopathies (AV BLOCK); & arthralgias

What illness presents as a viral hemorrhagic fever & petechiae?

Ebola

What are the s/s of orbital wall fracture & how is it managed?

Ecchymosis; OCULAR ENTRAPMENT (UNABLE TO LOOK UP W/AFFECTED EYE-CN 3); diplopia; swelling; subconjunctival petechiae; infraorbital hypesthesia (reduced sensation) Management: elevate HOB; ICE PACK (NOT CHEMICAL but real ice); educate on ophthalmic f/u; using ice packs to face; AVOID VALSALVA MANEUVER, STRAINING & BLOWING NOSE

Abdominal pain w/referred shoulder pain & ORTHOSTATIC HYPOTENSION is seen with?

Ectopic pregnancy

Fertilized ovum implants in the Fallopian tube, ovary, or abdominal cavity leading to pelvic pain that possibly radiates to shoulder & vaginal bleeding is known as what? How is it managed?

Ectopic pregnancy Pregnancy test, pelvic exam, quantitative B-hCG; CBC; TXM; u/s; large bore IV; METHOTREXATE IM; surgery if ruptured or in shock. Emotional support

What are causes to seizures?

Electrolyte disturbances; ETOH w/d; hypoxia; meningitis; illicit drugs; trauma; tumor; stroke; febrile.

How is a corneal donation managed?

Elevate HOB 20-30degrees; tape eyelids shut w/paper tape; APPLY ICE OVER EYES

How do you manage increased ICP?

Elevate HOB to 30-45 degrees Keep NEUTRAL ALIGNMENT- chin to umbilicus AVOID HIP FLEXION (increases IAP which increases ICP) ALLOW NO HYPOTENSION (SBP 90-100), hypoxia (keep SpO2 >98), hypercarbia (keep CO2 35-37) or hyponatremia/serum osmolality Monitor ICP pressure (transducer at the level of the tragus) Administer osmotic diuretic- Osmitrol (Mannitol) 1 gm/kg or hypertonic saline as ordered (MONITOR ELECTROLYTE DISORDERS & dehydration) Avoid venous compression of neck (remove that rigid cervical collar) Maintain normothermia (treat fever aggressively) Dark quiet room & contra agitation Limit visitors appropriately Speak softly

What is MODS (multiple organ dysfunction syndrome)?

Elevated BUN/creat >2.0; elevated ammonia & bilirubin (>2.0); elevated CO2; thrombocytopenia; coagulopathy

Liver Failure/Cirrhosis

Elevated LFTs & direct bilirubin, jaundice (icterus eyes), ascites, asterixis, spider angiomas R/o GI bleed from PORTAL HTN & ESOPHAGEAL VARICES Give lactulose for elevated ammonia levels d/t hepatic encephalopathy Tx is effective if increase in LOC

EL in HELLP stands for?

Elevated Liver enzymes

How is cholecystitis diagnosed?

Elevated WBC (LEUKOCYTOSIS); elevated ALT & bilirubin; abdominal u/s

What is Cushing's syndrome?

Elevated levels of cortisol or glucocorticoids (decadron)—>moon face, buffalo hump, fat above clavicles, hirsutism (male pattern hair growth) w/baldness, insomnia Tx: TAPER OFF GLUCOCORTICOIDS

What is thrombocytosis?

Elevated platelet count

Grave's disease results in what?

Elevated thyroid levels

What is Prehn's sign and when is it seen?

Elevation relieves pain. Epididymitis

What class of tooth fracture is fracture that extends through the enamel of the teeth—>tooth to appear chalky white?

Ellis class I

What class of tooth fracture extends through the enamel & dentin—>tooth to appear yellowish (exposed dentin)?

Ellis class II

What class of tooth fracture extends into pulp & nerve of tooth—>pink or bloody appearance?

Ellis class III

What is an ectopic pregnancy?

Embryo plants outside the uterine cavity (most often in Fallopian tube); ruptures usually around 6-8 weeks Most common with h/o PID S/s: sudden onset of severe unilateral pelvic pain RADIATING TO SHOULDER (KEHR'S SIGN); s/o shock; possible vaginal bleeding; ORTHOSTATIC HYPOTENSION Dx: HcG +; CBC, TXM; transvaginal u/s

Patient presents with a high pressure injection injury. How is it managed?

Emergency surgery for debridement & exploration; possible amputation; tetanus prophylaxis; analgesia; ABX

If advanced uterine size prevents resuscitation of the unstable mother what is required?

Emergent C-section.

How is a hip dislocation managed?

Emergently w/in 6 hours to prevent FEMORAL HEAD NECROSIS

Irreversible, progressive damage to the alveoli that causes tachypnea, accessory muscle use, tripod position, pursed lip breathing & over inflation of the lung & diaphragm on xray is what?

Emphysema

Where should you AVOID placing XYLOCAINE W/EPINEPHRINE (LET) & why?

End arterial points (ears, nose, fingers, toes & "hose"-penis Limited circulation & INCREASED RISK OF INFECTION

Fever; pleuritic pain; NEW ONSET MURMUR; Janeway lesions; Roth spots; and possible JVD (if involving tricuspid valve) is seen in what? How is it managed?

Endocarditis ABX STAT.

What is the thumbprint sign? And what is it seen with?

Enlarged epiglottis seen on LATERAL neck xray Epiglotittis

What is cor pulmonale?

Enlargement & failure of the RV

What is Raynaud's disease?

Episodic intense VASOSPASM OF THE DIGITS, nose, or ears in response to stress/cold; buerger's disease from smoking

infectious mononucleosis

Epstein-Barr Virus Transmitted by saliva. (Kissing disease). S/s: fever, malaise, exudative pharyngitis; cervical lymphadenopathy (posterior > anterior); rash s/p amoxicillin tx Labs: heterophile antibody test Tx: supportive care. AVOID contact sports (d/t ENLARGED SPLEEN)

What causes mono?

Epstein-Barr virus

Difficulty swallowing & drooling from "something stuck" or "food bolus" is called? How is it managed?

Esophageal obstruction Keep upright; consider carbonated beverage; GLUCAGON IV or NTG sl

A patient presents to the ED with c/o BURNING SUBSTERNAL PAIN that peaked 30-60mins s/p eating; what is going on? And what is it?

Esophagitis/GERD- reflux of gastric contents into esophagus—>inflammation

What is status asthmaticus?

Exacerbation that is unresponsive to treatment—>hypoxia; hypercarbia; respiratory failure

SIADH

Excessive ADH secretion—> water intoxication—>DILUTIONAL hyponatremia, decreased u/o, & hypo-osmolality Pituitary gland releases EXCESSIVE amts of ADH d/t brain lesions, trauma, infections. Seen in head injuries. OAT CELL CARCINOMA (lung cancer) S/s: decreased u/o (WATER INTOXICATION), muscle cramps, sudden weight gain, confusion, SEIZURE activity Labs: DILUTIONAL HYPOnatremia (<125) & serum osmolality <280, decreased creatinine Tx: free H2O restriction; hypertonic saline. Lasix. Dilantin. *Better if SODIUM INCREASES* Risk for SEIZURES.

Pheochromocytoma

Excessive catecholamine secretion (epinephrine & norepinephrine) from a benign tumor of the ADRENAL MEDULLA S/s: HA; diaphoresis; tachycardia; palpitations; HTN; visual changes Tx: PHENTOLAMINE (alpha blocker), Nitroprusside, or Nicardipine while awaiting tumor removal. Non-selective beta blockers (e.g. propranolol) are CONTRAINDICATED w/alpha blockade.

When do you see postpartum hemorrhage?

Excessive vaginal bleeding s/p delivery/abortion & up to 6 weeks PP S/s: bright red bleeding; s/o shock; boggy uterus

What is a cluster HA?

Excruciating, unilateral (periaorbital/temporal); episodic (multiple per day, short lived); EXCESSIVE TEARING (LACRIMATION); nasal congestion on affected side

What happens during a hypersensitivity reaction?

Exposure to allergen—>IgE antibodies produced—>histamine & leukotriene react—>VASODILATION & mucus production

What type of consent is obtained by patient agreeing to treatment?

Expressed

How is a patella/knee dislocation managed?

Extend knee to reduce; compression; place in KNEE IMMOBILIZER.

What are the s/s of retrobulbar hematoma & how is it managed?

Eye pain; PROptosis (PROtrusion of the eye); decreased visual acuity; elevated IOP CT of orbit or ocular u/s; emergent ophthalmology consult; lateral canthotomy (surgical exposure of the lateral canthal tendon); mannitol; methylprednisolone/hydrocortisone; acetazolamide (diamox; diuretic)

Endocarditis: FROM JANE

F: fever R: roth's spots O: osler's nodes M: murmur J: Janeway lesions A: anemia N: nail hemorrhage E: emboli

What are the risks if Lyme disease is left untreated?

FACIAL PARALYSIS; ARTHRITIS; MYOCARDITIS

What is neglect?

FAILURE TO PROVIDE BASIC NEEDS Medical, physical, educational, etc

If a patient presents to the ED with c/o L. SHOULDER PAIN (esp following Mono) what should be done?

FAST EXAM.

What diagnostic test to done INITIALLY to assess for abdominal injuries?

FAST exam

What is a complication of a hip dislocation?

FEMORAL HEAD NECROSIS

What are signs of fetal distress?

FETAL TACHYCARDIA (FHT >160bpm); Loss of variability (HR increased w/contraction); decreased fetal movement

What is the first sign of fetal distress?

FHT > 160

What should be used if intubation is necessary for a tracheobronchial injury?

FIBEROPTIC BRONCHOSCOPY

What is Grey-Turner's sign?

FLANK ECCHYMOSIS

What is the treatment of choice for bowel obstruction?

FLUID & ELECTROLYTE REPLACEMENT for HYPOVOLEMIC SHOCK; bowel rest; prepare for surgery.

How is foreign body in the ear managed?

FLYING INSECTS MAY FLY TO THE LIGHT SUFFOCATE LIVE INSECT W/VISCOUS LIDOCAINE OR MINERAL OIL Irrigate & ATTACH WALL SUCTION Consider sedation then use forceps to remove object w/o pushing it deeper as a last resort. *If foreign body is of an organic material (breads; peas; beans) use an alcohol based solution in irrigation

What do we monitor for during administration of TPA?

FOR DECREASED LOC; bleeding during infusion

What is FOUR?

FULL OUTLINE UNRESPONSIVE SCORE. Useful for ventilated patients since it includes reflexes & respiratory patterns.

How is postpartum hemorrhage treated?

FUNDAL MASSAGE w/suprapubic pressure (enhances that uterine agony) Treat shock with BLOOD PRODUCTS STAT OXYTOCIN (PITOCIN) (stimulates uterine atony-contractions)

When is an NG contraindicated?

Facial/head trauma or suspected basilar skull fracture

Hemophilia B if a deficiency of what factor?

Factor IX; aka Christmas disease.

Hemophilia A

Factor VIII deficiency

Hemophilia A is a deficiency of what factor?

Factor VIII; most common type

What are delusions?

False personal beliefs about oneself

What is a characteristic of a lightening injury?

Feathering or fern appearance to skin Aka LICHTENBERG figures.

SBO

Fecal vomiting w/minimal distention

What is anxiety?

Feeling of nervousness about an imminent event or something an uncertain outcome

What are s/s of suicide?

Feeling of worthless; hopeless; helpless; indifference; social isolation

What is the first sign of FETAL DISTRESS?

Fetal tachycardia (FHT> 160 bpm)

What can make sickle cell disease an ESI 2?

Fever or chest pain

What are symptoms that can be seen w/a postpartum infection? And how are they treated?

Fever; tenderness; foul-smelling lochia; up to 10 days s/p delivery. Tx w/sepsis protocols; IVFs; STAT ABX.

To test the median nerve what motion do you instruct the patient to do?

Fist bump; "power to the people"

How are crutches fitted?

Fit w/wearing shoe on unaffected side; ARM PIECE 2 INCHES BELOW AXILLA; elbow at 30 DEGREE ANGLE; keep crutches 6 inches to side for stable gait.

A fracture of 3+ sequential ribs in 2 or more locations resulting in a free floating segment creating paradoxical chest wall movement is known as?

Flail chest

What are GU injury red flags?

Flank, back, or scrotal pain; inability to void; gross hematuria; blood at the urinary meatus or vaginal introitus; perineal ecchymosis; high-riding prostate

A patient w/a COHb (carboxyhemoglobin) of <20% could be exhibiting what?

Flu like symptoms & HA

hypervolemic hyponatremia

Fluid overload; SIADH; excessive H2O ingestion. Tx'd w/3% saline

What is the treatment for hepatitis?

Fluid resuscitation for acute A&E. INTERFERON or Ribavirin for CHRONIC hepatitis.

How is CNS stimulant ingestion managed?

Fluid resuscitation, activated charcoal, BENZODIAZEPINES, cool pt aggressively, vasodilators

What is the treatment of diverticulitis?

Fluid resuscitation; BOWEL REST; ABX; surgery if ruptured

Causes of macrocytic anemia

Folate deficiency; pernicious deficiency (Vitamin B12-cobalamin); liver disease; alcoholism

An elbow dislocation presents with & is managed by?

Following a fall on outstretched arm; elbow swelling & neurovascular compromise noted IMMEDIATE REDUCTION & SLING

What is the antidote for ethylene glycol?

Fomepizole

When should you consider an AUTOTRANSFUSION?

For significant blood loss for injury <4-6hrs Advantages: fresh, warm, whole blood w/o risk of transfusion rx Contraindicated in lower chest injury (risk of bowel injury)

What activity should be avoided w/an isolated orbital fracture?

Forcefully blowing nose

What is a CHANCE fracture?

Fracture of the T12-L2; seen in hyperflexion "LAPBELT" only injuries With concurrent hollow organ injury (BOWEL; stomach)

What is a basilar skull fracture?

Fracture of the skull base bone Altered LOC; HA; & vomiting

What are examples of pediatric abuse regarding fractures?

Fractures in varying stages of healing ; fractures in a child < 3yrs of age; fractures in the fingers of a non-mobile child; spiral fracture

What are common causes of compartment syndrome?

Fractures; external compression from CIRCUMFERENTIAL BURNS; casts; splints; ace bandages; edema; soft tissue/vascular injury (CRUSH INJURY, bleeding, recent surgery)

A patient with folic acid deficiency anemia should be educated on eating what?

Fresh fruits & vegetables (rich in folic acid) but not cooked (decreases folic acid)

What is numb, pale w/blisters tissue that is most commonly seen on fingertips, ears, nose, toes & cheeks?

Frost Bite

What are s/s of iron poisoning?

GI corrosion; hemorrhage; hepatic failure.

What is Activated charcoal w/sorbitol (cathartic) given for & how?

GI decontamination for Extended release or ENTERIC COATED MEDICATIONS (common w/APAP OD) Given Q4-6hrs x12-24hrs.

How is a beta blocker toxicity managed?

GLUCAGON Tx bradycardia & HYPOGLYCEMIA

What is dementia?

GRADUAL onset of disorientation, memory loss; "SUN DOWNING"; lack of awareness of problem

patient presents with ASCENDING SYMMETRICAL PARALYSIS, tingling of the extremities following a viral infection. What is going on & what should be monitored?

GUILLAIN-BARRE Monitor breathing pattern d/t diaphragm paralysis

What ray is blocked by lead or concrete & requires special shielding?

Gamma

What alternative therapies may interact with anticoagulants & diabetics medicine? (The Gs)

Garlic, GINSENG (ENERGY DRINKS), ginkgo, ginger

What is PUD?

Gastric acid breaks down lining of stomach INCREASED IN NSAID USE & w/ helicobacter pylori Tx: H2 BLOCKERS or PPIs; stop NSAIDs; clarithromycin or amoxicillin for H. Pylori

What is the treatment of peritonitis?

Gastric tube; IVFs; analgesics; antiemetics; ABX

Inflammation of the stomach & intestinal lining is known as?

Gastroenteritis

Hyperactive bowel sounds are noted in?

Gastroenteritis & early bowel obstruction

A seizure which causes erratic electrical activity throughout the brain are what type of seizure?

Generalized

Absence or petit mal seizure can be classified as what?

Generalized

How do you assess a pelvic fracture?

Gentle COMPRESSION INWARD & DOWNWARD

How do you assess a pelvic fracture?

Gentle INWARD COMPRESSION & DOWN OVER SYMPHYSIS PUBIS (if no obvious injury); ONLY ONCE SO YOU DON'T DISLODGE A CLOT

What is rubella?

German (or 3 day measles) Contagious up to week prior to symptoms & week s/p rash Rash starts on face Complication: birth defects & arthritis

When you see ST depression in V1 & V2 where should you get an EKG from?

Get a POSTERIOR view; V7-V9

What is a protozoan parasite that spreads by water contaminated by fecal matter (I.e. campers/hikers)?

Giardia

An ascending paralysis which can cause respiratory insufficiency & paralysis is seen in what? What is the highest priority of care?

Gillian Barre Administration of high flow O2 until intubation & mechanical ventilation can be carried out

What is our blood pressure parameters w/TPA?

Give Labetalol or Cardene if SBP >185 or DBP >110

What does glucagon do?

Given IM; signals liver to convert stored glycogen into glucose NOT EFFECTIVE IN LIVER DISEASE & children < 3 years Watch for VOMITING

What type of foreign bodies can be clearly visualized clearly on an xray

Glass & metal

Loss of integrity of globe r/t trauma (penetrating-knife; blunt- r/t increase IOP) is called?

Globe rupture

Patient presents w/teardrop shaped pupil ; extrusion of aqueous/vitreous humor; decreased IOP & enophthalmos (posterior displacement of eyeball w/in the orbit). What is the possible diagnosis & how is it managed?

Globe rupture Secure any extruding object in place; shield eyes to minimize consensual movement; place in semi-Fowler's position; avoid topical medications

What is the antidote for beta blockers?

Glucagon; insulin

If you see CLEAR CSF leakage what do you check for?

Glucose

What is used for WHOLE BODY IRRIGATION & why?

Go-lytely or MiraLAX for BODY PACKERS (cocaine packs)

Acute arthritis w/URIC ACID CRYSTALS in synovial fluids is called?

Gouty arthritis

Increased uric acid production deposits in joints that causes intense pain, redness & tenderness to joints is caused? And how is it managed?

Gouty arthritis Colchicine & prednisone

What is epididymitis?

Gradual onset of scrotal pain; urinary frequency; urgency; urethral discharge (if d/t GC); pain relieved w/elevation (prehn's sign) & ice

What are 1st & 2nd Rib fractures associated with??

Great vessel (aortic dissection-WIDENED MEDIASTINUM) &/or Lung injury (Pneumothorax)

When should SUCCINYLCHOLINE NOT be used?

H/o malignant hyperthermia Penetrating eye injury Increased ICP Conditions that precipitate hyperkalemia (72hrs s/p burn, crush injury, renal failure)

What are the s/s of mastoiditis & how is it managed?

H/o otitis media; pain & swelling in mastoid area; ear pain; fever; possible TM rupture; HA; hearing loss Tx: admission; IV ABX; analgesics; surgical intervention

What are the s/s of meningitis?

HA; fever; nuchal rigidity; malaise; photophobia; photophobia

How do you assess CSF?

HALO test if it's bloody; check for glucose if it's clear

What dx tests would be ordered for a spontaneous AB?

HCG; blood type & RH; uterine U/s

On an ABG what is DIRECTLY PROPORTIONAL to pH?

HCO3

What is the treatment for a concussion?

HEAD INJURY PRECAUTIONS. APAP for pain; NO NARCOTICS. No caffeine to stimulate brain COGNITIVE REST. GRADUATED RETURN TO PLAY. MEDICAL CLEARANCE for return to play. Too early return- SECONDARY IMPACT syndrome from minor injury.

What causes NEUROGENIC DI?

HEAD INJURY; brain tumors; meningitis; PHENYTOIN; LITHIUM

What is a potential complication of abruptio placentae?

HELLP/DIC

How is sickle cell managed in the ED?

HIGH FLOW O2; IVFs for rehydration; ABX (infection); OPIOIDS; early stem cell transplantation (goal)

SWOLLEN LYMPH GLANDS; fever; fatigue; diarrhea; weight loss; ORAL CANDIDIASIS; SHINGLES are s/s of ?

HIV

What are s/s of a hypersensitivity reaction?

HIVES; URTICARIA; itchy eyes; sneezing; runny nose

What is the management for a CORNEAL DONATION?

HOB up to 30 degrees; dressing w/paper tape & ice over eyes

What is the goal HR during neonatal resuscitation?

HR >100

What causes afterload to be increased?

HTN & aortic stenosis Tx: NTG, nipride, ACE-I, CCB

When is afterload INCREASED?

HTN; aortic stenosis; Shock compensations Tx: vasoDILATORS (NTG)

What are s/s & tx of chronic kidney disease?

HTN; volume overload; HYPERKALEMIA; metabolic acidosis; anemia; uremic syndrome; bruises; pruritus Tx: correct electrolyte & fluid imbalances; dialysis; CRRT (if unstable)

What lab changes are seen in DI?

HYPERNATREMIA & increased serum osmolality despite polyuria Low urine specific gravity & urine osmolality

How is SIADH treated?

HYPERTONIC SALINE & LASIX then fluid resuscitation

How is SIADH treated?

HYPERTONIC SALINE or IV NS & lasix; WATER RESTRICTION

What are the common lab findings for a thyroid storm?

HYPERglycemia; HYPERcalcemia; metabolic ACIDOSIS; elevated thyroid levels

What electrolyte abnormality is seen with pancreatitis?

HYPOCALCEMIA

What are complications of pancreatitis?

HYPOCALCEMIA (Chvostek's & Trousseau's signs & Tetany); PLEURAL EFFUSIONS(starts w/L. Pleural); ARDS (monitor that respiratory status); hemorrhagic (look for Grey-Turner & Cullen signs); sepsis

What is a risk of the massive transfusion protocol?

HYPOCALCEMIA; hyperkalemia; & hypothermia

What risks does Massive Transfusion Protocol come with?

HYPOCALCEMIA; hyperkalemia; & hypothermia

What is the risk of MTP?

HYPOCALCEMIA; hyperkalemia; & hypothermia.

What are the common causes of prerenal ARF?

HYPOPERFUSION FROM HYPOVOLEMIC SHOCK NOT KIDNEYS

What is a complication of DI?

HYPOVOLEMIC SHOCK

What are the risks associated w/a pelvic fracture?

HYPOVOLEMIC SHOCK URETHRAL DAMAGE BLADDER RUPTURE

What are the s/s of an acute angle-closure Glaucoma & how is it managed?

Halos around lights; cloudy vision; "TUNNEL VISION"; HA; eye pain d/t increased IOP; nausea Increase HOB; miotic gtts; beta-blockers (timolol); carbonic anhydride inhibitors; antiemetics; narcotics

What do tocolytics do?

Halt premature labor & decrease postpartum bleeding

What are the causes of SIADH?

Head trauma; infections (MENINGITIS); malignancies (OAT CELL LUNGD CANCER); oral hypoglycemic/psychotropic drugs; general anesthetics

What is the temporal lobe responsible for?

Hearing; emotions

Atropine is ineffective for?

Heart transplants. Consider isoproterenol.

Lead, mercury, & arsenic are examples of what's?

Heavy metals

what are s/s of upper gi bleed?

Hematemesis; s/o SHOCK (dizziness; tachycardia)

What are s/s of a bladder injury & how is it managed?

Hematuria; suprapubic pain; difficulty voiding; ecchymosis around thighs; abdominal distention & guarding; hematuria present in U/A CT or cystogram; prepare for exploratory laparotomy & repair

H in HELLP stands for...?

Hemolysis

What are the risks of an open fracture?

Hemorrhage & infection

What are complications of diverticulitis?

Hemorrhage; perforation; intestinal obstruction

What are potential complications of CNS stimulant ingestion?

Hemorrhagic stroke, coronary artery vasospasm (Prinzmetal's angina), MI, rhabdomyolysis (leading to hyperkalemia & acute tubular necrosis)

Bleeding in to the pleural space is known as?

Hemothorax

What communicable disease can a human bite transmit?

Hepatitis B

What is hemophilia?

Hereditary genetic disorder that impair the body's ability to control blood clotting

Restlessness, insomnia, v/d, abdominal pain are seen in the withdrawal of what? How is it managed?

Heroin Tx w/methadone, buprenorphine, naltrexone

How is mono diagnosed?

Heterophile antibody test

What causes MASSIVE UNDERLYING TISSUE TRAUMA ;carries high risk for complication such as COMPARTMENT SYNDROME & infection & requires IMMEDIATE SURGICAL INTERVENTION?

High Pressure Injection Injuries (grease gun; paint gun; hydraulics)

When is atropine ineffective?

High degree HB or Heart transplants Consider Isoproterenol for transplants

How is carbon monoxide poisoning managed?

High flow O2 via nonrebreather until COHb <10; HBO for pregnancy (fetus most vulnerable)

A kinked tube; bronchospasm; obstruction from secretions; intraabdominal compartment pressure can cause what alarm?

High pressure alarm

What is measles (rubeola)?

Highly contagious; INCUBATION PERIOD OF 8-12 DAYS S/s: conjunctivitis, Coryza (rhinitis); cough; fever; eyelid edema; KOPLIK SPOTS rash first (small specks on buccal mucosa); maculopapular rash from HEAD TO TRUNK to LE. Tx: supportive care; immunizations for patient's family/contacts

An elderly patient presents via EMS following a fall with c/o pain in hip/knee; medic verbalized noting a SHORTENING OF THE EXTREMITY & external rotation. What is the possible fracture & how is it managed?

Hip Minimize movement; surgical repair

What is a dislocation commonly seen w/ head on frontal impact MVCs where driver's leg was extended on the brake pedal?

Hip.

When is succinylcholine contraindicated?

History of malignant hyperthermia; penetrating eye injury; increased ICP; conditions that precipitate HYPERKALEMIA (burns, CRUSH INJURIES, renal failure); neuromuscular disorders (Guillain Barre, MS, myasthenia gravis)

What are the EARLY signs of hypoglycemia?

Hunger, cool, diaphoretic skin, TACHYCARDIA, & tremors

What meds are used to treat an adrenal crisis?

Hydrocortisone & dexamethasone

What is the antidote for cyanide?

Hydroxycobalamin

What medication impedes RBC sickling & stimulates the production of fetal hemoglobin & is used for the management of sickle cell disease?

Hydroxyurea

A transplant rejection that occurs IMMEDIATELY d/t incompatibility w/nothing that can prevent it from happening is called?

Hyper-acute

septic shock

Hyperdynamic phase f/b cold phase

What are the mechanisms that cause spinal cord injuries?

Hyperflexion ("whiplash"; hyperextension; axial loading/ compression (diving or landing on feet); rotation; penetrating injury

What electrolyte imbalance is seen with a patient in adrenal crisis & how is it managed?

Hyperkalemia Give IV insulin & dextrose

What can aluminum hydroxide can be given to patients with what electrolyte imbalance and why?

Hyperphosphatemia d/t aluminum hydroxide binds with phosphates in the GI tract decreasing their uptake.

BLOOD IN ANTERIOR CHAMBER from trauma that increases IOP is called?

Hyphema

Patient presents with c/o blood tinged vision; it is noted that there is blood in the anterior chamber. What is the cause & how is it managed?

Hyphema Shield affected eye; elevated HOB 30-45 degrees; administer Amicar (antifibrinolytic); educate on avoiding ASA & NSAIDS as well as chance of rebleed

Chvostek's & Trousseau's are seen in what?

Hypocalcemia

Hypoparathyroidism causes what electrolyte imbalance?

Hypocalcemia & hyperphosphatemia

What should be monitored for when giving a patient large amounts of blood products? How does it manifest?

Hypocalcemia. Monitor for tingling of the fingers, toes, nose, lips, & earlobes; hyperactive DTRs; abdominal pain; facial grimacing; muscle twitching/cramping.

What electrolyte imbalance causes the nervous system to be irritable which can causes to muscle irritability w/tremors & cramping?

Hypocalcemia; hyperphosphatemia; hypernatremia

Muscle weakness is associated with electrolyte imbalance?

Hypochloremia

What do you need to r/o with a stoke?

Hypoglycemia; migraines; Bell's palsy

Excess water in the body leads to what electrolyte imbalance?

Hyponatremia (it dilutes the serum sodium)

What lab results are seen with Adrenal Crisis?

Hyponatremia; HYPOGLYCEMIA; HYPERKALEMIA

What is a LATE SIGN of shock in a peds patient?

Hypotension

What is a LATE sign in a PEDIATRIC patient?

Hypotension

Fosphenyatoin is known to causes what s/e?

Hypotension & respiratory depression during administration & up to 20mins s/p administration.

Pediatrics Cardiovascular

Hypotension (30% blood loss)-late sign of shock. [70 + (2 x age)] for minimum systolic Bradycardia is OMNIOUS. Hypoxemia most common cause of peds arrest.

What is severe sepsis?

Hypotension (SBP<90; MAP <65) that RESPONDS TO FLUID resuscitation

What is septic shock?

Hypotension UNRESPONSIVE TO FLUID resuscitation that requires VASOPRESSORS

What is beck's triad?

Hypotension, distended neck veins & muffled heart sounds

What is being seen in anaphylactic shock?

Hypotension; decreased end-organ perfusion; multiple organ dysfunction

What are examples of secondary injuries from head trauma?

Hypotension; hypoxia; hypercarbia; cerebral edema

Patient's w/myxedema coma will usually be what?

Hypotensive; Bradycardic; hypoventilation or even respiratory arrest

What are the common s/s of myxedema coma?

Hypothermic w/o shivering; fatigue; lethargy; impaired mentation; seizures; coma; fluid retention; dry skin

0.45% NS is considered what type of fluid?

Hypotonic. Most likely to move from vascular bed to extra vascular bed. Should be administered in s/o tissue dehydration. (Poor skin turgor; sunken fontanels; dry skin)

What are common causes of respiratory acidosis? (The ABCs)

Hypoventilation {A}cute respiratory failure {B}rain injury {C}NS depression; Chest injury (e.g. rib fractures; flail chest) {S}edation

What DECREASES preload?

Hypovolemia & vasoDILATORS Tx: fluids/blood

What shock is d/t loss of circulating volume? How is it managed?

Hypovolemic (tank). Fill the pump & plug the leaks

What is the drive for respiration in a COPD patient?

Hypoxia

Lateral MI leads and artery

I, AVL, V5 & V6; circumflex

How is MRSA managed?

I/D; tx w/Mycins or tetracycline

What is intracerebral hemorrhage?

ICH. Rapid onset of HA & FOCAL DEFICITS Neurosurgical consult; manage ABCs & BP. Administer vitamin K, FFP, and/or TXA to stop bleeding

Inferior MI leads and artery

II, III, aVF; right coronary artery

What type of consent is involved in an unresponsive patient?

IMPLIED

What is a RED FLAG w/low back pain?

INCONTINENCE

What labs will you see with cirrhosis/liver failure?

INCREASED DIRECT BILIRUBIN (JAUNDICE), LFTS, PT, PTT, & AMMONIA (HEPATIC ENCEPHALOPATHY) Decreased urea, albumin, & calcium

What lab findings do you see w/rhabdomyolysis?

INCREASED POTASSIUM, CK, MYOGLOBIN, BUN, creatinine, phosphate, AST, ALT, uric acid

What are complications of PID?

INCREASED RISK OF ECTOPIC PREGNANCY & infertility

Hemophilia

INHERITED sex-linked disease. Prevents the formation of a stable fibrin clot (patient continues to bleed) S/s- bruising w/minor injuries; hemarthrosis of knee, ankle, elbow; epistaxis; subdural hematoma Tx- REPLACE DEFICIENT FACTOR or administer CRYOPRECIPITATE or FFP; DDAVP (stimulates factor VIII release); immobilize & elevated injured extremity; direct pressure s/p venipuncture. Head CT if head injury or altered LOC. ** Packed RBCs are not usually part of the treatment. (You don't need blood you need to clot!)

What is the therapeutic Coumadin level?

INR 2.0-3.0.

Uncal herniation presents with what?

IPSILATERAL PUPIL DILATION

What do you see w/Uncal/Transtentorial Herniation?

IPSILATERAL PUPIL DILATION early; bilateral fixed & dilated pupils Contralateral hemiparesis- abnormal posturing

If a patient presents with a purpuric nonblanching RASH what is the priority?

ISOLATION

What TB medication is contraindicated w/Dilantin?

ISONIAZID

What is the treatment of choice for "lost" items in the vagina?

IUD hook or ring forceps for IUD Ring forceps for Diaphragm

Endocarditis is commonly seen in who?

IV DRUG USERS; tattoo & piercings

What is treatment of pancreatitis?

IV access for FLUID RESUSCITATION; antiemetics; analgesics (opioids); IV CALCIUM GLUCONATE; H2 blockers & GLUCAGON (suppress pancreatic enzymes)

How is cholecystitis treated?

IV access; antiemetics; analgesics; NPO; gastric tube; ABX; possible surgery

What is the treatment for hypoglycemia in an UNCONSCIOUS patient?

IV dextrose

what is the treatment of gastroenteritis?

IVF & electrolyte replacement; antiemetics; ABX (bacterial); steroids (parasitic)

How is DKA treated in the pediatric population?

IVF at 10-20ml/kg boluses; repeat as needed Insulin 0.05-0.1 u/kg/hr (boluses lead to cerebral edema) until metabolic acidosis resolves Add 5% glucose when bgl <300 Replace K Measure bgl hourly

What is the treatment of appendicitis?

IVF; analgesics; antiemetics; prepare for possible surgery (NPO)

How do you treat pyloric stenosis?

IVFs & prep for surgery to dilate the pylorus

What is the treatment for ectopic pregnancy?

IVFs. RHOGAM IF RH - Prepare for surgery or METHOTREXATE IM (no signs of bleeding & must be compliant about f/u)

What is the treatment of hypercalcemia?

IVFs; furosemide; glucocorticoids (decrease GI absorption of Ca+); dialysis

How is a dissecting aortic aneurysm managed?

IVs x2; IVFs; LOWER HR TO 60-80 & BP TO 100-120 WITH A BETA BLOCKER

How is hemophilia managed in the ED?

Ice, compression, immobilization & elevation of joints TOPICAL THROMBIN FOR LACERATIONS & OBSERVE FOR 4HRS S/P SUTURING Hold pressure on venipunctures for at least 5 mins; no IM injections

When a specialty consultation ordered for an eyelid laceration?

If it's through the lacrimal gland

When is an MRI ordered for low back pain?

If neurovascular compromise suspected or incontinent.

What is the treatment of torsades?

If pulse palpable: cardioversion & magnesium sulfate (slow infusion) If pulseless: BLS, defibrillation, epinephrine, & magnesium 2 gm IVP

Unstable Tachycardia

If pulse present: Synchronized cardio version on R wave (ventricular contraction). Sedation prior to as appropriate. 50-200 J biphasic or 0.5-1 J/kg for peds. Pulseless: BLS & defibrillation (120-200j-adults or 2-4j/kg peds)

How is Vtach w/a pulse treated in pediatrics?

If s/o cardiopulmonary comprise (hypotension; AMS; s/o shock) perform IMMEDIATE synchronized cardioversion at 0.5-1 J/kg; may increase up to 2j/kg Adenosine (0.1mg/kg) is rhythm is regular & QRS monomorphic Expert consultation Amiodarone (5mg/kg over 20-60mins) or Procainamide (15mg/kg over 30-60mins)

How do you Dx & treat intussusception?

If stable: AIR OR BARIUM ENEMA (which also treats) May require surgery

For a lip laceration, when should a specialty consultation be considered?

If the laceration if through the VERMILLION BORDER

What are considered SYMPATHOMIMETICS/STIMULANTS? What are the risks of ingestion/OD?

Illegal street drugs (e.g. COCAINE; amphetamines; methamphetamine) Illicit DESIGNER/synthetic drugs (MDMA-ecstasy; Molly) OTC cold agents & dietary supplements containing ephedrine Prescription drugs (albuterol; Ritalin; Adderall; etc) R/o VENTRICULAR ARRTHYMIAS; MI; aortic dissection; RHABDOMYOLYSIS

What immediate management is required for ankle/malleus dislocation?

Immediate reduction

How is volvulus treated?

Immediate surgery

What happens in aplastic crisis & how is it managed?

Impaired RBC production worsens the anemia. Dx based on CBC & reticulocyte count Restore blood products

Neurogenic (central) DI

Impaired secretion of ADH. Seen w/head injury or brain surgery

What is the cause of the hypotension associated with a tension pneumothorax?

Impaired venous return to the heart

What is placentae previa?

Implantation of placentae over cervical os; hemorrhage may occur as uterus expands S/s: sudden, PAINLESS BRIGHT RED BLEEDING with s/o shock Tx: OB consult. TURN PATIENT ONTO LEFT SIDE 15-30 DEGREES. Treat shock. NO VAGINAL EXAM UNTIL U/S COMPLETED. Prepare for C-section.

what type of consent ALLOWS FOR APPROPRIATE TREATMENT IN AN EMERGENCY SITUATION?

Implied

What is Systolic HF?

Inability to PUMP effectively

What is Diastolic HF?

Inability to adequately FILL

What is phimosis?

Inability to fully retract foreskin over glans penis; may appear as a tight ring or "rubber band" of foreskin around tip of penis preventing full retraction. Tx: manual reduction; consider circumcision or dorsal slit

heart failure

Inadequate cardiac output & O2 delivery to tissues; BNP >100

What causes "THE BENDS"?

Inadequate decompression s/p exposure to increased pressure—>bubbles growing in tissues—>local damage BODY ABSORBS NITROGEN DURING ASCENT, if ascent too quick the nitrogen forms bubbles. Aka decompression sickness

Retained productions is noted with what type of abortion?

Incomplete

What is the management for electrical burns?

Increase IVF (d/t risk of RHABDOMYOLYSIS) EKG MONITORING FOR 24hrs.

What is Kussmaul Sign?

Increase in JVD in inspiration

How can you tell treatment has been effective for a patient w/rhabdomyolysis?

Increase in urine output and clearer urine

What discharge education is provided to patient to prevent recurring episodes?

Increase water & FIBER IN DIET; take stool softeners.

What are the s/s of SYMPATHETIC (ACUTE) pain?

Increased BP, HR,& RR; rapid speech; pupil dilation

What is polycythemia & what is an example of what can seen with a patient w/polycythemia and why?

Increased RBC count Respiratory crackles due to the thick viscous blood that may lead to heart failure

What is secondary polycythemia?

Increased RBCs in response to high altitudes & hypoxia Seen in COPD INCREASED BLOOD VISCOSITY Give ASA FOR CLOTTING RISK

What is one of the hallmark findings in Reye's syndrome?

Increased ammonia level

What are normal variances of pregnancy?

Increased risk of aspiration COMPENSATED RESPIRATORY ALKALOSIS (CO2 27-32) Increased circulating blood volume—>increased pulse & decreased BP (SUPINE VENA CAVA HYPOTENSION SYNDROME) Increased clotting factors (r/o HELLP)

How do you know that MANNITOL has been effective?

Increased urine output & improving LOC.

Normal fluctuations in the water-seal chamber does what?

Increased w/inspiration Decreased on exhalation

What are the s/s of malignant hyperthermia & how is it managed?

Increasing ETCO2; hyperthermia; muscle rigidity Dantrolene sodium for treatment.

What is diphtheria?

Incubation: 1-8 days S/s: sore throat; low grade fever; THICK, GRAY MEMBRANOUS (pseudo membrane) COVERING ON TONSILS & PHARYNX; orchitis Management: throat culture & gram stain; ERYTHROMYCIN STAT; diphtheria antitoxin (counteracts toxin produced by bacteria)

Emergency Thoracotomy

Indicated in traumatic cardiac arrest (e.g. penetrating injury)

Cushing triad

Indicative of increased ICP Widened pulse pressure or increased SBP, bradycardia, irregular breathing pattern

What is epididymitis?

Infection of the epididymis from chlamydia/gonorrhea Seen in sexually active males S/s: urinary frequency/urgency; urethral discharge (chlamydia); "DUCK WADDLE" gait Pain relieved w/elevation (PREHN'S SIGN) & ice Dx: U/S shows INCREASED blood flow Tx: ABX

What are the causes of gastroenteritis?

Infectious or food poisoning (raw or inadequately cooked food-2-6 s/p)

Epigastric pain, BRADYCARDIA, HYPOTENSION, second degree HB are s/s of what?

Inferior

ST segment elevation in leads II, III, AVF are indicative of what?

Inferior MI (RCA)

What is pancreatitis?

Inflammation & autodigestion of the pancreas

What is diverticulitis?

Inflammation of colon (sigmoid-large)

What is Endocarditis?

Inflammation of endocardium from cardiac sx; IV DRUG USE; body piercing that affects the cardiac valves S/s: infection; FEVER; chills; night sweats; myalgias; NEW ONSET MURMUR (d/t vegetation on valves); pleuritic CP; splinter hemorrhages in nail beds; conjunctival petechiae; Osler's nodes (tender nodules on digits); Janeway lesions (macules on palms & soles); Roth spots (retinal hemorrhages) Dx: increased WBC; blood cultures; increased ESR; ECHO Tx: IV ABX; admission; valvular repair

What is cholecystitis?

Inflammation of the gallbladder usually associated w/gallstones. Seen more in fair skin obese older females

What is Pericarditis?

Inflammation of the pericardial sac from infection; MI (dressler's); RF; etc S/s: sudden onset of RETROSTERNAL CP exacerbated by INSPIRATION, ACTIVITY, & SUPINE position; pericardial FRICTION RUB; tachycardia; tachypnea; low-grade fever Dx: CXR; ECHO; 12 lead EKG Tx: NSAIDS, ASA, steroids (NO NTG) PAIN Relieved by leaning forward/sitting up

What is peritonitis?

Inflammation of the peritoneum Typically from ruptured appendix; pancreatitis; penetrating trauma; PERITONEAL DIALYSIS

What is spontaneous bacterial peritonitis?

Inflammation of the peritoneum from ruptured appendix, pancreatitis, liver disease, penetrating trauma or peritoneal dialysis

What is prostatitis?

Inflammation of the prostate gland S/s: sudden onset of DYSURIA; FREQUENCY & URGENCY; hematospermia (blood in semen); tender prostate Dx: possible elevated PSA; u/a Tx: ABX (fluoroquinolones); increase fluid intake

What is orchitis?

Inflammation of the testes from STI or MUMPS Tx: NSAIDS & Cipro

What is pyelonephritis?

Inflammation or infection (bacterial) of the kidneys that can lead to urosepsis. S/s: severe COSTOVERTEBRAL PAIN (pain on either side of the vertebral column between the last rib & the lumbar vertebrae); fever; chills; N/V; urinary symptoms Dx: U/A for PYURIA (presence of pus in urine) & hematuria; BUN & Creat; renal US Tx:ABX; encourage fluids. ADMIT IF PREGNANT.

What causes lower GI bleeding?

Inflammatory bowel disease

What education needs to be provided to a COPD patient?

Influenza & pneumococcal VACCINATION; smoke cessation

What type of consent is obtained when a patient understands the risks & benefits of treatment; is not under the influence & has legal capacity to make the decision?

Informed

EMS brings in a patient who is noted to have CARBONACEOUS SPUTUM, burns to the neck & face, hoarseness, wheezing & dyspnea. What is the cause & how is it managed?

Inhalation injury 100% oxygen & EARLY INTUBATION before excessive edema occurs

How is acetaminophen toxicity managed?

Initially- activated CHARCOAL Antidote- ACETADOTE once the 4 hour level is obtained

Management of Vasopressor Extravasation

Initially: switch to another IV/IO/CVC. Leave cannula in & suck out as much as you can SQ & IV PHENTOLAMINE (regitine)-use 25g or smaller needle; administer SQ around extravasation site (placed 5mg/ml in 9ml of NS); administer 0.1-0.2mg/kg (to max of 10mg) through catheter site Topical NTG 4mm/kg of 2% ointment to affected area; repeat dose in 8hrs if needed SQ Terbutaline: digital- 0.5mg (1mg/ml) SQ into area; Extremity-1mg (1mg/10ml) SQ into area of extravasation. Can repeat dose in 15mins.

What is neurogenic shock?

Injury at T6 or above with LOSS OF SYMPATHETIC NERVOUS SYSTEM INNERVATION (unopposed parasympathetic)

What should not be done/given to a patient with a ruptured globe?

Instilling medications

When should anti venom CrobFab/Anavip be given?

Intense HEMORRHAGIC SWELLING d/t a venomous pit viper bite.

What are the s/s of gouty arthritis & how is it managed?

Intolerable pain in TOES; increased at night Tx w/colchicine; allopurinol; steroids; NSAIDS Educate on AVOIDING HIGH PURINE DIET (heart, herring, mussels, salmon, sardines, anchovies, veal, bacon, organ meats). Caution w/ASA; alcohol & thiazide diuretics. Increased risk of KIDNEY (URIC) STONES

What are the s/s of hyperemesis gravidarum & how is it managed?

Intractable N/V—>dehyration, electrolyte imbalances & malnutrition during pregnancy Antiemetics, IVF, electrolyte replacement

What presents with a SAUSAGE SHAPED MASS & CURRANT JELLY STOOL? How is it treated?

Intussusception. Air or contrast enema

What type of consent is obtained ensuring needed treatment when an individual refuses care? E.g suicidal; delusional; dementia

Involuntary

What is a lateral MI?

Involves LAD/Circumflex I & aVL high lateral, V5 & V6 Reciprocal changes: ST depression in II, III, & aVF (for high)

Crohn's Disease

Involves small bowel Can cause anal fissures Tx w/ANTICHOLINERGICS

Inflammation of iris, ciliary body, & choroid (middle portion of eye) d/t idiopathic, infection, trauma, rheumatic diagnosis, syphilis, lupus, etc is called?

Iritis (Uveitis)

Activated charcoal does not bind with iron?

Iron

Causes of microlytic anemia

Iron deficiency; thalassemia (inherited); lead poisoning

What is the determination of death?

Irreversible cessation of circulatory & respiratory function or brain death

How are Hydrofluoric acid (rust remover; glass etching) exposure managed?

Irrigate for at least 30 minutes (until pain relieved); apply w/2.5% CALCIUM GLUCONATE GEL until pain relieved; if ineffective administer 5-10% calcium gluconate via local infiltration. IF pain continues, administer 10ml of 10% calcium gluconate intra-arterially

For an ocular chemical burn what is the management?

Irrigate until pH 7.4; no patching

What is vulvovaginal candidiasis?

Irritation, ITCHING; COTTAGE CHEESE LIKE d/c Dx: wet prep for budding yeast Tx: nystatin vaginal creams or fluconazole (diflucan)- prolongs that QT- PO once (contraindicated in pregnancy)

What is Prinzmetal's angina?

Ischemia d/t coronary VASOSPASM/stimulants Cyclical pain at REST Vasospasm precipitated by stress ST elevation & pain resolves when vasospasm resolves. Beta blockers may exacerbate vasospasm d/t unopposed alpha stimulation

What is a shunt?

It diverts CSF from the lateral ventricle (hydrocephalus) to low pressure space (like peritoneum-VP shunt; or atrium)

What is Pericardial/Cardiac Tamponade?

It is a type of OBSTRUCTIVE SHOCK in which the pericardial sac accumulates additional fluid (usually 25-50ml); usually from pericarditis; trauma; MI S/s: BECK'S TRIAD; MUFFLED heart sounds; Kussmaul's sign (increased JVD on inspiration); pulsus paradoxus (decrease in BP upon inspiration); small EKG amplitude; Electrical alternans (alternating amplitude of QRS); risk of PEA Tx: prepare for PERICARDIOCENTESIS or sx; support BP w/vasopressors

What is AVPU?

It is the LOC assessment {A}lert {V}erbal {P}ain {U}nresponsive

Why is xylocaine w/epinephrine not used on ears, nose, fingers, toes & penis??

It would cause a DECREASE IN CIRCULATION TO THOSE DISTAL AREAS—>increase in r/o infection

What precautions need to be taken w/methotrexate?

It's CYTOTOXIC so DON PPE INCLUDING CHEMOTHERAPY GLOVES FOR HANDLING

What is spinal shock?

It's not Neurogenic shock so.... It's an injury AT ANY LEVEL (remember neurogenic is T6 of ^); loss of motion/sensation below level of injury; FLACCID PARALYSIS

What is the benefit to FIXED WING (airplane) transport?

It's pressurized & can fly in inclement weather.

Why is a class V fracture the worst (according to the salter Harris classification)?

It's through the growth plate

Treating all patients fairly (even those addicts & alcoholics) is what?

JUSTICE

What is Beck's Triad?

JVD; hypotension; & muffled heart sounds S/s of cardiac tamponade

How should the airway be opened in a trauma patient?

Jaw-thrust

What has stinging darts that fire (nematocysts) producing severe pain & reddened welts? And how is it managed?

Jelly fish RINSE W/NORMAL SALINE (WATER STIMULATES VENOM); vinegar, or apply paste of baking soda w/saline; REMOVE TENTACLES USING FORCEPS. Administer topical antihistamines or corticosteroids

Collection of fluid in a joint space (KNEE most common) is caused?

Joint effusion

What is the frontal lobe responsible for?

Judgement; "mother"; affect; coordinating voluntary motor movements Speech center on Left side for most people Broca area responsible for PRODUCTION of speech Wernicke area for COMPREHENSION of spoken word

Providing equal care to the impaired driver as you would others is an example of?

Justice

What education is needed for a patient with migraines?

KEEP A JOURNAL TO FIND TRIGGERS. If Depakote is used on a female patient they need to be on BC (d/t risk of birth defects)

What is a common cause of patella/knee fracture?

KNEE INTO DASHBOARD

What is the tx of COPD?

Keep SpO2 88-92% via NC or Venturi mask (REDUCE O2 if decreased RR; hypoxia is required for respiratory drive); NIV; bronchodilators; sit on edge of bed w/feet dangling Educate on PNA vaccination; avoid exposure to those respiratory infections; smoking cessations

What is the treatment for an Esophageal Obstruction?

Keep patient upright. GLUCAGON IV (will make vomit!); NTG SL. Esophagoscopy for removal

Inflammation of cornea d/t EXPOSURE TO UV LIGHT (e.g. snow blindness; glare off water; welding) is called..?

Keratitis

What is induction medication of choice for asthma patients?

Ketamine because it is also a bronchodilator. **also with intubating asthmatics: INCREASE inspiration:expiration ratio from 1:2 to 1:3-4, no or little PEEP

Patient presents with hematuria, flank tenderness & ecchymosis; it is noted that they have rising BUN & creat levels. What abdominal organ has been impacted?

Kidneys

Knee pain & inability to bend or straighten knee (depending on current position) are s/s of what?

Knee fracture

During DKA what is happening to compensate for METABOLIC ACIDOSIS?

Kussmaul's respirations

ulcerative colitis

L. Sided abdomen pain (Large Intestine) BLOODY diarrhea

diabetes inspidus (DI)

LACK of ADH->polyuria. Pt is HYPOVOLEMIC, hypotensive & HYPERNATREMIC. S/s: Increased DILUTED u/o & polydipsia Labs: HYPERnatremia & INCREASED serum osmolality (DESPITE u/o); LOW urine specific gravity (1.001-1.005; d/t urine dilution) & osmolality Tx: DESMOPRESSIN (ADH) & volume replacement. Aqueous VASOPRESSIN. Monitor for s/o cerebral edema. Better if DECREASED u/o Risk for HYPOVOLEMIC shock. Seen in head injuries; lithium & Dilantin use.

What is one of the hallmark symptoms of a CLUSTER HA?

LACRIMATION (excessive tearing)

What is the treatment of choice for cirrhosis/liver failure?

LACTULOSE or PEG (MIRALAX) (remove that ammonia)-give that Lactulose immediately; neomycin (decrease production of ammonia); replace albumin, calcium, potassium & vitamin K Monitor for esophageal varices (d/t PORTAL HTN)

What do you see w/Left sided HF?

LEFT is LUNGS SOB; paroxysmal nocturnal dyspnea; dyspnea; crackles; S3 heart sounds; pulmonary edema Occurs w/Anterior MI

Being struck by lightening can cause...?

LICHTENBERG FEATHERING RUPTURED TM

What are the s/s of ulcerative colitis & how is it managed?

LLQ crampy abdominal pain; RECTAL BLEEDING; diarrhea; tenesmus (urge to poo); affects only the colon Corticosteroids & antiinflammatories; surgical removal

Diverticulitis

LLQ pain (large intestine) Alternating diarrhea & constipation Tx is bowel rest

What are the s/s of a concussion?

LOC under 30mins; GCS of 13-15; confusion; disorientation; HA; fatigue; poor concentration; post-traumatic amnesia; photophobia; phonophobia

How is COPD managed?

LOW FLOW O2 via NC or Venturi mask; nebulized bronchodilators; steroids

Due to the increased urinary output (polyuria), patient's with DI exhibit what lab findings?

LOW urine specific gravity & osmolality HIGH serum osmolality & SODIUM.

Distorted reality, tachycardia, HTN, dilated pupils, hyperthermia, "FLASHBACKS" (long term use) are seen in what type of ingestion?

LSD

splenic sequestration

LUQ abd pain; splenomegaly; hypovolemic shock

What is one of signs of a spleen injury?

LUQ pain radiating to L. Shoulder (KEHR'S)

What is happening w/cardiogenic shock?

LVF. Inadequate tissue perfusion because of PUMP FAILURE Most commonly from anterior wall MI S/s: tachypnea; CRACKLES; pulmonary edema; S3 heart sound; cool clammy skin; hypotension Tx: airway management w/PEEP; DECREASE PRELOAD w/NTG & diuretics; DECREASE AFTERLOAD w/NTG or Nitroprusside; INCREASE CONTRACTILITY w/ POSITIVE INTOTROPES (like Dobutamine) & IABP & treat dysrhythmias

What is the inflammation of the inner ear (labyrinth) d/t recent infective process (fluid)

Labyrinthitis

What are presentations that are associated w/renal trauma?

Lacerations; contusions; vascular injury associated w/posterior rib fractures Flank/back pain & ecchymosis (GREY-TURNER'S sign) w/HEMATURIA

Patient presents with c/o gradual onset of crampy abd pain; constipation; obsipation & MARKED DISTENTION. What is going on?

Large Bowel obstruction

Patient presents w/MARKED DISTENTION; rare vomiting; crampy pain; verbalizes a gradual onset. What kind of obstruction is this?

Large bowel.

How is rhabdomyolysis managed? And what is potential complication?

Large volumes of IVF (6-12L) to produce urine output of at least 200ml/hr; urine alkalization; diuretics; dialysis; until urine clear Complication of acute tubular necrosis

St elevation in leads I, AVL, V5 & V6 are indicative of what?

Lateral MI (circumflex)

"A man with a MUSTACHE" describes what Le Fort fracture?

Le Fort I

A transverse maxillary bone fracture that separates the teeth from the maxilla is known as ?

Le Fort I

Transverse detachment of the entire MAXILLA ABOVE THE TEETH at the level of nasal floor; FREE-FLOATING MAXILLA; describes what type of fracture?

Le Fort I

"Goes to the PYRAMIDS" describes what Le Fort fracture?

Le Fort II

A Pyramidal fracture across the mid-face traversing the bridge of the nose is known as?

Le Fort II

PYRAMIDAL SHAPED FRACTURE; w/transverse detachment of maxilla (base of pyramid); fracture at bridge of nose (top of pyramid); fracture through lacrimal & ethmoid bones (sides of pyramid) describes what fracture?

Le Fort II

Complete craniofacial separation fracture is known as?

Le Fort III

Free-floating segment of mid face; CRANIOFACIAL DISJUNCTION involving maxilla, zygomatic arch, orbits, & cranial base bones describes what fracture?

Le Fort III

"And takes off his Halloween MASK" describes what le fort fracture?

Le fort III

What is an INFERIOR MI?

Leads II, III, aVF Commonly affects the RCA. S/s: bradycardia (leading to HYPOTENSION) ; heart block (2nd degree type 1); epigastric pain; N/V; r/o mitral regurgitation & papillary muscle rupture

What is an ANTERIOR MI?

Leads VI-V4 Commonly the LAD "widow maker" S/s: crushing CP; ventricular dysrhythmias; tachycardia; feeling of impending doom; BBB; SOB; crackles in lungs; S3 (d/t LVF & CARDIOGENIC SHOCK); 2nd degree type 2 HB (ominous sign)

What are reciprocal leads?

Leads that OPPOSE (opposite views) contiguous leads I & aVL in inferior II, III, & aVL in High Lateral

What education do you provide a patient who has received adhesive tape strips (steri-strips)?

Leave strips until they fall off on their own (7-10 DAYS) Avoid petroleum ointments bc it weakens glue

Nocturnal dyspnea & pulmonary edema that is managed with DIURETICS, NTG, & NIPPV is what?

Left (LUNGS) HF

What position should a patient be placed in with an air embolism following a rapid ASCENT while scuba diving?

Left Lateral Decubitus in Trendelenburg

Left sided HF

Left=Lungs LV fails->blood "stacks up" in lungs. E.g. cause- Anterior MI S3 heart sound; SOB; dyspnea (nocturnal) ; CRACKLES d/t pulmonary edema. Support ABCs, O2; NIPPV; diuretics; vasodilators; ACE-I

A narcotic OD with decreased respirations; hypoglycemic patient who is unresponsive; severe respiratory distress are all examples of what ESI?

Level 1

What is epiglottitis?

Life threatening bacterial (HIB) respiratory infection S/s: drooling; dysphagia; distress; tri pod position; high fever THUMBPRINT sign on lateral neck xray Allow position for comfort; consult peds anesthesiologist

What Neuroleptic Malignant Syndrome?

Life threatening conditioning caused by antipsychotic medications (e.g. HALDOL; thorazine) S/s: HYPERTHERMIA; MUSCLE RIGIDITY; AUTONOMIC INSTABILITY (fluctuations in BP) Tx: ABCs; REDUCE TEMPERATURE w/COOLING BLANKETS & ice packs

What is Bipolar Einthoven's Triangle?

Limb lead I, II, III

What is SAD LITH?

Lithium toxicity {S}eizures {A}taxia-impaired balance {D}ystonia-muscle contractions {L}ethary/{L}eukocytosis {I}nsipidus-DI {T}remors {H}ypothyroidism

Patient presents w/ RUQ pain; muscle rigidity; umbilical ecchymosis (Cullen's sign) & guarding what abdominal organ has been impacted? And how is it managed?

Liver Monitor LFTs, coagulation studies; & h/h

RUQ pain Periumbilical ecchymosis (Cullen's Sign) Rigid Abdomen Rebound tenderness Right 9-12 Rib Fractures Are all indications of what abdominal organ injury?

Liver PREPARE FOR MASSIVE TRANSFUSION PROTOCOL & damage control surgery

What is characteristic for Diabetes Insipidus?

Lose volume but are HYPERNATREMIC.

What is Alzheimer's Disease?

Loss of Cognitive ability over time

what is a complete injury?

Loss of all motor/sensory function & reflexes below the level of injury Loss of bowel/bladder function; PRIAPRISM; Poikilothermia; loss of PROPRIOCEPTION Spinal or NEUROGENIC SHOCK

What are the s/s of DESCENDING aortic dissection?

Loss of distal pulses; LE weakness; decreased U/o

What happens w/a central cord injury?

Loss of motor & sensory function More pronounced in arm than legs ** THEY CAN WALK TO THE BAR BUT CAN'T DRINK THE BEER THEY ORDERED**

What happens with an ANTERIOR CORD INJURY?

Loss of motor function; pain, crude touch & temperature. RETAINS PROPIOCEPTION

What happened w/a Posterior cord injury?

Loss of proprioception, vibration, fine touch & fine pressure Intact motor function.

Retrobulbar hematoma can cause what?

Loss of vision d/t abrupt increase in IOP—>orbital compartment syndrome

LP in HELLP stands for?

Low Platelets (DIC)

What is neutropenia?

Low WBC count->increasing risk of infection.

What are the s/s of neutropenia & how do we manage it in the ED?

Low grade to absent fever w/LOW NEUTROPHIL COUNT; recurrent infections Tx: PROTECTIVE (REVERSE) ISOLATION; avoid invasive procedures; EARLY ABX; avoid raw/undercooked meat, well water, & unwashed produce; bone marrow stimulants (neupogen)

What is anemia?

Low hgb from blood loss, low iron, low vitamin b12, or low folic acid Tx: stop blood loss, PO IRON REPLACEMENT (educate on dark stools & constipation); SQ vitamin B12

What are the risks for a sickle cell attack?

Low oxygen saturation; infection; DEHYDRATION; EXPOSURE TO COLD

What is idiopathic thrombocytopenia purpura?

Low platelet count with normal bone marrow function; d/t autoimmune disorder seen after viral infection (PEDS 2-4YRS OLD-SPONTANEOUSLY RESOLVES) or chronic in adults S/s: indications of bleeding-bruising; PETECHIAE; PURPURA; epistaxis; bleeding gums; prolonged bleeding from minor injuries; brain hemorrhage

A cuff leak or disconnection can cause what alarm?

Low pressure alarm

Lab findings for myxedema coma include?

Low thyroid levels; hypoglycemia; hyponatremia; respiratory/metabolic acidosis

Patient presents w/HEMATOCHEZIA, crampy abd pain, & s/o shock. What is going on? How is it managed?

Lower GI bleed Fluid resuscitation; possible PRBCS; colonoscopy & CBC

What are the s/s of PID? How is it managed?

Lower abd pain/tenderness; cervical motion tenderness; adnexal tenderness; mucopurulent cervical discharge; fever; positive culture for N. Gonorrhea or C. Trachomat; WBC >10k; elevated CRP or ESR Cefriaxone 250mg IM; doxycycline 100mg BID x 14 days

When is WHEEZING seen?

Lower airway bronchial obstruction- asthma & bronchiolitis

What is pulmonary contusion?

Lung injury & edema from chest trauma or barotrauma S/s: respiratory trauma; CHEST WALL BRUISING; crackles; wheezes Dx: CXR may not reveal infiltrates until 12 HOURS LATER Tx: O2 is PRIORITY; NIV; ventilation; JUDICIOUS USE OF FLUIDS TO PREVENT ARDS

What causes a non-pruritis target like, circular BULLS EYE RASH & flu-like symptoms? How is it treated?

Lyme Disease. Tx w/ABX (doxycycline)

What is volvulus?

MALROTATION. Bowel rotation resulting in strangulation. Typically occurs in 1st month of life. S/s: BILIOUS VOMITING w/abdominal distention, bloody stools; visible peristaltic waves PREPARE FOR SURGERY IMMEDIATELY.

What are interventions during cardiac arrest that are only seen during maternal arrest?

MANUAL DISPLACEMENT OF THE UTERUS Caution w/vasopressors.

How do you determine cerebral blood flow?

MAP-ICP = CPP MAINTAIN > 60 IN HEAD INJURY **normal in absence of injury 70-90

What increases the risk of a dissecting aortic aneurysm?

MARFAN'S & Ehrler Danlos

What education needs to be given upon discharge for a patient w/TB?

MEDICATION COMPLIANCE; containment of respiratory secretions (zip lock bags); avoidance of close contact w/others until medically cleared; RIFAMPIN STAINS BODY FLUIDS BRIGHT ORANGE (NO CONTACTS)

Female patient presents with recurrent episodes of vertigo, visual disturbances & tinnitus. What is the potential cause & how is it managed?

MENIERE'S DISEASE MECLIZINE & antihistamines.

Your patient presents with a NON BLANCHING PETECHIAL RASH ON TORSO & LEGS. What do you think they have?

MENINGOCOCCEMIA.

If a patient is leaving AMA, what should be assessed & documented prior to leaving?

MENTAL COMPETENCE.

If the pH & HCO3 are both increased or decreased what is it?

METABOLIC

Salicylates increase ATP production which leads to what?

METABOLIC ACIDOSIS

TRACTION SPLINT is used for what & why?

MID SHAFT FEMUR FRACTURE to decrease bleeding, muscle spasms & relieve pressure on nerves.

What is the concern for a patient who reports being hit in the temporal brain?

MIDDLE MENINGEAL ARTERIAL BLEED (EPIDURAL HEMATOMA)

Body temp <95F (35.3C) w/shivering & dizziness is considered? And how is it managed?

MILD HYPOTHERMIA Passive external (e.g. warmed blankets)

In what phase does the assessment of HAZARDOUS VULNERABILITY occur?

MITIGATION

Body temp 33-35 C w/slurred speech & disorientation is considered? How is it managed?

MODERATE HYPOTHERMIA Active external w/warming devices

What is the order for suctioning neonate w/bulb syringe?

MOUTH THEN NOSE

If SCIWORA is suspected in a child what diagnostic test is used?

MRI

How is an Achilles Tendon Rupture diagnosed & tx'd?

MRI; Surgery.

What are examples of primary injuries for head trauma?

MVC, fall, sports, assaults

What serum levels should be checked prior to administering calcium gluconate for a patient w/hypocalcemia? Why?

Magnesium. Hypomagnesemia responds poorly to administration of calcium gluconate

How is a pulmonary contusion managed?

Maintain SpO2 94-98%; keep patient in semi fowler's; administer IVF JUDICIOUSLY

How are anxiety & panic attacks treated?

Maintain calm appearance; use therapeutic communication; PROVIDE SUPPORTIVE (NONJUDGEMENTAL) ENVIRONMENT & SHOW ACCEPTANCE; encourage SLOW BREATHING; reassure patient they are "not crazy"; administer benzodiazepines ('pams) as ordered.

What are the S/s of Hepatitis?

Malaise; N/V/A (early); jaundice; clary-colored stool; steatorrhea; DARK COLORED FOAMY URINE (LATER)

What are the s/s of rhabdomyolysis?

Malaise; fever; MYALGIA; DARK BROWN URINE; INCREASED K+, MYOGLOBIN, & CK Complication: ACUTE TUBULAR NECROSIS (ARF)

Most urethral injuries are seen in who?

Males from a SADDLE INJURY

in a multi-patient trauma resuscitation; what types of patient can receive O positive blood?

Males of any age

Tears of the distal esophagus at the gastroesophageal junction from SEVERE RETCHING (bulemia) or heavy lifting is known as?

Mallory-Weiss syndrome

What are the s/s of a Le-Fort I?

Malocclusion, LIP LACERATION; fractured teeth; swelling to area

What are the s/s of a mandibular fracture (Broken jaw)?

Malocclusion, trismus, edema & ecchymosis of the lower face; paresthesia of lower lip & chin Facial CT or panoramic xrays; elevate HOB; suction frequency; ice; prepare for airway & sx

What are the s/s of a TMJ dislocation & how is it managed?

Malocclusion; pain w/jaw movement; POP, CLICK, SNAP SENSATION; limited ROM Tx: analgesia & muscle relaxants (relax that jaw) then MANUAL REDUCTION. Educate patient on soft diet x4 days; avoidance of triggers & teeth clenching, grinding, yawning; using a mouth guard at night to avoid grinding.

What are s/s of a mandibular fracture and how is it treated?

Malocclusion; trismus; edema; ecchymosis; NUMBNESS OF LOWER LIP; pain SECURE AIRWAY (d/t LOSS OF TONGUE CONTROL); elevate HOB; suction frequently; ice; surgery; analgesics; ABX

volvulus

Malrotation Requires immediate surgery

What are s/s of gonorrhea & how is it managed?

Many asymptomatic Urinary symptoms; mucoid discharge Infection may spread & involve joints- fever; chills; rash CEFTRIAXONE IM

A spontaneous pneumothorax is commonly seen in who?

Marfan's Tall male age 20-40 Smokers COPD

Hyperosmolar Hyperglycemic Syndrome (HHS)

Marked elevation of blood glucose & hyperosmolarity. SEVERE DEHYDRATION. Seen in DM II. Symptoms develop over days-weeks S/S: polyuria, polydipsia, HA, tachycardia, SHALLOW breaths, hypotension, & confusion that progresses to unresponsiveness. *NO FRUITY SMELL* Labs: BGL >600, serum osmolality >320, elevated BUN d/t DEHYDRATION. Minimal-no KETONES. *Nml pH, no acidosis* Tx: 0.9NS-monitor for fluid overload. Less insulin required than in DKA. MORE FLUID LESS INSULIN. R/o renal failure & DVT d/t dehydration.

What is used for a patient with rhabdomyolysis to prevent ACUTE TUBULAR NECROSIS?

Massive IVFs & urine alkalization.

What is a complication of otitis media that causes erodes the mastoid & affects surrounding structures?

Mastoiditis

Sinusitis in what sinus causes pain below the eyes, over the cheekbones, upper teeth/jaw with increased pain of chewing?

Maxillary

Conjunctivitis, coryza, cough & Koplik spots are seen with?

Measles

What communicable disease has an incubation period of 8-12 days & presents w/Conjunctivitis, Coryza, Cough, fever & Koplik spots?

Measles

What communicable infection requires standard & airborne isolation?

Measles

How do you insert a gastric tube?

Measure tip of nose to earlobe to xyphoid process. Benzocaine, tetracaine, or lidocaine may be sprayed x2 in nose/mouth Place patient in HIGH FOWLERS (IF ALERT); flex head forward; give sips of H2O If patient is ALTERED; place on LEFT SIDE (decreases risk of aspiration)

What is a Hamman's sign?

Mediastinal CRUNCH heard upon auscultation

Dehydration may exacerbate vertigo associated w/?

Meniere's disease

What causes the acute inflammation of the lining of the brain & spinal cord?

Meningitis

What is psychosis?

Mental disorder characterized by BIZARRE THINKING; often accompanied by sensory HALLUCINATIONS (illusions) & delusions (misconceptions of beliefs) Dx: r/o brain tumors w/HEAD CT; use of psychoactive drugs (LSD); sepsis; etc Tx: HALDOL

What blood gas abnormalities could be seen with myxedema coma?

Metabolic & respiratory acidosis

What are some causes of hypokalemia?

Metabolic alkalosis; overuse of diuretics; acute alcoholism; cirrhosis; INTESTINAL TRACT DISEASES (MALABSORPTION)

What can happen to a welder if not treated immediately?

Metal may leave a RUST RING if not removed immediately

What is the antidote for methemoglobin?

Methylene blue

Where do you expect to be fractured if your patient presents w/ "BATTLE'S SIGN" (MASTOID ECCHYMOSIS) & OTORRHEA

Middle Fossa fracture

What is a concussion?

Mild TBI. "Dimming of the consciousness" Dazed. GCS 13-15 w/HA; dizziness; RETROGRADE AMNESIA; vomiting; answers questions slowly

What are the degrees of injuries?

Mild- GCS 13-15 w/LOC <30; no deficits Moderate GCS 9-12 w/LOC & focal deficits Severe GCS of 8 or < w/significant LOC; abnormal pupils & posturing.

What is the treatment of pediatric V/D?

Mild: small amounts (3-5ml)SIPS of ORAL REHYDRATION Frequenty Severe: 20ml/kg of ISOTONIC CRYSTALLOIDS; repeat if necessary; zofran;

The main treatment goals with diffuse axonal injuries are what?

Minimize or reduce ICP (which in turn could improve LOC)

What type of ophthalmic medication should be given to a patient w/glaucoma?

Miotic eye drops (constriction of pupil may enhance drainage from anterior chamber of eye & help relieve symptoms

What are the phases of Emergency Management?

Mitigation Preparedness Response Recovery

What phase of emergency management involves preventing/minimizing potentially adverse affects?

Mitigation.

How do you give Dilantin?

Mix in 0.9% NS only & infuse NO FASTER THAN 50MG/MIN; monitor closely for INFILTRATION. Monitor cardiac/BP/RR monitor DURING INFUSION & 20 MINS S/P

What is heliox?

Mixture of oxygen & helium that aids in ventilation by reducing resistance of airflow

2nd degree HB type 2

Mobitz II. Fixed PR interval. Dropped QRS complex

What is caused by the Epstein-Barr virus & spread by body fluids (especially saliva)?

Mononucleosis

What are s/s of chlamydia & how is it managed?

Most asymptomatic Dysuria, mild abdominal pain; THIN MUCOPURULENT DISCHARGE AZITHROMYCIN; doxycycline or erythromycin

What is a spontaneous pneumothorax?

Most common in young, thin, tall males or smokers S/s: dyspnea; decreased or absent BS on affected side; pleuritic CP; HYPERRESONANCE ON PERCUSSION Tx: High Fowler's; supplemental O2; potential chest tube placement at 5th-6th ICS at mid-axillary line

Abducting (fanning) fingers is assess what nerve?

Motor assessment of the Ulnar nerve; assessing sensation to 4th & 5th fingers

Opposing thumb to fingers is assessing what nerve?

Motor assessment of the median nerve; assessing sensation of the index finger

Vertigo associated w/labyrinthitis can be exacerbated by?

Movement

What is a beck's triad?

Muffled heart tones; JVD; hypotension

Swollen salivary & PAROTID GLANDS are seen in what?

Mumps

What causes salivary & parotid gland swelling?

Mumps

What is a complication of Mumps in a male patient post puberty?

Mumps

What communicable infections require standard & droplet isolation?

Mumps; pertussis; diphtheria

The inability to take deep breaths during palpation of the R. Costal arch below the hepatic margin is known as? And what is it seen in?

Murphy's Sign Cholecystitis

What are s/s of hypokalemia?

Muscle weakness, n/v; PARALYTIC ILEUS; abdominal distention/gas; shallow respirations; mental depression; leg cramps; tachycardia; FLAT T WAVES; possible U waves; ventricular irritabilty

Edrophonium is used as a treatment for what?

Myasthenia Gravis. It should improve muscle strength

What eye drops dilate the eye?

Mydratic & cycloplegic

What is one of the complications of diphtheria?

Myocarditis

Dangerously low thyroid levels lead to what?

Myxedema Coma

What commonly affects WOMEN 40-60 YEARS of age w/RECURRING EPISODES OF NYSTAGMUS, VERTIGO, TINNITUS, hearing loss, & n/v?

Ménière's disease

What is the antidote for APAP?

N-acetylcysteine

What are S/S of lithium toxicity?

N/V/D, Parkinson like movements; ataxia; possible diabetes Insipidus

What are the S/S of HBV? And how is it prevented?

N/V/D/A; RUQ pain; pruritis from bile salts; body aches; dark urine (bile); clay-colored stool (d/t lack of bile); JAUNDICE Prevent w/vaccine

What are s/s of gastroenteritis and how is it managed?

N/V/D; abd cramping; s/o dehydration Check CBC, CMP, & stool sample IVF & ELECTROLYTE REPLACEMENT; antiemetics; ABX if bacterial cause

What are the s/s of Iron OD & how is it managed?

N/V; abdominal pain (early); HEMATEMESIS Tx: Desferal (DEFEROXAMINE) which is a CHELATING AGENT that binds free iron; excreted renally (rust, pink or "VIN ROSE" URINE COLOR EXPECTED)

What are the s/s of a Le Fort II?

NASAL FRACTURE; epistaxis; malocclusion; LENGTHENING OF FACE

What is the priority of patient who presents with TB?

NEGATIVE ISOLATION ROOM

Profound BRADYCARDIA; hypotension; skin warm & dry below the level of injury; PRIAPSIM; poikilothermy is seen with what?

NEUROGENIC SHOCK

What is risk of using HALDOL?

NEUROLEPTIC MALIGNANT SYNDROME

What criteria is used for spinal immobilization?

NEXUS (intoxicated; focal deficit; distracting injury)

How is noncardiogenic pulmonary edema treated?

NIPPV or mechanical ventilation w/low tidal volumes (lung protective) JUDICIOUS FLUID ADMINISTRATION

IV Dextrose in Peds

NO higher than D25% in children or D12.5% in infants

pyloric stenosis

NON BILIOUS PROJECTILE vomiting & continual hunger. Olive shaped mass. Requires dilation of pylorus.

What is autonomic dysreflexia?

NOXIOUS STIMULUS—>MASSIVE SNS (ADRENALINE) RESPONSE—>severe HTN, pounding HA; nasal congestion; flushed face; sweating w/piloerection IDENTIFY & TX CAUSE (over-distended bladder; bowel impaction; skin pressure; infection; lower BP)

What are the s/s of labyrinthitis & how is it managed?

NYSTAGMUS; VERTIGO; TINNITUS; pain in ear (otalgia); n/v; hearing loss Tx: CORTICOSTEROIDS; MECLIZINE (MOTION SICKNESS); anithistamines

How is a subungual hematoma managed?

Nail trephination (hole drilled/incised/scraped) by penetrating fingernail w/nail drill, scalpel, or cautery

What is the antidote for Opioids?

Narcan

What is pyloric stenosis?

Narrowing of the pylorus preventing emptying of the stomach causing NON-BILIOUS PROJECTILE VOMITING (because it hasn't reached that gallbladder) & continuous hunger; poor weight gain; "olive shaped mass"; s/o dehydration Usually in the 1st 3 months of life.

What is croup & how is it managed?

Narrowing of the upper airway; 1-3 day h/o low grade fever Most common in 6mos-3yrs STEEPLE sign on CXR Nebulized RACEMIC EPINEPHRINE, dexamethasone; Heliox

What education are you going to provide your patient re: Chlamydia?

Need to have protected sex or no sex x7 days

Sarin or VX gas are examples of ?

Nerve agents

what are complications of a femur fracture?

Nerve damage (hip fx-avascular necrosis); compartment syndrome; DVT; wounds from immobility; fat embolism; pulmonary embolism

What is seen in a SCI above T6 that causes a disruption of sympathetic regulation leaving unopposed parasympathetic stimulation?

Neurogenic shock

At what position should the extremity be kept in for compartment syndrome?

Neutral position (I.e. AT LEVEL OF THE HEART)

How is preeclampsia managed?

Nifedipine; labetalol; hydralazine until BP <160/110 Turn patient on Left side MAGNESIUM SULFATE 4-6g IV loading dose over 15mins

What are examples of Vasodilators?

Nitroglycerin & Nitroprusside

Management of STEMI

Nitroglycerin SL tab/spray initially. IV infusion PRN. Contraindicated in SBP <90 or HR <50, recent phosphodiesterase inhibitors ASA 160-324mg chewable. Clopidogrel for antiplatelet effects PCI (goal is <90mins) Fibrinolytic therapy (if PCI unavailable w/in 90-120mins). Expect REPERFUSION DYSRHYTHMIAS (accelerated idioventricular or VT)

What is the treatment for hypoglycemia in an UNCONSCIOUS child?

No HIGHER THAN 25% DEXTROSE (HALF STRENGTH D50%- 25CC OF EACH); consider D5W boluses as alternative to D50%

What education do you need to provide your patient re: trichomonas?

No alcohol consumption while taking flagyl. Sexual partners need to be treated.

What education do you need to provide for a patient re: vaginal discharge?

No alcohol intake while on Flagyl. Sexual partners need to be treated. Avoid douching & bubble baths (BV) Clindamycin cream weakens condoms (BV)

What is the treatment for hypoglycemia in a UNCONSCIOUS INFANT?

No higher than 12.5% infants; consider D5W boluses as alternative to D50%

The accumulation of fluid in the lungs w/an increase in permeability that causes tachypnea, SOB, anxiety, sensation of suffocation, pink frothy sputum, crackles & moist skin usually d/t ARDS, HAPE, submersion injury, inhalation of toxic gases (e.g. chlorine, anhydrous ammonia), heroin OD is known as?

Noncardiogenic pulmonary edema

What is the duty to help others? (E.g. report discharge of unstable patient)

Nonmalficence

Failing to provide car to the legally impaired driver which could cause further harm is an example of ?

Nonmalificence

Hematemesis , abdominal pain, melena, & s/o shock are seen with? How is it managed?

Nonvariceal Upper GI bleed CBC; txm; CMP; liver function; coags Hemodynamic resuscitation; NG tube to lavage & clear GI tract for endoscopy

How is asthma managed?

O2 SABAs (albuterol; Levalbuterol) nebulized q20mins x3 Anticholinergics (ipratropium bromide) PO steroids (reduce that inflammation) Fluids IV magnesium (inhibit bronchial muscle contraction) Heliox Ketamine

Cardiac tamponade; tension pneumothorax; abdominal compartment syndrome can cause what?

OBSTRUCTIVE SHOCK

What are examples of Cholinergics?

ORGANOPHOSPHATE (OP) PESTICIDES; chemical warfare agents (sarin)

A cholinergic toxicity is seen with?

ORGANOPHOSPHATES

Following intubation where is the first place that should be auscultated?

OVER THE EPIGASTRUM; then lung fields

A fracture that runs at an angle through the bone?

Oblique

What is obsessive compulsive disorder?

Obsessions that are persistent thoughts; compulsions that are repetitive behaviors; rituals that are performed to reduce anxiety. (E.g. hand washing; counting. Etc) Tx: allow them to continue repetition unless dangerous; anxiolytics & supportive therapy

What is appendicitis?

Obstruction of the appendix that leads to peritonitis Most commonly seen in males ages 10-30

Postrenal AKI/RF

Obstruction of urinary tract through calculi or prostatic hypertrophy Tx: based on cause

A massive PE can cause?

Obstructive shock

What shock is d/t obstructed blood flow & how is it managed?

Obstructive. Relieving the obstruction

Pain upon flexion & internal rotation of the hip is known as? And what is it seen in?

Obturator Sign Appendicitis

What is the CRITICAL INCIDENCE STRESS MANAGEMENT?

Occurs after a significant event to educate staff how to grieve through debriefing. Should occur 24-48 hrs s/p

R on T phenomenon

Occurs when electrical impulse too close to RESTING PHASE of ventricle->VT/VF

What is the antidote for sulfonylureas?

Octreotide

What is a TRUE OCULAR EMERGENCY?

Ocular burns; retinal detachment

What are the s/s of a corneal abrasion & how is it managed?

Ocular pain; sensation of foreign body; photophobia; tearing; blurred vision Management: visual acuity; TOPICAL ANESTHETIC (TETRACAINE); FLUORESCEIN STAINING Ophthalmic ABX gtts; nonsteroidal agents for eye; systemic analgesics. Educate on NO PATCHING REQUIRED SINCE THERE IS CONSENSUAL MOVEMENT OF EYES

Late decelerations w/cardiotocography indicate what?

Ominous sign for fetal distress

How do you position a pregnant patient who comes in for trauma related complaints?

On to her LEFT SIDE; TILT BACKBOARD; or MANUALLY DISPLACE UTERUS TO SIDE (d/t SUPINE VENA CAVA SYNDROME)

When is the umbilical cord clamped?

Once it stops pulsating

tumor lysis syndrome

Oncology emergency from lysis of cancer cells. Most common in acute leukemia; may follow chemo or in Burkitt's lymphoma S/s: N/V/D, anorexia, lethargy, hematuria, arrhythmias (d/t >K), cramps & muscle spasms (d/t <Ca) Labs: HYPERkalemia (>6), HYPERphosphatemia (>4.5), HYPERuricemia (>8), HYPOcalcalcemia (<7) Tx: IVF to achieve u/o of 100ml/hr to prevent AKI; Allopurinol (decrease Uris acid production); calcium replacement; renal replacement therapy.

With an inevitable AB, what should the os be?

Open

What is the management for drowning?

Open airway w/jaw-thrust maneuver. Restrict cervical motion. Ventilate. CPR as needed. Passive REWARMING by removing wet clothes; active REWARMING through ECPR/ECMO Decompress stomach with naso/orogastric tube

Penetrating wound through the chest that causes air to become trapped in the intrapleural space is known as?

Open pneumothorax

Patient presents with a SUCKING CHEST WOUND (on inhalation) & bubbling blood (on exhalation); dyspnea; SQ emphysema & CP. what is the possible cause? How is it managed?

Open pneumothorax O2; OCCLUSIVE DRESSING taped securely on THREE SIDES AT END EXHALATION.

CNS/respiratory depression, miosis (pupil contraction), pulmonary edema, apnea, coma can be seen with what type of ingestion?

Opiates

What additional symptoms for meningitis are seen in infants?

Opisthotonos (arched back); bulging fontanelles; irritability

Cavernous sinusitis & meningitis are potential complications of what?

Orbital Cellulitis

Marked periorbital erythema & edema s/p sinusitis, insect sting or laceration is known as?

Orbital Cellulitis

Limited upward gaze & diplopia is often seen in what?

Orbital fracture

What injury presents with limited upward gaze (CN III)?

Orbital fracture

What is a fracture of the orbit (what holds the eye in proper placement)?

Orbital wall fracture

BRONCHORRHEA is seen with what toxicity?

Organophosphate

INFECTION OF THE BONE & surrounding tissue that may lead to sepsis d/t common causes of open fractures; infection in area of fracture; PUNCTURE WOUNDS (like a bite or wound on bottom of foot) are called?

Osteomyelitis

An infection (usually bacterial) of the external auditory canal is called...?

Otitis Externa

An infection of the inner ear canal; blocked Eustachian tube (leading to fluid building up behind TM) are called?

Otitis media

What is polycythemia Vera?

Overactive bone marrow that results in increase in RBC, WBC, & platelets. HCT >55%; hepatosplenomegaly (enlarged spleen & liver); increased blood viscosity Tx: phlebotomy to remove whole blood & infuse NS; chemotherapy to decrease blood cell production.

What are common causes of Respiratory alkalosis?

Overventilation; tachypnea Hyperventilation d/t {P}ulmonary embolus, {A}nxiety, {M}yocardial infarction

How are cluster HA's treated?

Oxygen

What is the Treatment for a STEMI?

Oxygen for SpO2 <94% or respiratory distress Nitroglycerin SL tablet/spray initially, IV infusion as needed; CONTRAINDICATED if SBP <90, HR <50, recent "phils" use ASA 160-324mg chewable; Clopidogrel for antiplatelet effects PCI goal <90mins; fibrinolytic therapy if PCI unavailable w/in 90-120mins; EXPECT REPERFUSION DYSRHYTHMIAS LIKE ACCELERATED IVR or VT- GOOD SIGN!! Beta blockers early for HTN STEMI pts ACE-I/ARBS to reduce infarct size & improve ventricular remodeling

You adjust the FiO2 first than PEEP for what type of problem?

Oxygenation (hypoxic) problems

What is the treatment for spontaneous AB?

Oxytocin; suction curettage; RHoGAM to RH - MOTHERS; psychosocial care for BOTH PARENTS (PARTNERS) BEDREST FOR 1-2 DAYS for threatened AB

How is frostbite managed?

PAIN MEDICATION (opioids) & QUICKLY REWARM affected part for 15-30mins in 40-42C (104-107F) water NSAIDS to block the production of thromboxane. Cotton padding between frost bitten digits. Avoid any friction or rubbing.

What is syphilis?

PAINLESS CHANCRE; rash on palms & soles; delirium/dementia (later years) Dx: VDRL (ventral disease research lab test) & RPR (rapid plasma reagin test) blood tests Tx: Penicillin IM once; Doxycycline or Tetracycline x14 days

Cat bite's have a high risk of infection due to??

PASTEURELLA; which is why we prophylactially give ABX.

Delusions, hallucinations, paranoia, dilated pupils, LACK OF PAIN SENSATION (—>INCREASED STRENGTH) is seen with ingestion of?

PCP

What is a dissociative anesthetic that causes decreased awareness of surroundings & decreased pain sensation?

PCP (Phencyclidine)

An abrupt onset of DYSPNEA and pleuritic CP, restlessness, HEMOPTYSIS, syncope, anxiety, RESPIRATORY ALKALOSIS, & hypoxemia are seen with?

PE

Tachycardia; TACHYPNEA; SENSE OF IMPENDING DOOM; accentuated S2 sound is seen in what?

PE

EKG changes w/hyperkalemia

PEAKED T waves at 5.5-6.5mEq/L Loss of P wave & widening of QRS at 6.5-8 mEq/L SINE wave pattern into VFib

What is contraindicated with placenta previa?

PELVIC EXAM (d/t risk of hemorrhage)

Pleuritic RETROSTERNAL chest pain that worsens with inspiration & supine position accompanied w/global diffuse/WIDESPREAD CONCAVE ST ELEVATION on the EKG & a friction rub is seen with what? How is it managed?

PERICARDITIS NSAIDS; allow to LEAN FORWARD; steroids if refractory

Your patient experiences an extravasation of a vasopressor. What should be used to treat it?

PHENTOLAMINE (Regitine)

What are complications of Gonorrhea?

PID; infertility & ectopic pregnancy

How is a spontaneous abortion managed?

PITOCIN (oxytocin) Suction curettage for inevitable or incomplete abortions RHOGAM to Rh negative mothers Emotional support!

The implantation of the placenta near/over cervical os; hemorrhage can occur w/enlargement (3rd trimester) is known as?

PLacenta Previa

Patient presents via EMS w/sudden onset of PAINLESS BRIGHT RED VAGINAL BLEEDING w/signs of shock. What is going on & how is it managed?

PLacenta previa Fluid/blood replacement; U/s; CBC; TXM; monitor FHT. Turn on LEFT side; consult OB; prepare for transfer to OB or OR.

What should be encouraged for smokers, pts <2 yrs or >65 years old?

PNEUMOCOCCAL VACCINE

What drugs block thyroid synthesis & are used in the treatment of a thyroid storm?

PO propylthiouracil & methimazole

What slows thyroid hormone release & is used to tx thyroid storm?

PO/IV iodine (Lugol's solution)

What type of splint is applied for a boxer's fracture?

POSTERIOR (ULNAR) SPLINT

Pediatric patient presents w/TRIAD of subdural hematoma; posterior rib fractures; & retinal hemorrhages. What are these symptoms for?

PROBABLE SHAKEN IMPACT SYNDROME

What do you see w/2nd degree Type I HB?

PROGRESSIVE prolongation of PR interval that eventually blocks impulse; INCONSISTENT PR & R-R interval

What are possible causes of an upper GI bleed?

PUD; Mallory-Weiss syndrome; freq NSAID use

What are the common causes of nonvariceal upper GI bleed?

PUD; NSAID or ASA use; ETOH use.

What clinical manifestations is most closely associated w/thyroid storm?

PUlmonary edema (elevated HR d/t the increased metabolic rate of the body from the increased thyroid hormones can cause HF—>pulmonary edema)

On an ABG what is INVERSELY PROPORTIONAL to pH?

PaCO2

What are the 6 P's?

Pain Paresthesia Pallor Pressure Paralysis Pulselessness

What are the s/s shingles & how is it managed?

Pain develops first f/b vesicular lesions (follow path of NERVE DERMATOMES; typically do not cross body's midline); SEVERE NERVE PAIN Tx: antivirals; PAIN CONTROL w/analgesics, xylocaine patches & nerve blocks. Prevention: varicella zoster vaccine

What are s/s of a nasal foreign body?

Pain in nasal/sinus cavity; unilateral PURULENT NASAL DRAINAGE; recurrent epistaxis; fever

What sign do you see w/a SCAPHOID fracture?

Pain in the ANATOMIC SNUFF BOX

What are the s/s of compartment syndrome & how is it managed?

Pain out of proportion on passive movement (despite analgesia); pulselessness (late sign) Check compartment pressure; keep in neutral position (AT LEVEL OF HEART); remove restrictive devices; prepare for fasciotomy

When is the FLACC scale used?

Pain scale for children up to 7 years; can be used for adults.

What are the s/s of otitis externa & how is it managed?

Pain w/movement of TRAGUS OR AURICLE; possible periauricular cellulitis; hearing loss; drainage from ear; swelling; erythema Tx: analgesics; ABX; warm otic gtts

What are s/s of bursitis & how is it managed?

Pain, REDNESS, WARMTH, swelling, decreased ROM Tx: NSAIDS; analgesia; bursal aspiration. Educate on RICE; decrease movement of extremity

What are the s/s of ocular foreign bodies & how are they managed?

Pain, photophobia; sensation of "something in eye"; tearing; blurred vision Tx: analgesics (tetracaine) before exam; invert upper eyelid & irrigate w/NS; remove object w/cotton tipped applicator or 25-27g needle; examine cornea for rust ring; tx as corneal abrasion s/p removal.

What are the s/s envenomation from a pit viper bite & how is it managed?

Pain, redness, & swelling to site; PROGRESSIVE EDEMA; BLOOD FILLED VESICLES (CANDIDATES FOR ANTIDOTE) Tx w/placing 2 lg bore IVs; REMOVE CONSTRICTIVE CLOTHING & jewelry; immobilize limb; ANTIVENOM (antivenin) if SEVERE HEMORRHAGIC SWELLING (progressive swelling & bleeding times); repeat until swelling subsides; mark border of edema Q15 mins; Xray for embedded teeth. NO ICE OR TOURNIQUET

What is cervical motion tenderness?

Pain/tenderness w/movement of the cervix by gloved hand (or bimanual examination)

What are the s/s of an ear foreign body?

Pain; ANXIETY/FEAR (INCREASED W/LIVE INSECTS); bleeding; hearing loss on affected side; N/V; DIZZINESS; purulent drainage from ear

What are the s/s of sinusitis & how is it managed?

Pain; NASAL CONGESTION; purulent drainage; malaise; fever; facial swelling; DECREASED TRANSILLUMINATION OF SINUSES Management: frontal view of maxillary sinus, orbits & nasal structures (Water's view X-ray); oral ABX; analgesia; antipyretics; LIMITED USE OF NASAL DECONGESTANTS. Education on monitoring BP FOR HTN FROM ANTIHISTAMINES.

What are the s/s of Keratitis & how is it managed?

Pain; PHOTOPHOBIA; red sclera; decreased vision; purulent drainage Tx: ABX; antifungals; antivirals; CYCLOPLEGICS; systemic analgesics; educate on maintaining a DARK ENVIRONMENT

What are the s/s of Hyphema & how is it managed?

Pain; REDDISH HUE TO VISION Analgesia; steroids; MAINTAIN HOB 30-45 DEGREES. Educate pt to AVOID NSAIDS/ASA; protect eye w/RIGID SHIELD; keep HOB elevated 30 degrees; minimize activities that increase IOP; f/u to monitor for rebleed (MOST COMMON 3-5 DAYS S/P EVENT)

What are the s/s of Iritis & how is it managed?

Pain; REDNESS AROUND THE OUTER RING OF IRIS; blurry vision; photophobia; tearing; decreased visual acuity; irregular shaped pupil Tx: cycloplegics; warm compress, ophthalmology consult

What are s/s of joint effusion & how is it managed?

Pain; REDNESS; WARMTH; swelling; stuffiness; decreased ROM Tx: NSAIDS; RICE; ARTHROCENTESIS. Educate on RICE; decrease movement of extremity

What are the s/s of acute angle closure glaucoma & how is it managed?

Pain; decreased peripheral vision ("TUNNEL VISION"); HALOS AROUND LIGHTS; n/v; HA; reddened eye; DILATED, FIXED PUPIL; CLOUDY CORNEA; FILM FEELING GLOBE; shallow chamber (d/t pressure) Managed: HOB elevated; MIOTIC GTTS (PILOCARPINE); TOPICAL BETA BLOCKERS (TIMOLOL MALEATE); carbonic anhydrate inhibitors (acetazolamide); antiemetics; opioids. Educate on ophthalmology f/u; NO LIFTING > 5 LBS; avoid coughing/straining; DO NOT LOVER HEAD BELOW WAIST

What are the 6 P's of a neurovascular assessment?

Pain; paresthesia (pins & needles; tingling sensation); pallor; pressure; paralysis; pulselessness

What are the s/s of ultraviolet keratitis & how is it managed?

Pain; photophobia; "sand in eyes" Dark environment

What are the s/s of corneal ulcerations & how are they managed?

Pain; photophobia; sensation of FB; tearing; blurred vision; eyelid swelling; "WHITE SPOTS"; purulent drainage Tx: ABX; antifungals; antivirals; cycloplegics.

What is dyspareunia?

Painful intercourse

Raynaud's disease

Pallor of digits, ears, & nose Most common in women (esp w/Lupus & scleroderma)

Epigastric pain (delayed) Abdominal distention Decreased bowel sounds Increased serum amylase/lipase Glucose fluctuations These are indications of what abdominal organ injury?

Pancreas

Drugs that stimulate production of dopamine or replace dopamine (e.g. Bromocriptine) are used to control the symptoms of what disease?

Parkinsons'

What is mumps?

Parotitis Contagious for 16-18 days Swollen salivary glands—>puffy cheeks & swollen jaw Complication: orchitis

What stage of pertussis presents w/UNREMITTING PAROXYSMAL BURSTS OF COUGHING "WHOOP"; PETECHIAL RASH ABOVE NIPPLE LINE D/T BURST BLOOD VESSELS?

Paroxysmal

What are the classifications of seizures?

Partial; complex; general; convulsive; nonconvulsive; febrile

What are you assessing for with the airway of a burn patient?

Patency: edema; hoarse voice; CARBONACEOUS SPUTUM; STRIDOR. Indicates oral burns (& not just singed nasal hairs). INTUBE IMMEDIATELY.

What ekg changes are seen in PE?

Peaked P waves in limb leads, depressed T waves in V1-V3, right BBB, & right axis deviation

What are the EKG changes in hyperkalemia?

Peaked T waves (early); widening of QRS; loss of P waves SINE WAVES (SINE-ing off!)

What happens in an acute chest syndrome?

Peaks in 2-4 yrs & during winter months. CP, SOB, cough & fever. Tx the underlying cause.

How does a radial head subluxation/nursemaid's elbow presents & managed?

Ped patient typically jerked up; refuses to use arm; LIMITED SUPINATION Easily reduced by manual reduction by supination/flexion or forced pronation

A pediatric patient presents w/bilateral BURNS with LINES OF DEMARCATION (clear line of "boundary"). What should this raise a flag to?

Pediatric abuse.

How do you manage preterm labor?

Pelvic exam; continuous fetal monitoring; TOCOLYTICS (stop that labor)- MAGNESIUM OR TERBUTALINE; admit to OB

REBOA (resuscitative endovascular balloon occlusion of the aorta) to occlude the distal aorta to control hemorrhage until definitive care can be implemented is a possible intervention for what?

Pelvic fracture

What are missile injuries?

Penetrating injuries r/t guns & industrial incidents APPEARANCE OF WOUND MAY NOT REFLECT ACTUAL TISSUE DAMAGE

How is syphilis tx'd?

Penicillin IM or doxycycline

How is Priapism dx'd & tx'd?

Penile Doppler or arteriography Tx: urology consult; sedation/analgesia; injection of epinephrine, phenylephrine, or terbutaline; irrigation of corpora w/NS & ASPIRATION OF CLOT.

Beck's triad; PULSUS PARADOXUS & electrical alternans are commonly seen in what? How is it managed?

Pericardial Tamponade Assist w/pericardiocentesis to the Left Xyphoid. Pt improved if the BP is increasing

Patella/knee dislocation is a risk for damage to??

Perineal & POPLITEAL ARTERY

What are Raccoon Eyes? And what are they seen in?

Periorbital ecchymosis d/t intra-orbital bleeding Basilar skull fracture

Ruptured diaphragm

Peristaltic gurgling sounds audible in lower to mid chest; progressive scaphoid abdomen. Requires surgical repair

A patient is lying RIGIDLY STILL; what is this a classic sign for?

Peritonitis

A DEVIATED UVULA is seen in what? How is it managed?

Peritonsillar abscess I&D

appendicitis

Periumbilical to RLQ abd pain (McBURNEY'S POINT) w/REBOUND tenderness (ROVSING's sign); nausea; anorexia

What is McBurney's Point?

Periumblical pain that migrates to RLQ

What is a potential complication of an ankle fracture?

Peroneal nerve damage

SPASMODIC COUGH that leads to VOMITING, GAGGING & EXHAUSTION w/ a petechial rash above the nipple line (d/t coughing) that is highly contagious is known as?

Pertussis (whooping cough)

What are some examples of products that contain salicylates?

Pesto Bismol (bismuth subsalicylate) Oil of wintergreen in Bengay (methyl salicylate)

What is an additional sign seen in meningococcal meningitis?

Petechial rash

What is the antidote for extravasation of a vasopressor?

Phentolamine (Regitine)

The inability to retract the foreskin of a penis is known as? How is it managed?

Phimosis Clinically significant after >4 yrs old Tx: retraction, incision, or cirumcision

What are the s/s of retinal detachment & how is it managed?

Photopsia (flashes of light); floaters/specks in visual field; curtain/veil/or cobwebs obscuring part of visual field Emergent ophthalmology consult; surgical repair

What s/e is seen with the treatment agent DEFEROXAMINE?

Pink-red color urine

What type of snakes are hemotoxic?

Pit vipers which includes rattlesnakes , copperheads & water moccasins (cotton mouth)

How is emergent delivery managed?

Place in low-Fowler w/knees bent up; clean perineum if time allows; ask mother to "pant"; assess for presentation & nuchal cord (cord wrapped around fetus neck 360 degrees); gently guide head & deliver shoulders & body

How is febrile seizure managed?

Place in side lying position; check BGL; administer benzos (Midazolam IN/IM or diazepam IV/rectally); phenytoin or fosphenytoin; consider infusion of propofol or ketamine.

How is an avulsed tooth managed if patient is ALTERED LOC; has concurrent injury; is a child?

Place tooth in saline, in MILK; or in a CALCIUM BASED SOLUTION (HANK'S) Replant tooth w/in 6 hours is possible

What is abruptio placentae?

Placental separation from uterine wall rupturing arterial vessels leading to hemorrhage & shock. S/s: painful contractions & BACKACHE with uterine rigidity; FRANK DARK RED vaginal bleeding or concealed; abnormal FHT. Tx: OB consult; continuous fetal monitoring; TURN PATIENT ONTO LEFT SIDE 15-30 DEGREES; treat shock with BLOOD PRODUCTS STAT; PREPARE FOR IMMED C-SECTION

What is NSTEMI?

Plaque rupture + TROPONIN ABSENT ST elevation

What blood products do not require ABO compatibility testing prior to administration?

Platelets

How do you know if your patient w/DIC is improving?

Platelets are improving

Crunching sound of the heart upon auscultation is seen with?

Pneumomedistinum

Viral, bacterial, or fungal infection in 1 or both sides of lungs that causes fever, pleuritic CP, productive cough, chills, malaise, dyspnea, increased fremitus (vibration felt on chest w/palpation), & decreased breath sounds over affected area is known as?

Pneumonia

What associated injuries may occur w/a clavicle fracture?

Pneumothorax Hemothorax Great vessel injury

What are some of the s/s of DI?

Polydipsia & POLYURIA

Arterial Clot

Popliteal most common. Pain at rest, pale, cyanotic, cold Tx: embolectomy

What causes esophageal varices?

Portal HTN—>dilated veins

LIght touch, vibration & proprioception are associated w/damage to what?

Posterior Cord

Loss of proprioception, vibration, fine touch/pressure but an intact motor function is seen in what?

Posterior Cord SCI

What dislocation is most common with KNEE TO DASHBOARD in MVC?

Posterior hip dislocation

Adducted arm with the inability to rotate arm externally is seen with what dislocation?

Posterior shoulder

Excessive bright red vaginal bleeding accompanied with boggy uterus & s/o shock s/p delivery or abortion & up to 6 weeks postpartum is seen with?

Postpartum hemorrhage

HTN (typically in a previously normotensive patient) after the 20th week gestation w/PROTEINURIA & edema of the face & hands; accompanied w/blurry vision & HA is known as?

Preeclampsia

Wolff-Parkinson-White Syndrome

Preexcitation syndrome where cardiac electrical conduction BYPASSES AV node through bundle of Kent. S/s: tachycardia (SVT), syncopal episodes EKG: SHORT PR interval & DELTA waves (early upstroke/slurring of QRS upstroke)

Pain when lifting the affected testicle is known as?

Prehn's sign

What is measured by CVP (right arterial pressure)?

Preload

What is evaluated by CVP?

Preload (VOLUME)

In what phase does mutual aid agreements, stockpiling, & training occur?

Preparedness

What phase of emergency management involves emergency training (NIMS/HICS); mutual aid agreements; stockpiles?

Preparedness

What is sepsis?

Presence/suspicion of infection w/2 or more SIRS criteria

How is an avulsed tooth managed in an ALERT ADULT?

Preserve by placing by IN SOCKET or in between cheek/gum or under tongue Replant tooth w/in 6 hours if possible

How is a penile amputation managed?

Preserve in saline moistened sterile gauze; place on ice until transplantation

What is the most common OB complication following trauma?

Preterm labor (regular contractions Q10mins or < at <37wks; possible vaginal weeks)

What phase affects AIR FILLED ORGANS where you see TM RUPTURE, alveolar rupture, PNEUMOTHORAX, air emboli, GI contusions/ruptures, cerebral hemorrhages, cerebral air emboli?

Primary

Angina caused by a vasospasm of coronary arteries that can be caused by STIMULANT use (cocaine) is called what?

Prinzmetal's (variant)

What are advantages & disadvantages to using Etomidate for medication assisted airway management?

Pro: little or no decrease in MAP in normovolemic patients Cons: pain upon ejection; involuntary skeletal muscle movements; adrenal suppression

What is Cushing's Triad?

Profound BRADYCARDIA; abnormal respirations (cheyne-stokes); INCREASED SBP w/widened pulse pressure

What are the s/s of tertiary syphilis?

Progressive DEMENTIA; neuropathy; tremulous extremities; thoracic aneurysm

The umbilical cord precedes the fetus through the birth canal obstructing fetal circulation is known as? How is it managed?

Prolapsed cord Place mom in KNEE-TO-CHEST POSITION; instruct mom to not push. Cover exposed cord w/saline moistened sterile gauze. Deliver baby if cervix is dilated or prepare for emergency c-section.

What do you see with 1st degree HB?

Prolonged PR interval >.20; consistent PR & R-R intervals; PR & RR consistent

How is DIC dx'd?

Prolonged coagulation times (PT/PTT); ELEVATED D-DIMER & FIBRIN DEGRADATION PRODUCTS; low hgb, hct, platelets, & fibrinogen

What is heat exhaustion? How is it managed?

Prolonged exposure to heat—>heat cramps, anorexia, vomiting, HA, syncope Tx w/rest in a cool environment & FLUID/ELECTROLYTE REPLACEMENT

What is priapism?

Prolonged painful erection. TRUE UROLOGIC EMERGENCY.

What medications are used to inhibit thyroid hormone synthesis in a patient w/elevated thyroid levels?

Propylthiouracil & methimazole. Needs to be given orally or through gastric tube

Your neonate patient's BGL<40 what is your next step?

Provide 10% dextrose

With JUMP START triage at pediatric patient presents with RR> 45 placing them in the RED. What intervention is provided before moving to black?

Provide 5 positive pressure breaths

What do you do during maternal cardiac arrest?

Provide chest compressions HIGHER; manually displace uterus to the LEFT (prevents VENA CAVA SYNDROME); prepare for emergency C-section

What is an inferior MI

Proximal RCA S/s: JVD; hypotension; shock; ST elevation at V4R at 5ICS R. MCL Tx: Infuse a NS bolus & Dobutamine infusion Caution w/PRELOAD REDUCING agents (NTG & Morphine)

Pain w/flexing thigh at hip is known as? And is seen in?

Psoas Sign Appendicitis

Why is neutropenia/leukemia a ESI level 2?

Pt at high risk for infection d/t low/abnml WBCs

What are the s/s of a penile fracture & how is it managed?

Pt may report a "popping" sound f/b immediate loss of erection; pain; swelling; visible deformity Emergency surgery to prevent impotence or erectile dysfunction

Neutropenia Leukemia

Pt w/leukemia or on immunosuppressive therapy Requires protective (reverse) isolation Avoid invasive procedures (foley/rectal exam) Should avoid raw meat & unwashed produce Start ABX early

Rib fractures of 4-9 are at risk of what additional injury?

Pulmonary contusion & blunt cardiac injury

A thyroid storm can contribute to the onset of what?

Pulmonary edema; heart failure;

What are normal variances in pregnancy?

Pulse increased BP decreased Respiratory alkalosis (PaCO2 27-32) HCT lower d/t dilution

What is cardiogenic shock?

Pump failure. The pump is pooped.

What causes nephrogenic DI?

Pyelonephritis; familial genetic disorder

What presents with PROJECTILE NON-BILIOUS VOMITING as one of the symptoms?

Pyloric Stenosis

Mom brings in her baby & reports PROJECTILE NONBILIOUS VOMITING right after feeding & baby continuously seems hungry. You palpate an olive shaped mass. What could be going on & how is it managed?

Pyloric stenosis U/S for dx; may require pyloromyotomy

How do you determine SVT in pediatrics?

QRS < 0.09; p waves may be absent/abnml HR >220 in infants; >180 in children

Research that involves a lARGE SAMPLE SIZE & SMALL P-VALUE is what?

QUANTITATIVE

What is an inductive process involving INTERVIEWS WHERE WORDS not numbers are used to give meaning to data & involves INTERACTION between researcher & subject?

Qualitative studies

How is an APAP OD managed?

Quantitative level AT 4 HOURS from ingestion; monitor LFTs Consider lavage & ACTIVATED CHARCOAL, N-ACETYLCYSTEINE (acetadote) w/in 8 hours for best response.

What is a deductive process that uses NUMERIC FINDINGS; examines the RELATIONSHIPS BETWEEN VARIABLES & determines the cause & effects of those variables?

Quantitative studies; LOOK FOR SMALL P-VALUE < 0.05 (probability)

In what phase does the inhalation of dust/TOXIC GASES lead to chemical, biological, thermal burns; radiation; nuclear (CBRN)?

Quaternary

An electrical impulse that is too close to the resting phase of the ventricle that could lead to VT or VF is known as?

R on T phenomenon

What do you see w/3rd degree HB?

R to R intervals are consistent; hidden P waves (inconsistent)

How is Bell's Palsy managed?

R/o stroke & meningitis Tx w/antivirals & CORTICOSTEROIDS (shorten progression); analgesics; & EYE LUBRICANTS

How is heat stroke managed?

RAPID COOLING (until 102 or 38.9) because of altered LOC Benzodiazepines to reduce shivering Replace electrolytes

What is delirium?

RAPID ONSET of disorientation; memory impairment; INATTENTION

What are common causes of a posterior shoulder dislocation & how does it present?

RARE. May be seen after a seizure or blow to anterior shoulder Arm held in ADDUCTION close to side

What are s/s of rhabdomyolysis? How is it managed?

REDDISH-BROWN URINE; muscle pain; malaise IVF W/SODIUM BICARBONATE to flush myoglobin out of kidneys & reduce r/o acute tubular necrosis; mannitol; diuretics.

Nitroglycerin

REDUCES PRELOAD & AFTERLOAD Reduced blood flow & decreased O2 consumption->lowered BP CONTRAINDICATED w/phophodiesterase inhibitors (the -FILS)

What is the management of foreign bodies (retained dirt/debris/gravel/glass/wood/metal) in the skin?

REMOVE VEGETATIVE MATERIAL (THORN/WOOD ETC) QUICKLY to prevent infection Irrigate w/NS; DO NOT SOAK Administer tetanus & broad spectrum ABX

START TRIAGE is based on what? (30-2-CAN DO)

RESPIRATION (30), PERFUSION (2), MENTAL STATUS (CAN DO)

If pH is <7.4 & CO2 is >45 or pH is >7.4 & CO2 <35 it is what?

RESPIRATORY

The increased RR of a pregnant woman leads to?

RESPIRATORY ALKALOSIS

What is the classic triad of opioid toxicity?

RESPIRATORY DEPRESSION; CNS depression; MIOSIS (pinpoint pupils)

What are the s/s of opioid ingestion/OD & how is it managed?

RESPIRATORY DEPRESSION; CNS depression; MIOSIS; hypotension; bradycardia; hypothermia. ASSIST W/BREATHING; intubation; Naloxone (Narcan) dose ONLY TO PRODUCE RESPIRATIONS (duration of action 30-60 mins)

Breakdown of skeletal muscle—>release of MYOGLOBIN, CK, & electrolytes (K+) d/t causes such as PROLONGED IMMOBILIZATION, STIMULANT DRUG USE, STATINS, HEATSTROKE, & CRUSH INJURIES is called?

RHABDOMYOLYSIS

What education needs to be provided following an orthopedic emergency?

RICE: rest; ICE AT 20MIN INTERVALS; compression; elevation

How are tendonitis & bursitis managed?

RICE; splints; NSAIDS

Treatment for Head Lice

RID shampoo (teach parents to repeat in 7 days NOT EVERY DAY). Comb for nits.

What TB medication stains body fluids orange?

RIFAMPIN

What do you see w/Right sided HF?

RIGHT w/REST of body JVD; peripheral edema; ascites; hepatomegaly; increased CVP Occurs a lot w/COR PULMONALE secondary from COPD or pulmonary HTN or RV MI

What are the s/s of Crohn's Disease & how is it managed?

RLQ abdominal cramping; N/V/D; weight loss; can affect entire GI tract Colonoscopy for dx Corticosteroids; remicade; humira

What are the s/s of esophageal varices & how are they managed?

RUQ Abd pain; nausea; dyspepsia Coag panel; liver enzymes; H/H Hemodynamic resuscitation w/O2, IVF, & blood products OCTREOTIDE (decreases portal pressure) VASOPRESSIN Transdermal NTG Endoscopic injection sclerotherapy

Cholecystitis

RUQ pain radiating to RIGHT SHOULDER, costal margin tenderness (MURPHY's SIGN)

What is a viral disease that is transmitted through the bite of an infected animal that can cause DELIRIUM; HALLUCINATIONS; EXCESSIVE SALIVATION? How is it managed?

Rabies. Early AGGRESSIVE WOUND MANAGEMENT-WASH IT THOROUGHLY!! Vaccination & immunoglobulins

2 year old patient presents to ED. Mom reports child is refusing to use arm after being yanked by older sibling. Limited supination is noted on exam. What is the potential diagnosis & how is it managed?

Radial head subluxation (nursemaid's elbow) Reduce by placing elbow at 90 degree angle & hyper supinating (turning or holding so that palm/sole is facing upward/outward) the wrist.

Having patient EXTEND WRIST/THUMB (HITCHHIKE OR THUMBS UP) is assessing what nerve?

Radial- assessing sensation to thumb

What is a large experimental research study that randomly assigns patient to EXPERIMENTAL & NONEXPERIMENTAL GROUPS comparing results of 1 form of treatment against a CONTROL GROUP & requires ETHICS BOARD?

Randomized controlled trials (RCT); best to have a LARGE SAMPLE SIZE

What defines a massive hemothorax?

Rapid accumulation of >1500ml of blood in the pleural space.

What are the s/s of heat stroke & how is it treated?

Rapid symptom onset w/Core body temp >41C (105.8F); tachycardia; tachypnea; hypotension; HOT & DRY SKIN; DECREASED LOC; RHABDOMYOLYSIS (from ms breakdown) Tx: COOL PATIENT QUICKLY TO 102. Remove clothing, evaporation not immersion. Cover w/wet sheets & blow fans on pt. Cool IVF (at 39 degrees); CORRECT ELECTROLYTE IMBALANCES. Monitor for RHABDOMYOLYSIS & HYPONATREMIA. Prevent pt from shivering w/BENZODIAZEPINES

What does RVR mean w/afib?

Rapid ventricular rate >100

What is acute renal failure?

Rapidly progressive loss of renal function; <400ml/day (adults) or ( 0.5ml/kg/hr in peds. Prerenal most common type

What is uterine rupture?

Rare. Usually results in fetal demise S/s: sudden onset of severe abdominal pain; vaginal bleeding; FETAL PARTS PALPATED OUTSIDE OF THE UTERUS; fetal bradycardia/asystole; maternal shock Tx: blood products STAT; ABCs; prepare for STAT C-SECTION/HYSTERECTOMY

Ankle-Brachial Index (ABI)

Ratio of BP in ankle to brachium Normal 0.9-1.2 <0.9: PAD Do NOT elevated extremity. Prepare for embolectomy.

What causes shingles?

Reactivation of dormant varicella virus

herpes zoster (shingles)

Reactivation of latent varicella zoster virus (VZV), which resides in the nerve ganglia after an initial chickenpox infection S/s: severe, localized, burning pain F/B (days later) vesicular rash that follows a nerve dermatome (e.g. unilateral along chest, abdomen, back or upper face) Tx: cover blisters w/non-adherent dressing (minimize transmission); antivirals; gabapentin *encourage vaccine in elderly. Pt can return to society when blisters DRIED or covered well*

In what phase does restocking occur?

Recovery

Transporting patients out of a damaged hospital to alternate facilities occurs in what phase of disaster management?

Recovery

What phase of emergency management involves restoring systems; replenishing supplies; disposing of waster; CISM?

Recovery

Hydroxocobalamin causes the urine to be what?

Red

How is an anterior shoulder dislocation managed?

Reduce (may require sedation to overcome muscle spasms); APPLY SLING & SWATH

Mannitol is often used to do what?

Reduce ICP

How is a posterior shoulder dislocation managed?

Reduce; apply sling & swath or SHOULDER IMMOBILIZER

What is the treatment for Esophagitis/GERD?

Reglan (moves food through GI system quickly); antacids (neutralize stomach acids); H2 BLOCKERS & PPIs TO BLOCK ACID PRODUCTION; sucralfate (coats esophagus & stomach)

3rd degree HB

Regular P-P intervals unrelated to regular R-R intervals.

What does SNS stimulation do?

Releases CATECHOLAMINES which INCREASES HR in response to shock (E.g)

How do you assess for Grey-Turner's Sign?

Remember that "P" in TNCC?? POSTERIOR....Turn your patient over & assess the posterior side of your patient. Look for ecchymosis of those flanks

what is the management for a child who swallowed an alkaline battery?

Remove from esophagus w/in 2 hours or liquification saponificiatrion of tissue starts to occur

How is Post renal ARF tx'd?

Remove obstruction

What are the initial interventions following an orthopedic emergency?

Remove rings if a hand injury Immobilize the prevent further damage by splinting. Splint should include the joints ABOVE & BELOW injury Assess neurovascular status prior to splinting and after. Elevate limb after splinting & apply ice pack

What are the common causes of Postrenal ARF?

Renal calculi; urethral blockage (e.g. BPH); neurogenic bladder; tumor Obstruction in flow of urine

What are some of the causes of hyperkalemia?

Renal failures, burns, crush injuries, ace inhibitors, RHABDOMYOLYSIS

What is the treatment of hypokalemia?

Replace K; correct alkalosis (K low = pH HIGH); correct hypomagnesemia; increase K in diets

What is the treatment of hypocalcemia?

Replace calcium; vitamin D; & parathyroid hormone (if needed); increase calcium in diet

What is veracity?

Requirement that we as healthcare workers reveal all pertinent information so an individual can make an informed decision

What is Obstructive Shock?

Resistance to ventricular filling from PERICARDIAL TAMPONADE; tension pneumonthorax; PE; SUPINE VENA CAVA SYNDROME (adv pregnancy); abdominal compartment syndrome (don't flex those hips!) tx: CORRECT UNDERLYING CONDITON

How do you determine that treatment is effective w/a tension pneumothorax?

Respirations are easier

What are the s/s of sedative ingestion/OD & how is it managed?

Respiratory depression; hypotension; bradycardia; hypothermia RISK FOR ASPIRATION Tx w/intubation; FLUMANZENIL (ROMAZICON) for acute toxicity NOT CHRONIC USE (r/o seizures)

What are the s/s of a coral snake bite & how is it treated?

Respiratory distress; local paresthesia; diplopia; ptosis; difficultly swallowing; increased salivation Tx w/supportive care; possible antivenom

What are the causes of bradycardias?

Respiratory distresses; aging; CAD; cardiac defects; medication induced S/s: Hypotension; AMS; s/o shock; cardiac ischemia; acute HF

What are the late signs of ICP?

Responds to pain only or unresponsive on AVPU scale; dilated nonreactive pupil(s); abnormal motor posturing-decorticate/decerebrate

Activation of the Hospital Incident Command Structures occurs during what phase of disaster management?

Response

What phase of emergency management involves evacuation; shelter; disaster triage?

Response

What are the early signs of increased ICP?

Restlessness, drowsiness, HA, vomiting, pupils sluggish to light, amnesia to event; tachycardia, mild HTN

What lab test is associated with production of new RBCs?

Reticulocyte counts

TEAR IN RETINA allowing vitreous humor to leak & reducing blood flow to retina

Retinal Detachment

A PAINLESS unilateral alteration in vision can occur w/what?

Retinal detachment

Photopsia "flashes of light" & floaters are often seen in what?

Retinal detachment

Loss of vision in one portion of the eye is characteristic of what?

Retinal detachment or central retinal artery occlusion

What is paraphimosis?

Retracted foreskin is entrapped causing ischemia to penis. Tx: small incision; consider CIRCUMCISION.

What is paraphimosis? How is it treated?

Retracted foreskin is entrapped—>ischemia to penis Tx w/incision & circumcision

An infection in the area that extends from the base of the skull to the mediastinum which is behind the tissue that makes up the nasopharynx, oropharynx, pharynx, hypopharynx, & larynx is called what? And it carries a risk of what?

Retropharyngeal abscess Airway obstruction, erosion of the carotid artery, jugular vein thrombosis, epidural abscesses & PNA

A child who takes ASPIRIN w/viral illness is at risk for developing?

Reye Syndrome (progressive encephalopathy w/hepatic dysfunction)

What is Cheyne-Stokes breathing?

Rhythm pattern including periods of apnea

JVD, ascites & peripheral edema that is common with cor pulmonale is what?

Right (rest of body) HF

What type of MI should caution be used regarding preload-reducing agents (NTG & Morphine)

Right Ventricular MI

Right sided HF

Right= Rest of body D/t progression of Left HF; lung disease (COR PULMONALE) or RV MI Distended neck veins; peripheral edema; abdominal ascites; hepatomegaly; splenomegaly Support ABCs, O2; NIPPV; diuretics; vasodilators; ACE-I

Patient presents with an avulsed tooth what is the management?

Rinse gently w/o touching. Store in CALCIUM BASED SOLUTION

aortic aneurysm

Risks: HTN, atherosclerosis, Marfan's Syndrome, Ehrler Danlos; cocaine use S/s: SEVERE RIPPING, tearing CP that radiates to back; pulsation mass; SBP difference of 20 between arms; Stroke like symptoms (aortic arch). Anuria, paraplegia, & loss of distal pulses (descending) Traumatic seen w/1st & 2nd rib fractures Dx: WIDENED mediastinum & obscured AORTIC KNOB on CXR; TTE/TEE; CT or MRI. Tx: O2; 2 lg bore IVs; keep HR 60-80 w/beta blockers (prevent reflex tachycardia); keep SBP 100-120 w/Nipride or NTG; opiates for pain

What causes non pruritic (NON BLANCHING) macules on the PALMS, WRISTS, FOREARMS, SOLES & ANKLES; n/v; fever; chills? How is it managed?

Rocky Mountain Spotted Fever (RICKETTSIA) Tx w/ABX (doxycycline)

What is the antidote for benzodiazepines?

Romazicon (Flumazenil)

Pressure applied to the LLQ intensifies RLQ pain is known as? And what is it seen in?

Rovsing's sign Appendicitis

What is an arterial gas embolism and how is it managed?

Rupture of air/nitrogen into PULMONARY VEIN & into LEFT HEART S/s- dysrhythmias; MI; cardiac arrest; focal motor, sensory, and/or visual deficits; seizures; death; cyanotic marbling & focal pallor of tongue; hematuria; proteinuria; RF; uterine & GI bleeding Immediate administration of 100% O2; place in supine position; HBO

What is a penile fracture?

Rupture of tunica albuginea or corpus cavernosa of penile shaft d/t torque (direct trauma/fall/sexual activity); report of "pop"; penile pain & IMMEDIATE LOSS OF ERECTION; urethral bleeding; edema & ecchymosis Dx: penile doppler Tx: SURGICAL REPAIR. Risks of erectile dysfunction.

What should you suspect if a patient presents w/inability to voic especially with a pelvic fracture?

Ruptured bladder

Anaphylaxis

S/S: Angioedema, facial swelling, difficulty swallowing, respiratory distress, s/o shock Tx: Epinephrine 0.2-0.5mg IM in the VASTUS LATERALIS H1 (e.g. diphenhydramine) & H2 antihistamines (e.g Ranitidine or Cimetidine) to help w/hives O2 & Beta-2 agonists (e.g. Albuterol) for bronchospasms Steroids for swelling (slower effect)

What are s/s of DKA?

S/o DEHYDRATION/SHOCK-tachycardia; n/v; abd pain; rapid, deep respirations (KUSSMAULS); ACETONE odor of breath; mental status changes

What do beta blockers mask?

S/o shock& hypoglycemia

What is SLUDGE?

S/s of cholinergic toxicity Salivation Lacrimation Urination Defecation GI distress Emesis (Plus BRONCHORRHEA-INCREASED BRONCHIAL SECRETIONS)

What is a ruptured ovarian cyst?

S/s: acute pain with sex/exercise; SHARP UNILATERAL PAIN Tx: analgesia; possible surgical intervention.

What is tachycardia?

S/s: anxiety; palpitations; chest discomfort; SOB; hypotension; mental status changes Tx: cause of SINUS

What is an Acute Coronary Syndrome

S/s: chest tightness; jaw/neck/left arm/ epigastric pain; scapular discomfort; N/V; s/o shock; dysrhythmias; diaphoresis; dizziness Women c/o increased fatigue Labs: TROPONIN (best indicator of tissue damage); myoglobin H/o use of phosphodiesterase inhibitor use w/in last 24hrs & cocaine (leaves unopposed alpha stimulation if giving beta-blockers)

Pulmonary TB (active)

S/s: cough >3wks; night sweats; fever; chills; wt loss; anorexia; fatigue; coughing up blood Dx: CXR; sputum culture for acid-fast bacilli. TB skin test (read w/in 48-72hrs) Tx: months of combination therapy (isoniazid & rifampin). Airborne & droplet precautions-place in negative pressure room STAT Teaching: med compliance. Containment of tissues in ziplock bags.

What are shunt dysfunctions?

S/s: decreased LOC; bulging fontanelle (infant); PROJECTILE VOMITING (increased ICP) Complications: infections (tx w/ABX); obstruction (aspirate CSF or surgical intervention); mechanical failure from mal position (surgical intervention)

What is a simple pneumothorax?

S/s: decreased/absent BS on affected side; HYPERRESONANCE (tympanic) to percussion Tx: high Fowler's position; O2; potential CHEST TUBE PLACEMENT at 5-6th ICS mid-axillary line

Pulmonary Contusion

S/s: dyspnea; increased WOB; hypoxia; chest wall contusion Tx: Keep SpO2 94-98%; administer IVF judiciously

Hyponatremia

S/s: fatigue; N/D; confusion; SEIZURES

What is hyperemesis gravidarum?

S/s: intractable nausea; vomiting; dehydration—> electrolyte imbalances & malnutrition Tx: IVF; ELECTROLYTE & VITAMIN REPLACEMENT; antiemetics; possible TPN

What is PTSD?

S/s: re-experiencing the traumatic incident (flashbacks; nightmares); avoidance behaviors; difficulty concentrating; HYPERAROUSAL & HYPERVIGILANCE (STARTLE & TENSE) Tx: provide a LOW STIMULUS AREA & counseling therapy

Peripheral Venous Insufficiency

S/s: varicose veins; swelling; leg pain; leathery-looking skin; stasis ulcers; venous claudication (intermittent cramping w/ambulation); delayed wound healing Complications: DVT; PE Teach pt to decrease risk: exercise regularly, lose wt, stop smoking, compression stockings.

blunt cardiac injury

S/s: ventricular dysrhythmias; chest pain; chest wall ecchymosis Tx: monitor cardiac rhythms. Tx dysrhythmias; administer analgesics

What is a SUBARACHNOID HEMORRHAGE?

SAH. Usually d/t aneurysm or AV malformation Explosive or "WORST HA OF MY LIFE" (74% of c/o); altered LOC (53%); N/v; photophobia; focal deficits; nuchal rigidity. Manage ABCs; raise HOB; control SBP; prepare for surgical intervention (clipping or coiling)

How is hypotension determined in peds?

SBP <60 in neonate (0-28 days) SBP <70 in infant SBP 70 + (2 x age) in 1-10 years old SBP <90 in >10 years

What determines a Hypertensive Emergency/Crisis?

SBP > 180 or DBP >120 + E/o impending end organ damage (I.e. AMI, aortic dissection, HF, pulmonary edema, intracranial hemorrhage, hypertensive encephalopathy, eclampsia)

hypertensive crisis

SBP >180 or DBP >120 w/evidence of (impending) end organ damage (AMI, aortic dissection, HF, pulmonary edema, cerebral infarction, intracranial hemorrhage, eclampsia) S/s: HA, CP; epistaxis, oliguria; visual disturbances Dx: EKG; BUN/creat; BP monitoring Tx: O2, IV, consider arterial line. Lower BP/MAP by 20-25% in first 1-2hrs w/nitro, Nitroprusside or labetalol E.g. initial BP 180/120 (map of 140). Lower map to 105

In what phase does flying debris become high speed PROJECTILE leading to penetrating injuries, lacerations, impaled objects, & sharp force trauma?

SECONDARY

Hyponatremia comes with what risk?

SEIZURES

What is complication of SIADH?

SEIZURES

How is syphilis dx'd?

SEROLOGIC TESTING VDRL & RPR

How do you diagnosis CO poisoning?

SERUM CARBOXYHEMOGLOBIN (don't trust that SpO2)

Body temp <33C; comatose w/no shivering, "Osborn or J wave", bradycardia, poss VFib is considered? And how is it managed?

SEVERE HYPOTHERMIA ACTIVE CORE REWARMING (warmed IVFs; heated humidified O2; warm peritoneal, gastric, or colonic lavage; HD)

What is ARDS?

SEVERE HYPOXEMIA that is REFRACTORY to HIGH CONCENTRATIONS of O2 & loss of surfactant

What are s/s of compartment syndrome & how is it managed?

SEVERE PAIN DISPROPORTIONATE TO THE INJURY THAT IS UNRELIEVED BY NARCOTICS PULSELESSNESS (LATE SIGN) Managed: measure compartment pressure w/compartmental pressure monitoring device (20-30 requires close observation); ELEVATED TO LEVEL OF HEART (NEUTRAL POSITION); remove any external compression; surgical decompression via FASCIOTOMY w/pressures >30

The triad of SDH, retinal hemorrhages, & rib fractures are seen with what?

SHAKEN IMPACT SYNDROME

What are the stages of Grief?

SHOCK DENIAL ANGER BARGAINING

What type of management is done for foot metatarsals?

SHORT LEG WALKING CAST

What are the s/s of a fractured femur & how is it managed?

SHORTENED LEG; EXTERNAL ROTATION; swollen thigh TRACTION SPLINT for mid-shaft fractures to REDUCE BLOOD LOSS & pain ORIF for femoral head fracture Highest risk of FAT EMBOLI (12-48hrs s/p injury)

The excessive secretion of ADH from the pituitary gland is called?

SIADH

What disorder causes a DECREASED urine output?

SIADH

What are common causes of priapism?

SICKLE CELL, leukemia, SPINAL CORD INJURY; PSYCHOTROPICS (e.g. TRAZADONE); erectile dysfunction meds

Sepsis

SIRS + suspected/present source of infection

What are the s/s of decompression sickness & how is it treated?

SOB; crepitus; numbness; tingling; diplopia; PETECHIAL RASH; SEIZURES; joint discomfort; pain Tx w/O2 administration; fluids; analgesia; HBO; heliox

What ingestion causes tachypnea, N/V, abdominal pain; TINNITUS; hypoglycemia? And how is it managed?

SODIUM BICARBONATE for urine alkalization DEXTROSE for hypoglycemia HD

What is Chvostek's sign?

SPASM OF LIP & CHEEK

What are complications of Mononucleosis?

SPLENOMEGALY (WATCH FOR SPLENIC RUPTURE; AVOID STRENUOUS ACTIVITIES; RETURN TO ED FOR LUQ & L.SHOULDER PAIN) Hepatomegaly

What medications should you NOT take St. John's Wort with because of risk for SEROTONIN SYNDROME?

SSRIs; TCAs; MAO inhibitors; migraine medications; dextromethorphan

EKG change for INJURY

ST elevation

What ekg changes occurs with injury?

ST elevation

What is STEMI?

ST elevation Complete obstruction of vessel(s) w/thrombosis + TROPONIN

What is a Posterior MI?

ST elevation in V7-V9 or DEPRESSION in V1 & V2

What EKG changes would you see w/ISCHEMIA?

ST segment DEPRESSION/inverted or tall T waves

What EKG changes would you see w/INJURY?

ST segment ELEVATION (STEMI); T wave may INVERT

What EKG changes would you see w/a STEMI/occluded vessel?

ST segment ELEVATION in at least 2 CONTIGUOUS leads

Non-STEMI

ST segment depression or T wave inversion from plaque rupture. Positive TROPONIN.

STEMI

ST segment elevation in CONTIGUOUS leads from obstruction of coronary vessel + TROPONIN.

What is the Frank-Starling law of the heart?

STRETCH heart w/VOLUME to increase CONTRACTILITY

What breath sound is associated w/an upper airway obstruction?

STRIDOR

How do you tx an upper GI bleed?

SUCTION & SECURE THAT AIRWAY if actively bleeding; IV access for fluid & BLOOD replacement (hgb <7); questionable gastric tube

What is the common cause of uterine rupture?

SUDDEN DECELERATION INJURY

What are the s/s of testicular torsion & how is it managed?

SUDDEN ONSET OF SEVERE INGUINAL PAIN; nausea; lack of cremasteric reflex; WORSENS W/ELEVATION & ICE Managed w/color Doppler u/s; manual depression may be attempted; surgical intervention required

What are renal calculi?

SUDDEN ONSET OF SEVERE, COLICKY FLANK pain that may radiate to groin; restlessness & pacing; urgency, frequency, dysuria, & hematuria; diaphoresis. Dx: U/a for hematuria; helical CT to R/O AAA Tx: IVF; NSAIDS; antiemetics; increase fluid intake; opioids; STRAIN URINE FOR ANALYSIS of stone type (most are calcium based); HOSPITALIZE IF UNABLE TO KEEP DOWN PO FLUIDS.

Patient presents with unstable VT w/a pulse. How is it managed?

SYNCHORNIZE CARDIOVERSION on the R wave

A patient with an incomplete spinal cord injury will have what intact?

Sacral sparing aka "anal wink"

What discharge teaching would you give to a patient with epididymitis?

Safe sex practices Partners need to be treated

What d/c teaching needs to be provided for epididymitis?

Safe sex practices; need partner treated

How is hyponatremia managed?

Saline (either 9% or hypertonic 3%)

Mumps

Salivary & parotid gland swelling. Risk of orchitis (inflammation of the testicles). Standard & airborne precautions

SLUDGE MD BBB is a mnemonic for what & what toxicity is it seen with?

Salivation, Lacrimation, Urination, Defecation, GI distress, Emesis Miosis, Diaphoresis BRONCHORRHEA, Bradycardia, Bronchospasm Cholinergic toxicity

What buries in the upper layer of skin; is spread by direct skin to skin contact & causes papular rash (vesicles/scales) w/an intense PRURITIS that increases at night? How is it treated?

Scabies/lice Tx w/permethrin cream (2 applications 1 week apart)

What fracture presents with pain at anatomic "snuff box"?

Scaphoid (navicular)

A fracture of one of the eight carpal bones (typically from a fall on outstretched hand) that presents w/PAIN OVER DEPRESSION IN THE WRIST ON THE THUMB SIDE is known as a?

Scaphoid fracture

What is a good outcome for APGAR?

Score of 7-10

HAs caused by another source are called what?

Secondary HAs

What causes CRACKLES?

Secretions

What do you do during ROSC?

Secure airway & optimize ventilation/oxygenation (ETCO2 35-45 & SpO2 >90%) Increase circulation w/1-2L of NS & vasopressor by wt to get SBP >90 TTM at 32-36 C for at least 24hrs. Control shivering, provide sedation, analgesia, & NMB

Post Arrest Resuscitation

Secure airway. Optimize ventilation & oxygenation. (ETCO2 35-45). Do NOT hyperventilate. Support circulation w/1-2L NS & vasopressor for SBP >90 Initiate TTM at 32-36 degrees C for 24-48hrs.

How is anticholinergic toxicity managed?

Sedate w/benzodiazepines; HALDOL; cool patient; PHYSOSTIGMINE IV SLOWLY.

priapism

Seen in sickle cell, leukemia, spinal cord injury & psychotropic meds (e.g. trazadone, risperdal)

A blood sugar drop faster than 65-125mg/dL/hour can precipitate what?

Seizures

Hypercalcemia is a frequent cause of what?

Seizures

What is the parietal lobe responsible for?

Sensory interpretation

severe sepsis

Sepsis + lactic acidosis or SBP <90 or SBP drop >40 of baseline Organ dysfx, hypotension or hypoperfusion

What is toxic shock syndrome?

Sepsis r/t retained tampons/sponges or secondary to necrotizing fasciitis S/s: sudden onset of HIGH FEVER; n/v; SUNBURN LIKE RASH ON PALMS/SOLES THAT PEEL (DESQUAMATION) Tx: CONTACT ISOLATION; ABCs; identify & remove source of infection; immediate ABX administration (sepsis protocol)

Low SVR & High CO/CI are commonly seen in what early stage of shock?

Septic

Patient presents with c/o hematochezia (blood from anus); painless bleeding & s/o shock. What diagnostic tests will you run? And based on the symptoms what are you treating? And with what?

Serial H/Hs; colonoscopy. Treating a lower GI bleed. Will place a IV access for fluid & BLOOD REPLACEMENT.

How do you dx upper GI bleed?

Serial h/h; coats; TXM; ENDOSCOPY (VASOPRESSIN may cause cardiac ischemia; consider NTG); high BUN (dry)

What is status epilepticus?

Series of continuous seizures lasting 5 or minutes that are unresponsive to traditional therapy; consequence of hypoxia, acidosis, & HYPOGLYCEMIA. Tx: ABCs. Identity & tx cause (antipyretics); continue LORAZEPAM & phenytoin or fosphenytoin; thiamine & dextrose; consider paralytics.

What labs should be done for APAP ingestion?

Serum APAP level initially & at 4 hrs s/p ingestion; plot on the RUMACK-MATHEWS NOMOGRAM Liver function studies; PT/PTT; bilirubin

How is a diagnosis of salicylates toxicity obtained?

Serum salicylate level initially & Q4hrs until concentration has peaked (may be 12-14hr for enteric coated tablets)

How do you set up a transcutaneous pacer?

Set the rate for 60-80 w/60 mA; increase until capture is obtained. Check for ELECTRICAL capture by seeing spikes followed by wave Check for MECHANICAL capture by palpating pulse

What are the s/s of a sympathomimetic/stimulant ingestion/OD & how is it managed?

Severe HTN; tachycardia; HYPERTHERMIA; DILATED PUPILS (MYDRIASIS) Tx w/sedation w/BENZODIAZEPINES; control BP/HR; MANUAL COOLING

Thyroid Storm (thyrotoxicosis)

Severe HYPERMETABOLISM from overactive thyroid gland (aka GRAVES); excessive production/secretion of hormones; ingestion of hormones S/s: extreme HYPERthermia, TACHYDYSRHYTHMIAS, restless, pulmonary edema, agitation, tremors, mania, goiter, exopthalamus Labs: T3 & T4 HIGH; TSH LOW Tx: Decrease HR w/BETABLOCKERS. Administer antithyroid drug propylethiouracil (PTU)-blocks hormone synthesis. Iodine to block conversion of T4 to T3. Manual cooling & APAP to reduce temperature. No ASA.

What is a Diffuse Axonal Injury?

Severe TBI. Widespread microscopic hemorrhage (NO FOCAL LESION) Leads to immediate & PROLONGED COMA (reticular activating system affected); HTN; hyperthermia; excessive sweating; & ABNORMAL POSTURING

Myxedema Coma

Severe complication of HYPOthyroidism- decreased metabolism & progressive mental deterioration. More common in older females during cold weather. Mortality rate of 60% even w/treatment S/s: fatigue, weight gain, tongue swelling (macroglossia), confusion to coma ("Myxedema Madness") Labs: DECREASED T3 & T4; Elevated TSH Tx: ABCs; rewarm; rehydrate; IV levothyroxine (T4)

What are S&S of cholecystitis?

Severe crampy RUQ PAIN RADIATING TO RIGHT SHOULDER; aggravated by deep breathing; pain often s/p fatty foods or large meal; fever; jaundice (sclera); dark urine. MURPHY SIGN (POINT TENDERNESS UNDER RIGHT COSTAL MARGIN); FLATULENCE

RV failure

Severe hypotension, absence of pulmonary congestion, increased CVP & JVD.

When is an OSBORNE "J" WAVE seen on the ekg?

Severe hypothermia

What are the s/s of ocular Burns & how are they managed?

Severe pain; photophobia; decreased visual acuity; tearing; involuntary spasms/closing of eyelids (BLEPHAROSPASM) Managed: IMMEDIATELY IRRIGATE (do not delay for assessment or visual acuity); TETRACAINE & irrigate w/NS/LR UNTIL PH IS 7-7.4 Tetanus; ophthalmology consult Educate on getting a OPHTHALMIC APPOINTMENT WITH 24 HOURS; dark environment

What is a tension pneumothorax?

Severe respiratory distress; absent/decreased BS; JVD & trachea DEVIATED to UNAFFECTED SIDE (late sign) along w/CYANOSIS Tx: NEEDLE DECOMPRESSION at 2nd ICS over 3rd rib (or 4-5th ICS) & immediate decompression

What are s/s of a tension pneumothorax & how is it managed?

Severe restlessness, significantly diminished or ABSENT BREATH SOUNDS on injured side; hypotension that worsens w/inspiration; distended neck & head veins; TRACHEAL DEVIATION TOWARD UNINJURED SIDE Decompress at 2nd MCL or 4th MAL over rib; thoracostomy

septic shock

Severe sepsis + hypotension despite adequate fluid resuscitation

What are the s/s of a dislocation & how is it managed?

Severe swelling; nerve/vein/artery damage; joint deformity; inability to move joint Assess distal pulse; splint as found; analgesia & sedation for reduction (CAREFUL MONITORING); have REVERSAL AGENTS available.

What are the s/s of otitis media & how is it managed?

Sharp ear pain; PULLING AT EAR; hearing loss; sensation of fullness; BULGING OF TM; h/o URI Tx: analgesics; possible systemic ABX; antipyretics

Carbon monoxide causes what to the oxyhemoglobin dissociation curve?

Shift to the LEFT (decreased O2 delivery to tissues)

What is Peripheral Artery Disease?

Shiny skin; hair loss; constant excruciating BURNING pain; cold extremity; decreased or no pulse Dx: ANKLE BRACHIAL INDEX Tx: DO NOT ELEVATE. Prepare for embolectomy for critical limb ischemia

What do Beta Blockers mask the signs of?

Shock and Hypoglycemia

What are the stages of grieving?

Shock/denial ANGER Bargaining Depression Acceptance

What s/s are seen with CCB ingestion & how is it managed?

Significant bradycardia, heart blocks, & hypotension CALCIUM CHLORIDE/GLUCONATE 15mg of glucagon if hypotension persists Dopamine infusion Transcutaneous pacing

What are the s/s of primary syphilis?

Single, painless genital ulcer (CHANCRE); inguinal adenopathy

What are common causes of rhabdomyolysis?

Skeletal muscle destruction s/p crush injury Overexertion Medications (e.g. Statins; stimulants) Heroin use

What are the LATE signs of hypoglycemia?

Slurred speech & disorientation

What is a delta wave & when is it commonly seen?

Slurred upstroke in the QRS complex associated with a short PR interval WOLFF-PARKINSON-WHITE

Patient presents w/minimal distention; copious FECAL VOMITING; crampy pain; reports a rapid onset. What kind of obstruction is this?

Small bowel

Patient presents with c/o rapid onset, crampy abd pain; copious FECAL VOMITING; minimal distention is noted. What are these s/s manifesting?

Small bowel obstruction

What are some of the anatomical differences in Pediatrics?

Smaller airway diameter (^ risk of airway obstruction from edema/foreign bodies) Cricoid cartilage narrow (secretions accumulate) Large occiput (infants/young children) Cartilaginous Ribs (fractures uncommon) HR increases to maintain perfusion Bradycardia is ominous sign

What are the s/s of cyanide poisoning? How is it managed?

Smell of "bitter almonds" (only 40% of population can smell this) Tachypnea Anxiety; confusion; nausea; possible "cherry red" skin Hydroxocobalmin (Vit B12) Sodium bicarbonate (tx metabolic acidosis)

What does a patient with Ménière's disease need to avoid?

Smoking, caffeine, chocolate, alcohol & salt.

What are medications that can cause a prolonged QTi?

Sotalol; procainamide; ABX like ERYTHROMYCIN, levofloxacin & ciprofloxacin; psychiatric meds like HALDOL & lithium; TRICYCLICS antidepressants (elavil & tofranil)

What is SCIWORA?

Spinal cord injury WITHOUT radiographic injury Seen in children <8; get an MRI

What is the treatment of choice for the neuro/spinal shocks?

Spinal motion restrictions; support those ABCS; AUGMENT VASCULAR TONE W/IV fluids; vasopressors (norepinephrine); & positive inotropes (dopamine)

What are examples of suspected child maltreatment?

Spiral fracture in a nonmobile infant BURNS WITH LINES OF DEMARCATION

LUQ pain w/possible radiation to left shoulder (especially in the supine position) Periumbilical bruising (Cullen's sign) Possible Left 9-12 Rib Fractures May see HOURS after injury due to encapsulation. Common MOI going over handlebars of bike Are all indications of what abdominal organ trauma?

Spleen

Patient presents w/LUQ pain that radiates to L. Shoulder (KEHR's); muscle rigidity; L. Flank ecchymosis (Grey Turner's sign) & guarding. What abdominal organ has been impacted?

Spleen

What are lower (9-12) rib fractures associated with?

Spleen (Left side) and Liver (right side)

How is an ankle dislocation managed?

Splint in neutral position; prepare for open reduction d/t blood vessel & nerve impingement

Sudden onset of pleuritic CP, dyspnea, tachypnea, decreased BS on affected side, SQ emphysema; hyperresonance w/percussion are s/s of what? How is it managed?

Spontaneous Pneumothorax Position in high Fowler's; O2; chest tube insertion in 5th or 6th ICS MAL to expand lung if >15%

Brown/bloody discharge to profuse vaginal bleeding w/passage of tissue or products of conception accompanied w/pelvic pain occurs during?

Spontaneous abortion

What is the difference in a stable versus an unstable pelvic fracture?

Stable has one point broken; unstable has multiple points broken

How is C. Diff managed?

Standard & Contact enteric isolation Tx: STOP ABX; IVFs; antiemetics; Flagyl; fecal transplant (chronic)

What are the potential complications of a human bite?

Staph, strep, & EIKENELLA CORRODENS infection Open fracture

What is anorexia?

Starvation. Amenorrhea; thinning hair; hypercarotenemia (yellowing of skin; increased beta carotene level)

What education do you need to provide your patient regarding Doxycycline?

Stay out of the sun

What does parathyroid hormone do?

Stimulates the release of calcium from the bone

What has venom coated barbed stingers that cause severe pain that peaks in 1-2hours; & swelling at site? And how is it managed?

Sting Rays IMMERSE IN WARM WATER (110F) for up to 2 hours UNTIL RELIEF OF PAIN; remove barbs w/hemostats

Generalized abdominal pain Hematemesis Decreased BS Rigid abdomen Septic appearance These are indicators of what abdominal organ injury?

Stomach/Bowel

How are pediatric burns managed?

Stop the burning process- cover w/sterile sheet; prevent hypothermia Warm LR IVFs at 3ml/kg/TBSA (1st half in 8hours) Monitor u/o

What finding is noted with trichomoniasis?

Strawberry cervix; elevated pH

What are the s/s of an ASCENDING aortic dissection?

Stroke like symptoms; Horner's syndrome (Ptosis, anhidrosis {inability to sweat}, miosis)

Nitroprusside

Strong PRELOAD & AFTERLOAD REDUCER. Used in hypertensive crisis & CHF. Complications: CYANIDE toxicity & METHEMOGLOBINEMIA.

What is nitroprusside?

Strong PRELOAD & AFTERLOAD reduction VasoDILATOR

A patient presents with the "WORST HA of my life". What is the concern? How is it treated?

Subarachnoid hemorrhage. CALICUM CHANNEL BLOCKERS

Elderly Patient presents to ED after family notices a gradual decline in LOC. Patient has been c/o HA & vomiting. What do you suspect is going on?

Subdural hematoma

What is the Triad for abusive head trauma/shaken impact?

Subdural hematoma; retinal hemorrhage; posterior rib fractures.

What is a steeple sign? And what is it seen in?

Subglottic narrowing seen on AP neck xray Croup

A depolarizing neuromuscular blocking agent that is often used in drug assisted intubation that has FASCICULATIONS that signal the onset of paralysis is known as?

Succinylcholine (anectine) (can trigger malignant hyperthermia)

What are the s/s of an open pneumothorax & how is it managed?

Sucking chest wound on inhalation; bubbling blood on exhalation; subcutaneous emphysema O2; occlusive dressing (nonporous like plastic wrap) taped securely on 3 sides at end-exhalation. Monitor for s/s of tension pneumothorax; remove dressing if s/o increased respiratory distress

What are the s/s of sickle cell disease?

Sudden EXPLOSIVE PAIN in abdomen, chest, back, & joints; splenic ischemia (increases r/o infection); cardiac ischemia; PRIAPISM Dx: CBC (detect infection); reticulocyte count (does not correlate to pain)

What are the s/s of ruptured TM & how is it managed?

Sudden SEVERE SHARP EAR PAIN (may be relief of pain s/p rupture); BLOODY/PURULENT DRAINAGE; vertigo; tinnitus; hearing loss Tx: analgesia; DO NOT IRRIGATE; abx ; educate on keeping EARS DRY; wear ear protection

What are s/s of retinal detachment & how is it managed?

Sudden decrease or loss of vision; VEIL OR CURTAIN EFFECT; FLASHES OF LIGHT (PHOTOPSIA); FLOATERS; specks of vision Ophthalmic referral; prepare for surgical intervention

What are s/s of pancreatitis?

Sudden onset of SHARP EPIGASTRIC PAIN RADIATING TO THE BACK; aggravated by eating, etoh intake, laying supine; fever; N/V/A; s/o shock Pain relieved by leaning forward D/x: ELEVATED WBC, AMYLASE (EARLY), LIPASE (LATE), GLUCOSE, & TRIGLYCERIDES; LOW CALCIUM level; need a CT or U/S of abdomen

What are the s/s of pancreatitis & how is it managed?

Sudden onset of epigastric to LUQ pain that radiates to back; pain decreases when leaning forward; N/V/A; abdominal distention; fever; tachycardia; hypotension; elevated WBC; elevated AMYLASE (EARLY); lipase (more specific); LOW CALCIUM Abdominal CT CXR (identify pleural effusion) IVF; narcotics; antiemetics; CALCIUM REPLACEMENT

What are the s/s of cholecystitis & how is it managed?

Sudden onset of epigastric to RUQ pain after eating fried/fatty foods; fever; right subcostal margin tenderness (MURPHY'S SIGN); FLATULENCE; n/v CBC; CMP; U/s (shows stones); CT IVF; antiemetics; broad spectrum ABX; prepare for laparotomy or laparoscopic cholecystectomy

What are the s/s of central retinal artery occlusion & how is it managed?

Sudden onset of painless loss of vision; 'CURTAIN/SHADE CAME DOWN OVER EYE"; cherry red spot on eye exam; AMAUROSIS FUGAX (transient episodes of blindness); INCREASED IOP Managed: HIGH TRIAGE ACUITY; digital massage by MD; topical BETA BLOCKER; acetazolamide; SUBLINGUAL NITROGLYCERIN (DILATE THOSE VESSELS); fibrinolytic therapy; hyperbaric

What are the s/s of a ruptured ovarian cyst & how is it managed?

Sudden onset of sharp pelvic pain & bleeding Pain control; hypovolemia; possible surgical intervention.

What are the s/s of kidney stones & what is the treatment?

Sudden onset of unbearable flank pain radiating to groin; restlessness & pacing; diaphoresis & pallor; frequency & urgency; hematuria (microscopic to gross) KUB; IVP; helical CT. IVFs; antiemetic; NSAIDS; narcotics; hospitalize if unable to keep fluids down

What are REG FLAGS of a HA?

Sudden onset w/peak intensity; "EXPLOSIVE"; "worst of life"; within minutes (SAH); nuchal rigidity & fever (meningitis); trauma & decreased LOC (intracranial bleed); escalating (tumor)

What are the s/s of a central retinal artery occlusion & how is it managed?

Sudden painless unilateral vision loss; amaurosis fugax (temporary loss of vision) Immediate ophthalmology consult; possible ocular massage; beta blockers; diamox; NTG; TPA; or HBO

pancreatitis

Sudden sharp EPIGASTRIC pain radiating to back Amylase (EARLY) elevated Lipase (LATER) elevated Calcium decreased Risk of pleural effusion & ARDS

What is orchitis?

Sudden swelling of the testes

What are s/s of trigeminal neuralgia & how is it tx'd?

Sudden, unilateral, severe, STABBING PAIN on one or more branches of the CN V (trigeminal); facial twitching (provoked by brushing teeth or chewing) Tx: TEGRETOL (carbamazepine); phenytoin; valproate acid; gabapentin; lamotrigine; clonazepam

A mid shaft humerus fracture is managed with what and what is a potential complication?

Sugar tong splint Radial nerve damage

The deviation of the eyes downward revealing the sclera above the irises is known as what? And it's a s/o what?

Sun-setting sign Increased ICP (esp in infants)

Patient presents with HA, blurred vision, EDEMATOUS FACE. They have h/o breast CA. What could the cause be? How is it managed?

Superior Vena Cava Syndrome. Increase HOB & oncology consult

What does the RCA supply?

Supplies blood to the SA & AV node

What is the treatment for HF?

Support ABCs. Cardiac monitoring. Keep SpO2>90%. Caution w/IVFs. Loop diuretics to DECREASE PRELOAD. VasoDILATORS (NTG). ACE/ARB to interrupt RAAS

What it is the tx of aortic dissection?

Support those ABCs: 2 large bore IVs.. Maintain HR 60-80 and SBP 100-120s w/ IV beta blockers first to AVOID reflex tachycardia; Nitroprusside; nitroglycerin; analgesics. Prepare for sx repair.

splenic injury

Suspect w/left lower rib fractures; LUQ pain to L. Shoulder (KEHR's sign). May see hours after injury d/t encapsulation.

Liver injury

Suspect w/right lower rib fractures; RUQ pain; Cullen's sign. Prepare for MTP & damage control surgery.

What is TEN 4 FACES?

Suspicious bruises in children {T}orso {E}ar {N}eck Under 4 years of age {F}renulum {A}uricular {C}heeks {E}yelid In infants under 4 years of age

What are heat cramps and how are they managed?

Sweat induced electrolyte depletion—>muscle cramps Tx w/rest in a cool environment & FLUID/ELECTROLYTE REPLACEMENT

What are the s/s of secondary syphilis?

Symmetric rash involving palms & soles; fever; chills; malaise; HA; lymphadenopathy

The earliest response to hypoglycemia is what?

Sympathetic stimulation—>cool, DIAPHORETIC skin; shallow respirations; dilated pupils; palpitations, tachycardia & shakiness

When is gastric lavage used?

Symptomatic patients w/RECENT INGESTION (<1hr) Risk for esophageal perforation.

What is depression?

Symptoms that interfere w/person's ability to work, sleep & actively participate in life S/s: loneliness; lack of energy; sleep disturbances; weight changes; decreased libido; decreased interest in usual activities. INQUIRE ABOUT RISK OF SUICIDE (ASK DIRECTLY)

How do you treat unstable tachycardia?

Synchronized cardioversion SYNC on "R" wave * Consider sedation prior if appropriate 50-200 joules biphasic adult or 0.5-1 joule/kg pediatrics

High levels of ADH can lead to?

Syndrome of Inappropriate Antidiuretic Hormone

What is it called when you do a REVIEW OF MULTIPLE STUDIES on specific topic to make PRACTICAL RECOMMENDATIONS based on findings? E.g. ACLS ALGORITHM

Systemic review & meta analysis

EKG change for ISCHEMIA

T wave INVERSION

What EKG change occurs with ischemia?

T wave inversion

What ekg changes are noted for HYPERkalemia?

TALL TENTED T WAVES initially then Widened complex Then SINE WAVE

What are the s/s of globe rupture & how is it managed?

TEAR DROP SHAPED PUPIL; visual disturbances; evisceration of aqueous/vitreous humor; DECREASED IOP Managed: SECURE PROTRUDING OBJECTS; DO NOT INSTILL TOPICAL MEDS; protect w/RIGID SHIELD; ophthalmology consult

What is intussusception?

TELESCOPING of one segment of bowel into another Most often seen in infants; males more common Episodic pain w/CURRANT JELLY STOOLS (necrotic tissue coming out) & blood mucus; infant consolable drawing up legs then periods of decreased LOC. Vomiting. May palpate SAUSAGE SHAPED MASS in RUQ.

Patient presents with severe stabbing pain in the temporal area & PALPABLE CORD-LIKE ARTERY is seen with what? How is it managed?

TEMPORAL ARTERITIS Tx w/STEROIDS

What diagnostic test is done for Myasthenia Travis?

TENSILON test

What is the most important thing to monitor w/epistaxis?

THAT AIRWAY

What is given to prevent WERNICKE'S ENCEPHALOPATHY?

THIAMINE prior to dextrose in malnourished

What happens immediately after emergent delivery?

THROUGHLY DRY & WARM (radiant warmer); gentle stimulation (rub back; tap foot); place newborn in SNIFFING POSITION; suction mouth then nose; clamp & cut cord between 30-60 secs s/p cord stopping pulsating Measure APGAR AT 1 & 5 MINS

What kind of splint is used for a scaphoid fracture?

THUMB SPICA SPLINT (splint w/thumb abduction)

Management of Sepsis in ED

TIME ZERO= time of triage Identify source; obtain lactic acid & blood cultures Rapid crystalloids bolus of 30ml/kg over 15-30mins if SBP <90, MAP <70, or lactate >4. Repeat until MAP >70 or if fluid compromised Give Broad spectrum ABX. Add vasopressor (initially w/norepinephrine) to maintain MAP >65

How is autonomic dysreflexia managed?

TREAT BY RELIEVING TRIGGER (impaction, full bladder, infection) & BP control

What is the treatment of HHS/HHNK?

TREAT HYPOVOLEMIA W/0.9%NS (lots); IV insulin infusion (if K+ >3.3) until BGL <300 (then switch to D5.45NS); replace electrolytes

What lab results do you see with a thyroid storm?

TSH LOW; T3 & T4 ELEVATED

What needs to be ruled out for Delirium?

TUMORS; MEDICATION USE; SEPSIS

What is the PRIMARY compensatory mechanism for LOW cardiac output in a PEDIATRIC patient?

Tachycardia

EKG changes in HYPOkalemia

Tachycardia; FLATTENED T waves; possible U wave; ventricular ectopy

What puts someone at higher risk for an Achilles' tendon rupture?

Taking FLUOROQUINOLONES (Cipro; Levaquin) especially if elderly

What is hypovolemic shock?

Tank. loss of circulating volume. Fill that pump that up and plug the leak

What is an aortic dissection?

Tear in the intimate layer of the aorta because of HTN; age >60; cocaine use; trauma (esp w/1st & 2nd rib fx); heart disease; connective tissue diseases (e.g. Marfan's & Ehlers Danlos) S/s: Sudden onset of TEARING, RIPPING PAIN to chest, shoulders, flank or back that is NOT relieved by analgesics DIFFERENCE of 20mm Hg in SBP between arms Dx: TTE or TEE, chest CT or MRI

What are the s/s of Bell's Palsy?

Tears; drooling; CAN'T BLINK/CLOSE AFFECTED EYE; FACIAL DROOPING; ipsilateral loss of taste; INCREASED SENSITIVITY TO SOUND (HYPERCUSIS)

intussusception

Telescoping of bowel. Infant INCONSOLABLE. Drawing up legs then periods of decreased LOC. Vomiting. May palpate sausage shaped mass & see CURRANT-JELLY (red mucus) stool. Air or barium enema is stable or surgical repair

Damage to what CN will impeded their ability of the patient to elevate/wrinkle skin of forehead?

Temporal branch of the CN VII (facial)

What is spinal shock?

Temporary loss of function to nervous tissue of the spine following trauma to the spinal cord S/s: loss of reflexes, flaccid paralysis; bowel/bladder dysfunction

What is a unilateral or bilateral DISPLACEMENT OF JAW d/t trauma; grinding teeth; yawning?

Temporomandibular joint (TMJ) dislocation

Inflammation of the TENDONS that connects muscle to bone that causes pain & loss of motion is known as?

Tendonitis (e.g. tennis elbow; golfers' elbow; rotator cuff)

What do you need to monitor for with an open pneumothorax?

Tension pneumothorax; remove dressing if signs of increased respiratory distress

In what phase does causes injuries from being thrown from the blast site-blunt force/CRUSH INJURIES, fractures, amputations, diffuse brain injuries?

Tertiary

Which worsens w/elevation- testicular torsion or epididymitis?

Testicular torsion

If your patient has not received their tetanus vaccine or they are unsure of when the initial vaccination was & they have gross contamination (e.g. migrant farm worker) what would you administer?

Tetanus toxoid + 250 UNITS OF IM IMMUNOGLOBULIN (TIG)

What is the Preeclampsia Triad?

The PRE: Proteinuria Rising BP Edema

What controls the release of epinephrine for fight-or-flight response?

The adrenal glands

stroke volume

The amount of blood ejected by left ventricle in one contraction. = Preload, afterload, & contractility

Where does decerebrate posturing come from?

The brainstem

Where does Decorticate posturing come from?

The cerebrum.

What is Herpes Simplex?

The gift that keeps on giving. Incurable w/exacerbations S/s: burning; PAINFUL VESICLES on genitalia and flu-like symptoms Tx: ANTIVIRALS (valtrex)

Your patients has a positive PREHN's sign, what does that mean? And what is that an indication of?

The lifting of the testicles relieves the pain. Indication of epididymitis.

You have a patient who presents with s/o pyloric stenosis; one with intussusception & one with volvulus. Who is the HIGHEST PRIORITY?

The patient with the symptoms of volvulus.

What is distributive shock?

The pipe/hose has failed. There is a maldistribution of blood. - anaphylactic: epinephrine IM - septic: fluids, ABX, & vasopressors -neurogenic: hypotension & bradycardia

Why must the patient be HEMODYNAMICALLY STABLE to use NIPPV?

The pressure decreases venous RETURN TO THE HEARt which can lower the BP & cardiac output

2 adult; 3 peds; 4 electrical means what in regards to fluid resuscitation for burns?

The recommendation set by the American Burn Association for resuscitation guidelines: Begin LR at 2ml/kg x TBSA for (adult) thermal burns 3ml/kg x TBSA for pediatrics 4ml/kg x TBSA for electrical burns

What is a prolapsed cord and how is it managed?

The umbilical cord precedes the neonate in the vaginal canal; place the mother in KNEE TO CHEST POSITION; insert a sterile gloved into the vagina to ELEVATE THE PRESENTING PART off the cord & leave hand in place; wrap EXPOSED CORD W/SALINE GAUZE; NEED IMMEDIATE C-SECTION.

What do Hemophilia A & B have in COMMON?

They are recessive sex-linked; occur in males

What do you add to prevent Wernicke's Encephalopathy?

Thiamine (Vitamin B1) 50-100mg (if malnourished)

Diptheria

Thick, gray PSEUDOMEMBRANOUS coating on tonsils & pharynx Standard & droplet precautions.

What are the s/s of a femur fracture?

Thigh pain & inability to bear weight; shortening of the affected leg; internal/external rotation Associated urethral damage

The doctor takes a syringe a ice cold water & irrigates the the left ear; the eyes stay stationary. What OCULOVESTIBULAR reflex was tested?

This "cold caloric" test checks for the eyes to look toward the ear that is being irrigated. Since the eyes remained stationary that indicates brain death. Womp womp. Dd

Who is at risk for Cholecystitis?

Those 5 Fs- FAT, 40, FERTILE, FEMALE, & FAIR

What type of abortion is note to have the cervical os CLOSED?

Threatened

What are the types of spontaneous abortion?

Threatened; inevitable.

How do you treat PE?

Thrombolytics

How is a scaphoid fracture managed?

Thumb spica splint; casting; surgery

To test the radial nerve what motion do you instruct the patient to do?

Thumbs up

Dangerously high thyroid levels lead to what?

Thyroid storm

What concomitant fractures can be seen with calcaneus fracture?

Tibial plateau fractures & compression fractures of the vertebrae

What is the intervention implemented to avoid inferior vena cava syndrome or supine hypotensive syndrome?

Tilt on side

Why do you rewarm the core body temp prior to the periphery?

To PREVENT REWARMING SHOCK (VFib)

What is the priority for an inhalation injury?

To maintain a patent airway

Why might hydralazine be use for a patient w/autonomic dysreflexia?

To treat the HTN secondary to the autonomic dysreflexia

Seizure that causes erratic electrical activity t/o brain but also involves gross abnml muscular activity is what?

Tonic-clinic

What is polycythemia?

Too many blood cells

A compacted fracture that leaves the cortical margins of the bone intact is what?

Torus

What are pediatric specific fractures?

Torus fractures (cortical/buckle) that occurs w/axial loading injuries Green stick fractures (occur when bones bend & splinter) Salter-Harris fractures (epiphyseal-metaphysical) are through the growth plate.

A fracture that runs straight across the bone is what?

Transverse

What does Brown-sequard syndrome involve? (Remember 2 WORDS - 2 SIDES)

Transverse hemisection (frequently d/t stab or GSW) Loss of motor function on SIDE OF INJURY (I.e. ipsilateral) Loss of pain & temperature on OPPOSITE side of injury (I.e. contralateral)

What are the causes of a ruptured TM?

Trauma (e.g basilar skull fx); infection; BAROTRAUMA (e.g. BLAST/EXPLOSION INJURY; flying; diving)

How is DKA managed in the ED?

Treat HYPOVOLEMIA shock w/0.9% NS; IV insulin infusion (no bolus) if K+ 3.5 or > until BGL <250 (then add dextrose). Replace electrolytes (POTASSIUM); treat w/sodium bicarbonate only if pH <7.0 despite IVF & insulin. Continue insulin until anion gap closes.

What is the "C BIG K DROP"?

Treatment for hyperkalemia {C}alcium gluconate 10%/10ml IV over 10mins- cardiac stabilizer {B}eta agonists-salbuterol 10-20mg in 4ml NS nebulized over 10mins OR Bicarbonate 8.4% (50meq) over 5mins (intracellular shift) {I}nsulin 10units IVP-intracellular shift {G}lucose D50W 1amp over 5mins-maintain glucose {K}ayexylate 15-30g IN 15-30ml (70% sorbitol) PO for GI removal (for chronic renal failure) {D}iuretics-lasix 40-80mg IVP (renal removal) {rop}-renal unit for extracorporeal removal

What is Shaken impact syndrome?

Triad of subdural hematoma; fractured posterior ribs; retinal hemorrhage Seen in children & interpersonal violence

Lethargy, delirium, coma, seizures, respiratory arrest, PROLONGED PR & QTi, tachycardia, VT/VF, hypotension, mydriasis (pupil dilation), flushed skin, hyperthermia are all seen in what OD? How is it managed?

Tricyclic antidepressants Secure that airway early. Activated charcoal. Tx dysrhythmias w/LIDOCAINE & SODIUM BICARBONATE. Fluid resuscitation. Vasopressors. Benzos (seizure)

What disorder involves CN V & comes with severe pain?

Trigeminal neuralgia

Compression of CN V from tumor; AV malformation; trauma or MS is called?

Trigeminal neuralgia (tic doloreux)

Sickle Cell Anemia Crisis

Triggered by stress, cold, high altitude, dehydration, or infection. Tx: HIGH FLOW O2, IVF (dilutes blood & aides in pain control); narcotics (vasodilates); pRBCs

What is a migraine?

Triggers Unilateral; photo/phonophobia; N/V; POSSIBLE AURA

Inflated BP cuff that leads to carpopedal spasm is known as? What is it seen with?

Trousseau's sign Hypocalcemia; pancreatitis

What are the NEUROGENIC causes of SIADH?

Tumors of the hypothalamus or pituitary gland Head injuries Meningitis or encephalitis Cerebral aneurysms

After 20 weeks what can be done to avoid supine hypotension?

Turn patient 15 degrees

How do you treat vena cava syndrome?

Turn that pregnant lady on her left side; laying supine at 15-30degrees supine

What is testicular torsion?

Twisting of the testicle or spermatic cord; most common in infants & during puberty (12-18) Rapid onset of severe unilateral inguinal pain (usually wakes them up at night); pain worse w/elevation or ice ; ABSENT OF CREMASTERIC REFLEX; scrotal redness; swelling; elevation on affected side. Dx: U/S will show ischemia Tx: manual detorsion or surgical repair

What is chvostek's sign characterized by?

Twitching of the lip to spasm of all facial muscles

How do you treat bradycardia?

Tx the underlying cause (if it's respiratory assist breathing w/BVM!) Atropine 0.5mg IVP Q3-5mg for low degree blocks (up to 3mg) Transcutaneous pacing Epinephrine infusion 2-10mcg/min Dopamine infusion 2-20mcg/kg/min Pacemakers for refactory bradycardia or HBs

How is DIC managed?

Tx underlying cause; control bleeding; antifibrinolytic agents (aminocaproic acid- Amicar & tranexamic acid- TXA), platelet transfusion

How do you treat EXTRAPYRAMIDAL symptoms & how do you know when it's effective?

Tx: DIPHENHYDRAMINE; benztropine (cogentin); trihexyphenidyl (artane) Effective: WHEN MUSCLES RELAX

When you are percussing the abdomen, what sound should be heard over the HOLLOW organs?

Tympany (hyperresonance)

Why type of disorder do you need to replace FACTOR VIII in?

Type A Hemophilia Von Willebrand

Anterior MI

Typically LAD. S/s: CRUSHING CP; tachycardias; ventricular dysrhythmias; feeling of impending doom; BBB, dyspnea; crackles in lungs & S3 heart sound (cardiogenic shock) Difficult to detect in L. BBB

Inferior MI

Typically RCA Epigastric pain, bradycardia, hypotension, 2nd degree HB type 1. Associated w/RV MI (proximal occlusion; do R.sided EKG-ST elevation present in V4)

What is a zygomatic fracture & how is it managed?

Typically seen w/orbital wall fracture S/s: trismus. (Reduce ability to open jaw r/t muscle spasm); infraorbital hyperthermia (abnormal loss of sensation to heat, cold, touch, or pain); diplopia (double vision); epistaxis; asymmetry; LOSS OF CHEEKBONE (MALAR) EMINENCE (what makes it look like a cheekbone) Managed: elevated HOB; ice pack (not chemical); EENT consult; educate on ophthalmic f/u; ice packs to face; avoid VALSALVA MANEUVER, STRAINING, & BLOWING NOSE

Patient presents with c/o abdominal distention; cramping in LLQ; DIARRHEA W/BLOOD; toxic megacolon (LARGE INTESTINE). What is going on and how will you treat?

ULCERATIVE COLITIS. Fluid resuscitation; antidiarrheal; anti-inflammatories; corticosteroids; immunosuppressants; surgery

In an electrical burn what is the most important thing to evaluate for in regards to fluid resuscitation and why?

URINE OUTPUT d/t the excretion of myoglobin

What s/s of Gonorrhea?

UTI SYMPTOMS & MUCOID DISCHARGE

You tell your patient to give a "peace out" motion to test what nerve?

Ulnar nerve.

A sunburn of the cornea caused by sunlight, snow blindness or welder's flash that is typically not noticed for hours is known as?

Ultraviolet keratitis

What is Cullen's sign?

Umbilical ecchymosis

What are the types of herniation?

Uncal/transtentorial Central

If the pH values on the ABG are <7.35 or >7.45 its said to be?

Uncompensated

How do you determine the ETT size for peds?

Uncuffed: (age/4) + 4 Cuffed: (age/4) + 3.5

What should be suspected w/rib fractures?

Underlying injury (e.g. pulmonary contusion & pneumothorax) Aortic disruption (1st-2nd rib fx) Liver injury (R. Lower ribs fx) Spleen injury (L. Lower rib fx)

Pediatric patient presents w/rib fractures in the ED what else should be considered?

Underlying thoracic (I.e. pulmonary contusion) & abdominal solid-organ injuries Child maltreatment

How do we manage suicide?

Undress patient & remove anything that may pose danger; ENCOURAGE COMMUNICATION; involve family/friends; look for positives in life; DEMONSTRATE WORTH

How do you use the stairs w/crutches?

Uninjured first ascending (going up) Injured first descending (going down)

neurogenic shock

Unopposed parasympathetic; bradycardia; hypotensive.

pertussis

Unremitting paroxysmal bursts of coughing. Standard & droplet precautions

What is Eclampsia? How is it managed?

Untreated preeclampsia + occurrence of tonic-clonic seizures (w/no prior hx) BENZODIAZEPINES; antihypertensives; reduce stimulus & dim lights.

What are the s/s of testicular torsion?

Upwardly retracted testes w/scrotal redness; swelling; abdominal pain; nausea; vomiting; ABSENT CREMASTERIC REFLEX

What are common causes of urinary retention?

Urethral strictures; enlarged prostate, renal calculi; neurogenic bladder; SIDE EFFECTS OF ANTIHISTAMINES (parasympathetic effect-contracts bladder- of OTC cold medicines)

what are common causes of urinary retention?

Urethral strictures; enlarged prostate; renal calculi; neurogenic bladder; side effect of ANTIHISTAMINES (OTC NASAL DECONGESTANTS)

What should you suspect if blood is present at the urethral meatus?

Urethral trauma.

What is akathisia?

Urge to move constantly

What are s/s of prostatitis? How is it treated?

Urinary urgency, frequency & dysuria; hematospermia; tender prostate; elevated PSA Collect a U/A; ABX (fluroquinolones); alpha-blockers

How is a nasal foreign body managed in ED?

Use LEAST INVASIVE MEANS POSSIBLE DECONGESTANTS OR PRESSOR AGENT PRIOR TO REMOVAL TO DECREASE SWOLLEN TISSUE Occlude unaffected nostril & ask child to BLOW nose or have parent blow in mouth or use BVM WALL SUCTION Forceps as a last resort

What education needs to be provided regarding a cane?

Use on side opposite of injury. Fit to keep elbow at 30 degree angle flexion

Transcutaneous Pacing

Used for hemodynamically significant bradycardia. Apply pads anterior/lateral or anterior/posterior. Do NOT place over pacemaker/AICD or medication patches. Activate pacemaker at rate of 70-80. SLOWLY increase output in milliamperes until capture acquired. Evaluate electrical capture (Q pacer spike should be f/b by WIDE QRS) & mechanical capture (palpate FEMORAL pulse or by u/s)

Acute severe abdominal pain, vaginal bleeding, FETAL PARTS PALPATED OUTSIDE OF UTERUS, fetal distress, & maternal shock that is rarely seen but can occur in blunt trauma & typically results in fetal demise is seen with? How is it managed?

Uterine rupture Blood transfusion; prepare for emergency c-section & hysterectomy.

What is the benefit of the majority?

Utilitarianism E.g. disaster triage.

Anterior MI leads and artery

V1-V4; LAD

Posterior MI leads

V7-V9 V1-V3 should remain unchanged from standard ECG; changes w/MI in V1-V3 include: horizontal ST depression; Tall broad R waves; Upright T waves; Dominant R wave (R/S >1) in V2 V7-L.posterior auxiliary line V8-tip of L. Midscapula V9- L paraspinal region

Right ventricular MI

VR4 at 5th intercostal space & midclavicular line. Cautious w/PRELOAD reducing meds like NTG & MSO4. Give IVF 250ml bolus or Dobutamine for hypotension. Improved if BP increasing.

What are treatments/medications for STABLE (I.e. alert & NML BP) tachycardia?

Vagal maneuvers (NARROW COMPLEX)- blow in 10ml syringe to move plunger Adenosine 6mg rapid IVP w/flush (slows SA & AV node conduction) for narrow SVT or MONOMORPHIC WIDE complex VT; repeat w/12mg only if effective CCB or BB for stable SVT Amiodarone 150mg for stable VT LIdocaine for PVCs & stable VT (esp if d/t prolonged QTi) Procainamide 20-50mg/min up to 17mg/kg or Sotalol for stable VT (monitor for HYPOTENSION & PROLONGED QT)

Stable Tachycardia

Vagal maneuvers for NARROW complex Adenosine 6mg RAPID IVP w/flush to SLOW SA & AV node conduction for narrow SVT or monomorphic wide-complex VT; repeat w/12mg only if effective Amiodarone 150mg SLOW (stable wide-complex) or Lidocaine (if d/t prolonged QTi) or procainamide 20-50mg/min (max of 17mg/kg) or sotalol

What is chicken pox?

Varicella. Infectious for 48 HOURS before rash appears. CONTAGIOUS UNTIL ALL LESIONS ARE CRUSTED OVER. S/s: purulent VESICULAR RASH that starts ON TRUNK; fever; pruritus; urticaria Tx: symptomatic care; antiviral agents; ANTIHISTAMINES; antipyretics

What is nitroglycerin?

VasoDILATOR. Coronary artery DILATOR. Decreases PRELOAD & AFTERLOAD Contraindicated w/in 24hrs of phosphodiesterase inhibitor use (erectile dysfx drugs like viagra, cialis, levitra)

What medication should be given to a brain dead patient while awaiting for the organ procurement team?

Vasopressin

What increases afterload?

Vasopressor

An MRI is used to detect what in the skin/wound?

Vegetative (biologic) material

What type of foreign object needs to be removed from a wound ASAP WITHOUT SOAKING (or it swells)?

Vegetative material (e.g. thorn)

What are the s/s of a subdural hematoma?

Venous bleed; progressively decreasing LOC

You adjust the tidal volume first then the rate in what type of problem?

Ventilation (hypercarbic- CO2 retention/elevated CO2 levels in blood) problems

What are the s/s of a blunt cardiac injury? How is it managed?

Ventricular dysrhythmias, CP, chest wall ecchymosis Monitor cardiac rhythm; tx dysrhythmias; administer analgesics

What happens in a vasoocclusive crisis & how is it managed?

Vessels occluded—>ischemia; pain; swelling—>severe pain & priapism Tx w/O2, IVFs & narcotics

What are possible causes for MALLORY-WEISS SYNDROME?

Violent retching w/etoh or BULIMA; ASA use; heavy lifting

What is bronchiolitis?

Viral infection; usually RSV in premature infants Produces COPIOUS NASAL SECRETIONS S/s: respiratory distress; WHEEZING; CRACKLES; grunting; poor feeding Dx: nasopharyngeal culture to r/o influenza; CXR Tx: SUCTION those nares; bronchodilators; admission if RR >70 breaths/min

What is acute bronchitis?

Viral inflammation due to overuse of ABX S/s: NP cough for > 5 days; airway hyperreactivity; pleuritic CP Dx: CXR to r/o PNA Tx: cough medicine, bronchodilators, corticosteroids

What is PNA

Viral/Bacterial (faster onset) Community/hospital acquired S/s: fever; pleuritic CP; productive cough; increased FREMITUS (vibration) over affected area w/decreased BS Dx: increased WBC; CXR; blood cultures Tx: ABX; bronchiodilators; O2 & fluids as indicated

Stasis, vein damage & hypercoagulation is know as? And it is seen with?

Virchow's Triad DVT

HIV/AIDS

Virus DESTROYS CD4 T cells (critical to fighting diseases) Primary mode of transmission is person to person through unprotected sex *PrEP lowers chances of getting infected S/s: fever, fatigue, swollen lymph glands Tx: antiretroviral therapy (prevent virus from REPLICATING) During advanced stage is when OPPROTUNISTIC infections occur.

What increases Preload?

Volume

What INCREASES preload?

Volume & vasoCONSTRICTORS Tx: diuretics

What causes preload to increase?

Volume excess (pulmonary edema) or VASOCONSTRICTION Tx: diuretics & vasodilators (NTG)

What is characteristic of SIADH?

Volume overload but are HYPONATREMIC

SIADH

Volume overload w/HYPONATREMIA. Risk of seizures

What presents with BILIOUS EMESIS most of the time during the 1st month of life?

Volvulus

hypovolemic hyponatremia causes

Vomiting, diarrhea, or burns tx'd w/0.9% NS replacement

What are common causes of Metabolic acidosis?

Vomiting; hypokalemia; gastric suctioning; diuretics; sodium bicarbonate

How does hydrocephalus VP shunt obstruction present? How is it managed?

Vomiting; sleepiness; fever; "sun-setting" eyes; seizures; erythema along shunt tract; meningeal & peritoneal signs Head circumferences; shunt series for dx Increase HOB to 30 degrees; drain shunt; administer antipyretics, diuretics, anticonvulsants & ABX

What hemophilia can affect both men & women?

Von Willebrand

What is trichomonas vaginalis?

Vulvar irritation; DYSPAREUNIA (painful intercourse); "FISHY" odor; yellow-GREEN DISCHARGE Dx: wet prep Tx: Flagyl once or Tinidzaole

SIRS is the presence of 2 or more of...?

WBC >12K (> bands) or <4K Temperature >38C (100.4F) or <36C (96.8F) HR >90BPM RR > 20BPM

What are examples of common pediatric medication errors?

WEIGHT BASED ERRORS; limited dose strengths available.

What breath sound is associated w/a lower airway obstruction?

WHEEZING

What is the treatment for "BODY PACKERS"?

WHOLE BOWEL IRRIGATION Surgical removal

What is Cushing's triad?

WIDENED pulse pressure; bradycardia, irregular breathing pattern Indicative of INCREASED ICP

In critical incident stress management when should DEBRIEFING occur & what happens during it?

WITHIN 24-72 HRS s/p event to discuss staff's EMOTIONAL RESPONSES & SYMPTOMS THEY ARE EXPERIENCING.

Loss of motor & sensory function that is more pronounced in arms than legs is seen in what?

Walk to the bar but can't drink Central cord incomplete SCI

What medication should you NOT take CoQ10 with?

Warfarin

What are the s/s of neurogenic shock?

Warm, FLUSHED SKIN w/FULL PULSES; hypotension; BRADYCARDIA (or lack of tachycardia-normally seen initially in shock)

Dehydration

Water depletion D/t V/D, DI, DKA, diuretic use, burn injury Tx: isotonic fluid resuscitation (NS/LR) 1-2 L in adults; repeat as needed; 20ml/kg boluses in peds

What are the s/s of an intestinal obstruction &how is it managed?

Wave like abdominal pain; fever; s/o shock; loud high pitched to absent bowel sounds Fluid & electrolyte replacement to treat HYPOVOLEMIC SHOCK; NG for decompression ; ABX; prepare for sx

Pronator drift is a way to assess for what?

Weakness to one side of the body

What d/c education should be provided to a patient w/bell's?

Wear sunglasses/eye protection to help w/eye irritations; moist heat from humidifier; ARTIFICIAL TEARS during awake hours; FACIAL MASSAGE (to prevent permanent contractures/paralysis)

What is pulsus paradoxus?

What the BP decreases w/inspiration

What is pertussis?

Whooping cough Highly contagious. Limits ability to clear secretions Incubation period 7-10 days. 3 stages Dx'd w/Dacron swab in posterior nasopharynx Tx: supportive care; MACROLIDE ABX; antitussives; antipyretics; tx family w/ABX; pertussis vaccination

What is a diagnostic finding that points to a traumatic aortic dissection?

Widened mediastinum & obscured aortic knob on CXR

What will you see on films that will dx an aortic dissection?

Widened mediastinum or obscured aortic knob

When do you give Adenosine?

With NARROW SVT or MONOMORPHIC WIDE COMPLEX VT; start w/6mg & give rapidly IVP and give w/flush to slow SA & AV node conduction; repeat w/12mg ONLY if EFFECTIVE.

When is STRIDOR seen?

With UPPER airway obstruction- croup; epiglottis; anaphylaxis

What education would you provide a patient who had a wound repaired w/a wound bonding agent?

Wound bonding agents leave a waterproof bandage that will SLOUGH OFF in 5-10 days Avoid petroleum ointments

Fundal height just below what indicates term fetus?

Xiphoid

Who is risk for Heat stroke?

Young & elderly; those who take thyroid meds, sympathomimetics; haldol, antihistamines and/or anticholinergics

How do you find the MAP?

[(2 x DBP) + SBP] /3

MAP

[(2xDBP) + SBP] /3

What are the unipolar leads?

aVR, aVL, aVF

What is croup?

acute laryngotracheobronchitis Viral infection S/s: gradual onset of URI; BARKY, SEAL LIKE COUGH; inspiratory stridor; low grade fever; STEEPLE SIGN on CXR Tx: humidification & RACEMIC EPINEPHRINE (wears off so consider admitting)

What is the ABCDEFG-LMNOP-HIJ of the initial assessment?

{A}cross the room assessment; Airway & Alertness w/simultaneous C-spine stabilization {B}reaching & ventilation {C}irculation & Control of bleeding {D}isability Da Brain {E}xposure & environmental control {F}ull set of VS & Family presence {G}et adjuncts & give comfort {L}abs {M}onitors {N}asogastric/orogastric consideration {O}xygenation & ventilation (oximetry); ETCO2 {P}ain assessment/management {H}istory & H-T assessment {I}nspect posterior surfaces {J}ust keep reevaluating

What is DKA?

{D}iabetic BGL 250-600; {K}etones; {A}cidosis- pH < 7.35 Most common in type 1 Develops RAPIDLY

How do you diagnosis a problem & how do you fix it regarding the intubate patient? (DOPES) & (DOTTS)

{D}isplaced ETT {O}bstructed ET tube {P}neumothorax {E}quipment malfunction {S}tacked breaths {D}isconnect from vent {O}xygen w/100% BVM while looking, listening, & feeling {T}ube position & function {T}weak the ventilator (decrease rate, tidal volume, & inspirations time {S}onography

What is HELLP syndrome? How is it managed?

{H}emolysis {E}levated {L}iver enzymes {L}ow {P}latelets Blood administration & emergent c-section.

What is the LMNOPQR of appendicitis?

{L}eukocytosis (>10k w/bands); Loss of appetite {M}arkle sign (heel jar) & McBURNEY'S point pain (RLQ) {N}/V {O}bturator sign (pain on R. Hip flexion {P}eriumbilical pain (early) {Q}uality-dull {R}ovsing sign (Rebound tenderness)

What are common causes of metabolic acidosis? MUDDLERS

{M}ethanol ingestion {U}remia {D}KA {D}iarrhea (if it comes out of you ASS it's ACIDOSIS) {L}actic acidosis {E}thylene glycol ingestion {R}habdomyolysis {S}alicylates

What are the s/s of zygomatic fractures? (TIDES)

{T}rismus {I}nfraorbital hypesthesia {D}iplopia {E}pistaxis Not {S}ymmetrical Loss of Malar eminence (lost those cheekbones)


Conjuntos de estudio relacionados

Computer Concepts Module One: Impact of Digital Literacy

View Set

Big ideas math geometry- Chapter 9 vocab

View Set

Remembering Through Repetition Quiz (Academic Success)

View Set

Lesson 8 - Influence of Independence

View Set

Chapter 2: RF Characteristics and Behavior

View Set

Professional Liability Errors & Omissions

View Set